eMRCS Pathology 326 MCQs

You might also like

Download as pdf or txt
Download as pdf or txt
You are on page 1of 101

1. A 38 year old lady presents with a recent episode of renal colic.

c. As part of her investigations the following results are


obtained: Corrected Calcium: 3.84 mmol/l; PTH: 88pg/ml (increased). Her serum urea and electrolytes are normal. What is
the most likely diagnosis?
A. Carcinoma of the bronchus
B. Secondary hyperparathyroidism
C. Primary hyperparathyroidism
D. Tertiary hyperparathyroidism
E. Carcinoma of the breast
Answer: C
In this situation the most likely diagnosis is primary hyperparathyroidism. The question mentions that serum urea and electrolytes
are normal, which makes tertiary hyperparathyroidism unlikely.
In exams primary hyperparathyroidism is stereotypically seen in elderly females with an unquenchable thirst and an inappropriately
normal or raised parathyroid hormone level. It is most commonly due to a solitary adenoma
Causes of primary hyperparathyroidism: 80%: solitary adenoma; 15%: hyperplasia; 4%: multiple adenoma; 1%: carcinoma
Features - 'bones, stones, abdominal groans and psychic moans' Polydipsia, polyuria; Peptic ulceration/constipation/pancreatitis;
Bone pain/fracture; Renal stones; Depression; Hypertension. Associations; Hypertension; Multiple endocrine neoplasia: MEN I and
II
Investigations: Raised calcium, low phosphate; PTH may be raised or normal; Technetium-MIBI subtraction scan. Treatment;
Parathyroidectomy, if imaging suggests target gland then a focused approach may be used

Theme: Head and neck lumps

A. Branchial cyst
B. Cystic hygroma
C. Carotid body tumour
D. Lymphadenopathy
E. Adenolymphoma of the parotid
F. Pleomorphic adenoma of the parotid
G. Submandibular tumour
H. Thyroglossal cyst
I. Thoracic outlet syndrome
J. Submandibular gland calculus
Please select the most likely lesion to account for the clinical scenario given. Each option may be used once, more than once or not
at all.

2. A 60 year old Tibetan immigrant is referred to the surgical clinic with a painless neck swelling. On examination it is located
on the left side immediately anterior to the sternocleidomastoid muscle. There are no other abnormalities to find on
examination.
Answer: The correct answer is Carotid body tumour
Carotid body tumours typically present as painless masses. They may compress the vagus or hypoglossal nerves with symptoms
attributable to these structures. Over 90% occur spontaneously and are more common in people living at high altitude. In familial
cases up to 30% may be bilateral. Treatment is with excision.

3. A 40 year old women presents as an emergency with a painful mass underneath her right mandible. The mass has appeared
over the previous week with the pain worsening as the lump has increased in size. On examination there is a 4cm mass
underneath her mandible, there is no associated lymphadenopathy.
Answer: Submandibular gland calculus
The sub mandibular gland is the most common site for salivary calculi. Patients will usually complain of pain, which is worse on
eating. When the lesion is located distally the duct may be laid open and the stone excised. Otherwise the gland will require
removal.

1
4. A 73 year old male smoker is referred to the clinic by his GP. On examination he has a 3cm soft mass immediately anterior
to his ear. It has been present for the past five years and is otherwise associated with no symptoms.
Answer: Adenolymphoma of the parotid
Warthins tumours (a.k.a. adenolymphoma) are commoner in older men (especially smokers). They are the second commonest
benign tumour of the parotid gland, they may be bilateral. They are soft and slow growing and relatively easy to excise.
Pleomorphic adenomas typically present in females aged between 40 - 60 years.

The table below gives characteristic exam question features for conditions causing neck lumps:

Reactive By far the most common cause of neck swellings. There may be a history of local infection or a
lymphadenopathy generalised viral illness
Lymphoma Rubbery, painless lymphadenopathy
The phenomenon of pain whilst drinking alcohol is very uncommon
There may be associated night sweats and splenomegaly
Thyroid swelling May be hypo-, eu- or hyperthyroid symptomatically
Moves upwards on swallowing
Thyroglossal cyst More common in patients < 20 years old
Usually midline, between the isthmus of the thyroid and the hyoid bone
Moves upwards with protrusion of the tongue
May be painful if infected
Pharyngeal pouch More common in older men
Represents a posteromedial herniation between thyropharyngeus and cricopharyngeus muscles
Usually not seen, but if large then a midline lump in the neck that gurgles on palpation
Typical symptoms are dysphagia, regurgitation, aspiration and chronic cough
Cystic hygroma A congenital lymphatic lesion (lymphangioma) typically found in the neck, classically on the left side
Most are evident at birth, around 90% present before 2 years of age
Branchial cyst An oval, mobile cystic mass that develops between the sternocleidomastoid muscle and the pharynx
Develop due to failure of obliteration of the second branchial cleft in embryonic development
Usually present in early adulthood
Cervical rib More common in adult females
Around 10% develop thoracic outlet syndrome
Carotid aneurysm Pulsatile lateral neck mass which doesn't move on swallowing

5. A 12 year old child is admitted with a 12 hour history of colicky right upper quadrant pain. On examination the child is
afebrile and is jaundiced. The abdomen is soft and non tender at the time of examination. What is the most likely cause?
A. Infectious hepatitis
B. Acute cholecystitis
C. Cholangitis
D. Hereditary spherocytosis
E. Gilberts syndrome
Answer: D
The child is most likely to have hereditary spherocytosis. In these individuals there may be disease flares precipitated by acute
illness. They form small pigment stones. These may cause biliary colic and some may require cholecystectomy.

Hereditary Spherocytosis: Most common disorder of the red cell membrane, it has an incidence of 1 in 5000. The abnormally
shaped erythrocytes are prone to splenic sequestration and destruction. This can result in hyperbilirubinaemia, jaundice and
splenomegaly. In older patients an intercurrent illness may increase the rate of red cell destruction resulting in more acute
symptoms.
Severe cases may benefit from splenectomy.

2
6. A 2 day old baby is noted to have voiding difficulties and on closer inspection is noted to have hypospadias. Which of the
following abnormalities is most commonly associated with the condition?
A. Cryptorchidism
B. Diaphragmatic hernia
C. Ventricular - septal defect
D. Bronchogenic cyst
E. Atrial septal defect
Answer: A
Hypospadias most commonly occurs as an isolated disorder. Associated urological abnormalities may be seen in up to 40% of
infants, of these cryptorchidism is the most frequent (10%).
Hypospadias: The urethral meatus opens on the ventral surface of the penis. There is also a ventral deficiency of the foreskin. The
uretral meatus may open more proximally in the more severe variants. However, 75% of the openings are distally located. The
incidence is 1 in 300 male births.

Features include: Absent frenular artery; Ventrally opened glans; Skin tethering to hypoplastic urethra; Splayed columns of
spongiosum tissue distal to the meatus; Deficiency of the foreskin ventrally
Management: No routine cultural circumcisions; Urethroplasty; Penile reconstruction. The foreskin is often utilised in the
reconstructive process. In boys with very distal disease no treatment may be needed.

Theme: Liver lesions

A. Cystadenoma
B. Hyatid cyst
C. Amoebic abscess
D. Mesenchymal hamartoma
E. Liver cell adenoma
F. Cavernous haemangioma

Please select the most likely lesion for the scenario given. Each option may be used once, more than once or not at all.

7. A 38 year old lady presents with right upper quadrant pain and nausea. She is otherwise well and her only medical therapy
is the oral contraceptive pill which she has taken for many years with no ill effects. Her liver function tests are normal. An
ultrasound examination demonstrates a hyperechoic well defined lesion in the left lobe of the liver which measures 14 cm in
diameter.
Answer: Cavernous haemangioma
Cavernous haemangioma often presents with vague symptoms and signs. They may grow to considerable size. Liver function tests
are usually normal. The lesions are typically well defined and hyperechoic on ultrasound. A causative link between OCP use and
haemangiomata has yet to be established, but is possible.

8. A 37 year old lady presents with right upper quadrant pain and nausea. She is otherwise well and her only medical therapy
is the oral contraceptive pill which she has taken for many years with no ill effects. Her liver function tests and serum alpha
feto protein are normal. An ultrasound examination demonstrates a 4cm non encapsulated lesion in the right lobe of the
liver which has a mixed echoity and heterogeneous texture.
Answer: Liver cell adenoma
Liver cell adenomas are linked to OCP use and 90% of patients with liver cell adenomas have used the OCP. Liver function tests are
often normal. The lesions will typically have a mixed echoity and heterogeneous texture.

9. A 38 year old shepherd presents to the clinic with a 3 month history of malaise and right upper quadrant pain. On
examination he is mildly jaundiced. His liver function tests demonstrate a mild elevation in bilirubin and transaminases, his
full blood count shows an elevated eosinophil level. An abdominal x-ray is performed by the senior house officer and
demonstrates a calcified lesion in the right upper quadrant of the abdomen.
Answer: Hyatid cyst

3
Hyatid disease is more common in those who work with sheep or dogs. Liver function tests may be abnormal and an eosinophilia is
often present. Plain radiographs may reveal a calcified cyst wall.

Benign liver lesions


Haemangioma: Most common benign tumours of mesenchymal origin. Incidence in autopsy series is 8%. Cavernous
haemangiomas may be enormous. Clinically they are reddish purple hypervascular lesions. Lesions are normally separated from
normal liver by ring of fibrous tissue. On ultrasound they are typically hyperechoic
Liver cell adenoma: 90% develop in women in their third to fifth decade. Linked to use of oral contraceptive pill. Lesions are
usually solitary. They are usually sharply demarcated from normal liver although they usually lack a fibrous capsule. On ultrasound
the appearances are of mixed echoity and heterogeneous texture. On CT most lesions are hypodense when imaged prior to
administration of IV contrast agents. In patients with haemorrhage or symptoms removal of the adenoma may be required
Mesenchymal hamartomas: Congential and benign, usually present in infants. May compress normal liver
Liver abscess: Biliary sepsis is a major predisposing factor. Structures drained by the portal venous system form the second largest
source. Common symptoms include fever, right upper quadrant pain. Jaundice may be seen in 50. Ultrasound will usually show a
fluid filled cavity, hyperechoic walls may be seen in chronic abscesses
Amoebic abscess: Liver abscess is the most common extra intestinal manifestation of amoebiasis. Between 75 and 90% lesions
occur in the right lobe. Presenting complaints typically include fever and right upper quadrant pain. Ultrasonography will usually
show a fluid filled structure with poorly defined boundaries. Aspiration yield sterile odourless fluid which has an anchovy paste
consistency. Treatment is with metronidazole
Hyatid cysts: Seen in cases of Echinococcus infection. Typically an intense fibrotic reaction occurs around sites of infection. The
cyst has no epithelial lining. Cysts are commonly unilocular and may grow to 20cm in size. The cyst wall is thick and has an
external laminated hilar membrane and an internal enucleated germinal layer. Typically presents with malaise and right upper
quadrant pain. Secondary bacterial infection occurs in 10%. Liver function tests are usually abnormal and eosinophilia is present in
33% cases. Ultrasound may show septa and hyatid sand or daughter cysts. Percutaneous aspiration is contra indicate Treatment is by
sterilisation of the cyst with mebendazole and may be followed by surgical resection. Hypertonic swabs are packed around the cysts
during surgery
Polycystic liver disease: Usually occurs in association with polycystic kidney disease. Autosomal dominant disorder. Symptoms
may occur as a result of capsular stretch
Cystadenoma: Rare lesions with malignant potential. Usually solitary multiloculated lesions. Liver function tests usually normal.
Ultrasonography typically shows a large anechoic, fluid filled area with irregular margins. Internal echos may result from septa.
Surgical resection is indicated in all cases

10. A 72 year old man presents with symptoms and signs of benign prostatic hyperplasia. Which of the following structures is
most likely to be enlarged on digital rectal examination?
A. Posterior lobe of the prostate
B. Median lobe of the prostate
C. Right lateral lobe of the prostate
D. Left lateral lobe of the prostate
E. Anterior lobe of the prostate
Answer: B
Carcinoma of the prostate typically occurs in the posterior lobe. The median lobe is usually enlarged in BPH. The anterior lobe has
little in the way of glandular tissue and is seldom enlarged.

Benign Prostatic Hyperplasia: Prostatic enlargement occurs in many elderly men . >90% of men aged over 80 will have at least
microscopic evidence of benign prostatic hyperplasia. Pathology: As part of the hyperplastic process increase in both stromal and
glandular components are seen. The changes are most notable in the central and periurethral region of the gland. Presentation: The
vast majority of men will present with lower urinary tract symptoms. These will typically be: Poor flow. Nocturia. Hesitancy.
Incomplete and double voiding. Terminal dribbling. Urgency. Incontinence. Investigation: Digital rectal examination to assess
prostatic size and morphology. Urine dipstick for infections and haematuria. Uroflowmetry (a flow rate of >15ml/second helps to
exclude BOO). Bladder pressure studies may help identify detrusor failure and whilst may not form part of first line investigations
should be included in those with atypical symptoms and prior to redo surgery. Bladder scanning to demonstrate residual volumes.
USS if high pressure chronic retention. Management: Lifestyle changes such as stopping smoking and altering fluid intake may
help those with mild symptoms. Medical therapy includes alpha blockers and 5 alpha reductase inhibitors.

4
The former work quickly on receptor zones located at the bladder neck. Cardiovascular side effects are well documented. The latter
work on testosterone metabolising enzymes. Although they have a slower onset of action, the 5 alpha reductase inhibitors may
prevent acute urinary retention. Surgical therapy includes transurethral resection of the prostate and is the treatment of choice in
those with severe symptoms and those who fail to respond to medical therapy. More tailored bladder neck incision procedures may
be considered in those with small prostates. Retrograde ejaculation may occur following surgery. The change in the type of
irrigation solutions used has helped to minimise the TURP syndrome of electrolyte disturbances.
11. A 58 year old man has been suffering from mechanical back pain for several years. One morning he awakes from sleep and
feels a sudden onset of pain in his back radiating down his left leg. Which of the following events is most likely to account for
his symptoms?
A. Prolapse of inner annulus fibrosus
B. Prolapse of outer annulus fibrosus
C. Prolapse of nucleus pulposus
D. Rupture of the ligamentum flavum
E. None of the above
Answer: C
The symptoms would be most likely the result of intervertebral disk prolapse. In disk prolapse the nucleus pulposus is the structure
which usually herniates. Intervertebral discs: Consist of an outer annulus fibrosus and an inner nucleus pulposus. The anulus
fibrosus consists of several layers of fibrocartilage. The nucleus pulposus contains loose fibres suspended in a mucoprotein gel with
the consistency of jelly. The nucleus of the disc acts as a shock absorber. Pressure on the disc causes posterior protrusion of the
nucleus pulposus. Most commonly in the lumbrosacral and lower cervical areas. The discs are separated by hyaline cartilage. There
is one disc between each pair of vertebrae, except for C1/2 and the sacrococcygeal vertebrae.

Theme: Paediatric neck masses

A. Cystic hygroma
B. Thyroglossal cyst
C. Rhabdomyosarcoma
D. Branchial cyst
E. Dermoid cyst

Please select the most likely underlying diagnosis for the situation that is described. Each option may be used once, more than once,
or not at all.

12. A 2 year old boy is brought to the clinic by his mother who has noticed that he has developed a small mass. On examination
a small smooth cyst is identified which is located above the hyoid bone. On ultrasound the lesion appears to be a
heterogenous and multiloculated mass.
Answer: Dermoid cyst
Dermoid cysts are usually multiloculated and heterogeneous. Most are located above the hyoid and their appearances on imaging
differentiate them from thyroglossal cysts.

13. A 22 month old baby is brought to the clinic by her mother who is concerned that she has developed a swelling in her neck.
On examination she has a soft, lesion located in the posterior triangle that transilluminates.
Answer: Cystic hygroma
Cystic hygromas are soft and transilluminate. Most are located in the posterior triangle.

14. A 3 year old boy is brought to the clinic by his mother who has noticed a mass in his neck. On examination he has a smooth
mass located on the lateral aspect of his anterior triangle, near to the angle of the mandible. On ultrasound it has a fluid
filled, anechoic, appearance.
Answer: The correct answer is Branchial cyst
Branchial cysts are usually located laterally and derived from the second branchial cleft. Unless infection has occurred they will
usually have an anechoic appearance on ultrasound.

5
Neck Masses in Children
Thyroglossal cyst: Located in the anterior triangle, usually in the midline and below the hyoid (65% cases). Derived from remnants
of the thyroglossal duct. Thin walled and anechoic on USS (echogenicity suggests infection of cyst)
Branchial cyst: Six branchial arches separated by branchial clefts. Incomplete obliteration of the branchial apparatus may result in
cysts, sinuses or fistulae. 75% of branchial cysts originate from the second branchial cleft. Usually located anterior to the
sternocleidomastoid near the angle of the mandible. Unless infected the fluid of the cyst has a similar consistency to water and is
anechoic on USS
Dermoids: Derived from pleuripotent stem cells and are located in the midline. Most commonly in a suprahyoid location. They
have heterogeneous appearances on imaging and contain variable amounts of calcium and fat
Thyroid gland: True thyroid lesions are rare in children and usually represent thyroglossal cysts or tumours like lymphoma
Lymphatic malformations: Usually located posterior to the sternocleidomastoid. Cystic hygroma result from occlusion of
lymphatic channels. The painless, fluid filled, lesions usually present prior to the age of 2. They are often closely linked to
surrounding structures and surgical removal is difficult. They are typically hypoechoic on USS
Infantile haemangioma: May present in either triangle of the neck. Grow rapidly initially and then will often spontaneously
regress. Plain x-rays will show a mass lesion, usually containing calcified phleboliths. As involution occurs the fat content of the
lesions increases
Lymphadenopathy: Located in either triangle of the neck. May be reactive or neoplastic. Generalised lymphadenopathy usually
secondary to infection in children (very common)

15. An unusually tall 43 year old lady presents to the surgical clinic with bilateral inguinal hernias. She develops chest pain and
collapses. As part of her investigations a chest x-ray shows evidence of mediastinal widening. What is the most likely
underlying diagnosis?
A. Pulmonary embolus
B. Aortic dissection
C. Tietze syndrome
D. Boerhaaves syndrome
E. Myocardial infarct
Answer: B
Marfans syndrome may present with a variety of connective tissue disorders such as bilateral inguinal hernia. They are at high risk
of aortic dissection, as in this case. Aortic dissection: More common than rupture of the abdominal aorta. 33% of patients die
within the first 24 hours, and 50% die within 48 hours if no treatment received. Associated with hypertension: Features of aortic
dissection: tear in the intimal layer, followed by formation and propagation of a subintimal hematoma. Cystic medial necrosis
(Marfan's). Most common site of dissection: 90% occurring within 10 centimetres of the aortic valve

Stanford Classification: Type A: Ascending aorta/ aortic root: Surgery- aortic root replacement. Type: B: Descending aorta:
Medical therapy with antihypertensives. DeBakey classification: Type I:Ascending aorta, aortic arch, descending aorta; Type II:
Ascending aorta only; Type III: Descending aorta distal to left subclavian artery. Clinical features: Tearing, sudden onset chest
pain (painless 10%). Hypertension or Hypotension. A blood pressure difference greater than 20 mm Hg. Neurologic deficits (20%)
Investigations: CXR: widened mediastinum, abnormal aortic knob, ring sign, deviation trachea/oesophagus. CT (spiral). MRI.
Angiography (95% of patients diagnosed). Management: Beta-blockers: aim HR 60- bpm and systolic BP 100-120 mm Hg. Urgent
surgical intervention: type A dissections. This will usually involve aortic root replacement.

16. A 72 year old man has just undergone an emergency repair for a ruptured abdominal aortic aneurysm. Pre operatively he
was taking aspirin, clopidogrel and warfarin. Intra operatively he received 5000 units of unfractionated heparin prior to
application of the aortic cross clamp. His blood results on admission to the critical care unit are as follows:
Full blood count: Hb: 8 g/dl; Platelets: 40 * 109/l; WBC: 7.1 * 109/l. His fibrin degradation products are measured and found
to be markedly elevated. Which of the following accounts for these results?
A. Anastomotic leak
B. Disseminated intravascular coagulation
C. Heparin induced thrombocytopenia
D. Adverse effect of warfarin
E. Adverse effects of antiplatelet agents
Answer: B
The combination of low platelet counts and raised FDP in this setting maked DIC the most likely diagnosis.

Disseminated intravascular coagulation – Diagnosis: Under homeostatic conditions, coagulation and fibrinolysis are coupled. The
activation of the coagulation cascade yields thrombin that converts fibrinogen to fibrin; the stable fibrin clot being the final product
6
of hemostasis. The fibrinolytic system breaks down fibrinogen and fibrin. Activation of the fibrinolytic system generates plasmin (in
the presence of thrombin), which is responsible for the lysis of fibrin clots. The breakdown of fibrinogen and fibrin results in
polypeptides (fibrin degradation products). In a state of homeostasis, the presence of plasmin is critical, as it is the central
proteolytic enzyme of coagulation and is also necessary for fibrinolysis.

In DIC, the processes of coagulation and fibrinolysis are dysregulated, and the result is widespread clotting with resultant bleeding.
Regardless of the triggering event of DIC, once initiated, the pathophysiology of DIC is similar in all conditions. One critical
mediator of DIC is the release of a transmembrane glycoprotein (tissue factor =TF). TF is present on the surface of many cell types
(including endothelial cells, macrophages, and monocytes) and is not normally in contact with the general circulation, but is exposed
to the circulation after vascular damage. For example, TF is released in response to exposure to cytokines (particularly interleukin
1), tumor necrosis factor, and endotoxin. This plays a major role in the development of DIC in septic conditions. TF is also abundant
in tissues of the lungs, brain, and placenta. This helps to explain why DIC readily develops in patients with extensive trauma. Upon
activation, TF binds with coagulation factors that then triggers the extrinsic pathway (via Factor VII) which subsequently triggers
the intrinsic pathway (XII to XI to IX) of coagulation.

Fibrin degradation products are often raised.


Disorder Prothrombin time APTT Bleeding time Platelet count
Warfarin administration Prolonged Normal Normal Normal
Aspirin administration Normal Normal Prolonged Normal
Heparin Often normal (may be prolonged) Prolonged Normal Normal
DIC Prolonged Prolonged Prolonged Low

17. A 53 year old man from Hong Kong presents with symptoms of fatigue, weight loss and recurrent epistaxis. Clinical
examination reveals left sided cervical lymphadenopathy and oropharyngeal examination reveals an ulcerated mass in the
naso pharynx. Which of the following viral agents is most commonly implicated in the development of this condition?
A. Cytomegalovirus
B. Epstein Barr virus
C. Coxsackie virus
D. Herpes simplex virus
E. None of the above
Answer: B
The clinical scenario is most typical for nasopharyngeal carcinoma. An association with previous Epstein Barr Virus is well
established. Infection with the other viruses listed is not a recognised risk factor for the development of the
condition.Nasopharyngeal carcinoma: Squamous cell carcinoma of the nasopharynx. Rare in most parts of the world, apart from
individuals from Southern ChinaAssociated with Epstein Barr virus infection. Presenting features: Systemic: Cervical
lymphadenopathy. Local: Otalgia, Unilateral serous otitis media, Nasal obstruction, discharge and/ or epistaxis, Cranial nerve
palsies e.g. III-VI. Imaging: Combined CT and MRI. Treatment: Radiotherapy is first line therapy.

18. An 18 year old male presents with lethargy, night sweats and on examination is found to have left supraclavicular
lymphadenopathy. A surgical registrar performs a left supraclavicular lymph node biopsy. The pathologist identifies Reed-
Sternberg cells on the subsequent histology sections, what is the most likely diagnosis?
A. Metastatic gastric cancer
B. Hodgkins lymphoma
C. Non Hodgkins lymphoma
D. Tuberculosis
E. None of the above
Answer: B
Reed-Sternberg cells are characteristic histological cell type found in Hodgkins disease. Lymphadenopathy: Lymphadenopathy in
the neck, axillae, groins and abdomen. Need to note: solitary/multiple, defined/indistinct, hard/rubbery/soft, tender/painless

Causes of lymphadenopathy: Mnemonic: Hodgkins disease: H aematological: Hodgkins lymphoma, NHL, Leukaemia; O
ncological: metastases; D ermatopathic lympadenitis; G aucher's disease; K awasaki disease; I nfections: TB, glandular fever,
Syphilis; N iemann Pick disease; S erum sickness; D rug reaction (phenytoin); I mmunological (SLE); S arcoidosis; E
ndocrinological (Hyperthyroidism); A ngioimmunoplastic lymphadenopathy; S LE; E osinophilic granulomatosis

19. Which of the following lesions is least likely to occur in the presence of severe atrophic gastritis?
A. Duodenal ulcer

7
B. Gastric cancer
C. Gastric polyp
D. Iron deficiency anaemia
E. Pernicious anaemia
Answer: A
Due the absence of acid a duodenal ulcer is unlikely to occur.

Gastric cancer: Overview: There are 700,000 new cases of gastric cancer worldwide each year. It is most common in Japan and
less common in western countries. It is more common in men and incidence rises with increasing age. The exact cause of many
sporadic cancer is not known, however, familial cases do occur in HNPCC families. In addition, smoking and smoked or preserved
foods increase the risk. Japanese migrants retain their increased risk (decreased in subsequent generations). The distribution of the
disease in western countries is changing towards a more proximal location (perhaps due to rising obesity).

Pathology: There is some evidence of support a stepwise progression of the disease through intestinal metaplasia progressing to
atrophic gastritis and subsequent dysplasia, through to cancer. The favoured staging system is TNM. The risk of lymph node
involvement is related to size and depth of invasion; early cancers confined to submucosa have a 20% incidence of lymph node
metastasis. Tumours of the gastro-oesophageal junction are classified: Type 1: True oesophageal cancers and may be associated
with Barrett's oesophagus. Type 2: Carcinoma of the cardia, arising from cardiac type epithelium or short segments with intestinal
metaplasia at the oesophagogastric junction. Type 3: Sub cardial cancers that spread across the junction. Involve similar nodal
stations to gastric cancer. Groups for close endoscopic monitoring: Intestinal metaplasia of columnar type. Atrophic gastritis. Low
to medium grade dysplasia. Patients who have previously undergone resections for benign peptic ulcer disease (except highly
selective vagotomy).

Table: Referral to endoscopy

Patients of any age with dyspepsia and Patients without dyspepsia Worsening dyspepsia
any of the following
Chronic gastrointestinal bleeding Dysphagia Barretts oesophagus
Dysphagia Unexplained abdominal pain or Intestinal metaplasia
weight loss
Weight loss Vomiting Dysplasia
Iron deficiency anaemia Upper abdominal mass Atrophic gastritis
Upper abdominal mass Jaundice Patient aged over 55 years with unexplained
or persistent dyspepsia

Staging: CT scanning of the chest abdomen and pelvis is the routine first line staging investigation in most centres. Laparoscopy to
identify occult peritoneal disease. PET CT (particularly for junctional tumours). Treatment: Proximally sited disease greater than 5-
10cm from the OG junction may be treated by sub total gastrectomy. Total gastrectomy if tumour is <5cm from OG junction. For
type 2 junctional tumours (extending into oesophagus) oesophagogastrectomy is usual. Endoscopic sub mucosal resection may play
a role in early gastric cancer confined to the mucosa and perhaps the sub mucosa (this is debated). Lymphadenectomy should be
performed. A D2 lymphadenectomy is widely advocated by the Japanese, the survival advantages of extended lymphadenectomy
have been debated. However, the overall recommendation is that a D2 nodal dissection be undertaken. Most patients will receive
chemotherapy either pre or post operatively. Prognsis: (UK Data) by percentage 5 year survival: All RO resections: 54%; Early
gastric cancer: 91%; Stage 1: 87%; Stage 2: 65%; Stage 3: 18%
Operative procedure: Total Gastrectomy , lymphadenectomy and Roux en Y anastomosis: General anaesthesia. Prophylactic
intravenous antibiotics. Incision: Rooftop. Perform a thorough laparotomy to identify any occult disease. Mobilise the left lobe of
the liver off the diaphragm and place a large pack over it. Insert a large self retaining retractor e.g. omnitract or Balfour (take time
with this, the set up should be perfect). Pack the small bowel away. Begin by mobilising the omentum off the transverse colon.
Proceed to detach the short gastric vessels. Mobilise the pylorus and divide it at least 2cm distally using a linear cutter stapling
device. Continue the dissection into the lesser sac taking the lesser omentum and left gastric artery flush at its origin.
The lymph nodes should be removed en bloc with the specimen where possible.
Place 2 stay sutures either side of the distal oesophagus. Ask the anaesthetist to pull back on the nasogastric tube. Divide the distal
oesophagus and remove the stomach. The oesphago jejunal anastomosis should be constructed. Identify the DJ flexure and bring a
loop of jejunum up to the oesophagus (to check it will reach). Divide the jejunum at this point. Bring the divided jejunum either
retrocolic or antecolic to the oesophagus. Anastamose the oesophagus to the jejunum, using either interrupted 3/0 vicryl or a stapling
device. Then create the remainder of the Roux en Y reconstruction distally.
Place a jejunostomy feeding tube. Wash out the abdomen and insert drains (usually the anastomosis and duodenal stump). Help the

8
anaesthetist insert the nasogastric tube (carefully!). Close the abdomen and skin. Enteral feeding may commence on the first post-
operative day. However, most surgeons will leave patients on free NG drainage for several days and keep them nil by mouth.

20. A 28 year old man develops an acute paronychia and subsequent spreading sepsis. The tissue exudate has a higher protein
content than normal tissue because?
A. Breakdown of tissue cells release protein
B. Capillary walls are more permeable
C. Increased blood flow transports more protein into the area
D. Intracapillary pressure is raised
E. Plasma cells release gamma globulin
Answer: B
The increased permeability allows the exudation of plasma proteins.

Acute inflammation: Inflammation is the reaction of the tissue elements to injury. Vascular changes occur, resulting in the
generation of a protein rich exudate. So long as the injury does not totally destroy the existing tissue architecture, the episode may
resolve with restoration of original tissue architecture.
Vascular changes: Vasodilation occurs and persists throughout the inflammatory phase. Inflammatory cells exit the circulation at
the site of injury. The equilibrium that balances Starlings forces within capillary beds is disrupted and a protein rich exudate will
form as the vessel walls also become more permeable to proteins. The high fibrinogen content of the fluid may form a fibrin clot.
This has several important immunomodulatory functions.

Sequelae: Resolution: Typically occurs with minimal initial injury. Stimulus removed and normal tissue architecture results.
Organisation:Delayed removed of exudate. Tissues undergo organisation and usually fibrosis. Suppuration:Typically formation of
an abscess or an empyema. Sequestration of large quantities of dead neutrophils. Progression to chronic inflammation: Coupled
inflammatory and reparative activities. Usually occurs when initial infection or suppuration has been inadequately managed
Causes: Microbacterial infections e.g. Viruses, exotoxins or endotoxins released by bacteria. Chemical agents. Physical agents e.g.
Trauma. Hypersensitivity reactions. Tissue necrosis. Presence of neutrophil polymorphs is a histological diagnostic feature of
acute inflammation

21. As a busy surgical trainee on the colorectal unit you are given the unenviable task of reviewing the unit's histopathology
results for colonic polyps. Which of the polyp types described below has the greatest risk of malignancy?
A. Hyperplastic polyp
B. Tubular adenoma
C. Villous adenoma
D. Hamartomatous polyp
E. Serrated polyp
Answer: C
Villous adenomas carry the highest risk of malignant transformation. Hyperplastic polyps carry little in the way of increased risk.
Although, patients with hamartomatous polyp syndromes may have a high risk of malignancy, the polyps themselves have little
malignant potential.

Colonic Polyps: May occur in isolation of greater numbers as part of the polyposis syndromes. In FAP greater than 100 polyps are
typically present. The risk of malignancy in association with adenomas is related to size and is the order of 10% in a 1cm adenoma.
Isolated adenomas seldom give risk of symptoms (unless large and distal). Distally sited villous lesions may produce mucous and if
very large electrolyte disturbances may occur.
Follow up of colonic polyps: Low risk: 1 or 2 adenomas <1cm. No follow up or re-colonoscopy at 5 years. Moderate risk: 3 or 4
small adenomas or 1 adenoma >1cm. Re-scope at 3 years. High risk: >5 small adenomas or >3 with 1 of them >1cm. Re scope at 1
year. It is important to stratify patients appropriately and ensure that a complete colonoscopy with good views was performed.
Segmental resection or complete colectomy should be considered when: 1. Incomplete excision of malignant polyp; 2. Malignant
sessile polyp; 3. Malignant pedunculated polyp with submucosal invasion; 4. Polyps with poorly differentiated carcinoma; 5.
Familial polyposis coli. Screening from teenager up to 40 years by 2 yearly sigmoidoscopy/colonoscopy. Panproctocolectomy and
Ileostomy or Restorative Panproctocolectomy. Rectal polypoidal lesions may be amenable to trans anal endoscopic microsurgery.

22. A 23 year old man presents to the surgical clinic with an inguinal hernia. On examination he has a small direct hernia.
However, you also notice that he has pigmented spots around his mouth, on his palms and soles. In his history he underwent
a reduction of an intussusception aged 12 years. Which of the following lesions is most likely to be identified if a colonoscopy
were performed?
A. Hamartomas
9
B. Tubulovillous adenoma
C. Colorectal cancer
D. Crohns disease
E. Hyperplastic polyps
Answer: A
He is most likely to have Peutz-Jeghers syndrome which is associated with Hamartomas.

Peutz-Jeghers syndrome: Peutz-Jeghers syndrome is an autosomal dominant condition characterised by numerous benign
hamartomatous polyps in the gastrointestinal tract. It is also associated with pigmented freckles on the lips, face, palms and soles.
Around 50% of patients will have died from a gastrointestinal tract cancer by the age of 60 years.
Genetics: Autosomal dominant. Responsible gene encodes serine threonine kinase LKB1 or STK11. Features: Hamartomatous
polyps in GI tract (mainly small bowel). Pigmented lesions on lips, oral mucosa, face, palms and soles. Intestinal obstruction e.g.
intussusception (which may lead to diagnosis). Gastrointestinal bleeding. Management: Conservative unless complications develop

23. A 56 year old surgeon has been successfully operating for many years. Over the past few weeks she has begun to notice that
her hands are becoming blistering and weepy. A latex allergy is diagnosed. Which of the following pathological processes
accounts for this scenario?
A. Type 1 hypersensitivity reaction
B. Type 2 hypersensitivity reaction
C. Type 4 hypersensitivity reaction
D. Type 3 hypersensitivity reaction
E. None of the above
Answer: C
Hypersensitivity reactions: ACID: type 1 –Anaphylactic; type 2 –Cytotoxic; type 3 --Immune complex type 4 --Delayed
hypersensitivity
Contact dermatitis of a chronic nature is an example of a type 4 hypersensitivity reaction. Type 4 hypersensitivity reactions are cell
mediated rather than antibody mediated.

Hypersensitivity reactions: The Gell and Coombs classification divides hypersensitivity reactions into 4 types

Type I Type II Type III Type IV


Description Anaphylactic Cytotoxic Immune complex Delayed type
Mediator IgE IgG, IgM IgG, IgM T-cells
Antigen Exogenous Cell surface Soluble Tissues
Response time Minutes Hours Hours 2-3 days
Examples Asthma Autoimmune haemolytic anaemia Serum sickness Graft versus host disease
Hay fever Pemphigus SLE Contact dermatitis
Goodpasture's Aspergillosis

24. A 56 year old motorcyclist is involved in a road traffic accident and sustains a displaced femoral shaft fracture. Not other
injuries are identified on the primary or secondary surveys. The fracture is treated with closed, antegrade intramedullary
nailing. The following day the patient becomes increasingly agitated and confused. On examination he is pyrexial, hypoxic
SaO2 90% on 6 litres O2, tachycardic and normotensive. Systemic examination demonstrates a non blanching petechial rash
present over the torso. What is the most likely explanation for this?
A. Pulmonary embolism with paradoxical embolus
B. Fat embolism
C. Meningococcal sepsis
D. Alcohol withdrawl
E. Chronic sub dural haematoma
Answer: B
This man has a recent injury and physical signs that would be concordant with fat embolism syndrome. Meningococcal sepsis is not
usually associated with hypoxia initially. Pulmonary emboli are not typically associated with pyrexia. Fat embolism:
Cardiothoracic:Early persistent tachycardia. Tachypnoea, dyspnoea, hypoxia usually 72 hours following injury. Pyrexia.
Dermatological: Red/ brown impalpable petechial rash (usually only in 25-50%). Subconjunctival and oral haemorrhage/ petechiae.
CNS: Confusion and agitation. Retinal haemorrhages and intra-arterial fat globules on fundoscopy. Imaging: May be normal. Fat
emboli tend to lodge distally and therefore CTPA may not show any vascular occlusion, a ground glass appearance may be seen at
the periphery. Treatment: Prompt fixation of long bone fractures. Some debate regarding benefit Vs. risk of medullary reaming in
femoral shaft/ tibial fractures in terms of increasing risk (probably does not). DVT prophylaxis. General supportive care

10
25. Which of these tumour markers is most helpful in identifying an individual with hepatocellular carcinoma?
A. Serum AFP
B. Serum CA19-9
C. CEA
D. Beta HCG
E. CA125
Answer: A
Hepatocellular carcinoma is commonly diagnosed with imaging and an elevated alpha fetoprotein. Biopsy may seed the tumour and
should be avoided. Up to 80% of hepatocellular carcinoma arise in cirrhotic livers.

The most common primary liver tumours are cholangiocarcinoma and hepatocellular carcinoma. Overall metastatic disease accounts
for 95% of all liver malignancies making the primary liver tumours comparatively rare.
Primary liver tumours include: Cholangiocarcinoma. Hepatocellular carcinoma. Hepatoblastoma. Sarcomas (Rare). Lymphomas.
Carcinoids (most often secondary although primary may occur)

Hepatocellular carcinoma: These account for the bulk of primary liver tumours (75% cases). Its worldwide incidence reflects its
propensity to occur on a background of chronic inflammatory activity. Most cases arise in cirrhotic livers or those with chronic
hepatitis B infection, especially where viral replication is actively occurring. In the UK it accounts for less than 5% of all cancers,
although in parts of Asia its incidence is 100 per 100,000.
The majority of patients (80%) present with existing liver cirrhosis, with a mass discovered on screening ultrasound. Diagnosis:
CT/ MRI (usually both) are the imaging modalities of choice. a-fetoprotein is elevated in almost all cases. Biopsy should be avoided
as it seeds tumours cells through a resection plane. In cases of diagnostic doubt serial CT and aFP measurements are the preferred
strategy. Treatment: Patients should be staged with liver MRI and chest, abdomen and pelvic CT scan. The testis should be
examined in males (testicular tumours may cause raised AFP). PET CT may be used to identify occult nodal disease. Surgical
resection is the mainstay of treatment in operable cases. In patients with a small primary tumour in a cirrhotic liver whose primary
disease process is controlled, consideration may be given to primary whole liver resection and transplantation.Liver resections are
an option but since most cases occur in an already diseased liver the operative risks and post-operative hepatic dysfunction are far
greater than is seen following metastectomy. These tumours are not particularly chemo or radiosensitive however, both may be used
in a palliative setting. Tumour ablation is a more popular strategy. Survival: Poor, overall survival is 15% at 5 years.

Cholangiocarcinoma: This is the second most common type of primary liver malignancy. As its name suggests these tumours arise
in the bile ducts. Up to 80% of tumours arise in the extra hepatic biliary tree. Most patients present with jaundice and by this stage
the majority will have disease that is not resectable. Primary scelerosing cholangitis is the main risk factor. In deprived countries
typhoid and liver flukes are also major risk factors. Diagnosis: Patients will typically have an obstructive picture on liver function
tests. CA 19-9, CEA and CA 125 are often elevated. CT/ MRI and MRCP are the imaging methods of choice. Treatment: Surgical
resection offers the best chance of cure. Local invasion of peri hilar tumours is a particular problem and this coupled with lobar
atrophy will often contra indicate surgical resection. Palliation of jaundice is important, although metallic stents should be avoided
in those considered for resection. Survival: Is poor, approximately 15% 5 year survival.

26. A 39 year old man has suffered from terminal ileal Crohns disease for the past 20 years. Which condition is he least likely to
develop?
A. Gallstones
B. Malabsorption
C. Pyoderma gangrenosum
D. Amyloidosis
E. Feltys syndrome
Answer: E
Felteys syndrome: Rheumatoid disease, Splenomegaly and Neutropenia
Feltys syndrome is associated with rheumatoid disease. Individuals with long standing crohns disease are at risk of gallstones
because of impairment of the enterohepatic recycling of bile salts. Formation of entero-enteric fistulation may produce
malabsorption. Amyloidosis may complicate chronic inflammatory states.

Crohns disease: Crohns disease is a chronic transmural inflammation of a segment(s) of the gastrointestinal tract and may be
associated with extra intestinal manifestations. Frequent disease patterns observed include ileal, ileocolic and colonic disease. Peri-
anal disease may occur in association with any of these. The disease is often discontinuous in its distribution. Inflammation may
cause ulceration, fissures, fistulas and fibrosis with stricturing. Histology reveals a chronic inflammatory infiltrate that is usually
patchy and transmural.

11
Ulcerative colitis Vs Crohns
Crohn's disease Ulcerative colitis
Distribution Mouth to anus Rectum and colon
Macroscopic Cobblestone appearance, apthoid ulceration Contact bleeding
changes
Depth of disease Transmural inflammation Superficial inflammation
Distribution Patchy Continuous
pattern
Histological Granulomas (non caseating epithelioid cell aggregates with Crypt abscesses, Inflammatory cells in the
features Langhans' giant cells) lamina propria

Extraintestinal manifestations of Crohns


Related to disease extent Unrelated to disease extent
Aphthous ulcers (10%) Sacroiliiitis (10-15%)
Erythema nodosum (5-10%) Ankylosing spondylitis (1-2%)
Pyoderma gangrenosum (0.5%) Primary sclerosing cholangitis (Rare)
Acute arthropathy (6-12%) Gallstones (up to 30%)
Ocular complications (up to 10%) Renal calculi (up to 10%)

Theme: Renal stones

A. Calcium oxalate
B. Uric acid
C. Cystine
D. Struvite
E. Calcium phosphate

Please select the most likely stone type for each of the following urinary tract stone scenarios. Each option may be used once, more
than once or not at all.

27. A 73 year old lady is undergoing chemotherapy for treatment of acute leukaemia. She develops symptoms of renal colic. Her
urine tests positive for blood. A KUB x-ray shows no evidence of stones.
Answer: Uric acid
Chemotherapy and cell death can increase uric acid levels. In this acute setting the uric acid stones are unlikely to be coated with
calcium and will therefore be radiolucent.

28. A 16 year old boy presents with renal colic. His parents both have a similar history of the condition. His urine tests positive
for blood. A KUB style x-ray shows a relatively radiodense stone in the region of the mid ureter.
Answer: Cystine
Cystine stones are associated with an inherited metabolic disorder.

29. A 43 year old lady with episodes of recurrent urinary tract sepsis presents with a staghorn calculus of the left kidney. Her
urinary pH is 7.3. A KUB x-ray shows a faint outline of the calculus.
Answer: Struvite
Chronic infection with urease producing enzymes can produce an alkaline urine with formation of struvate stone.
Renal stones: Calcium oxalate: Hypercalciuria is a major risk factor (various causes). Hyperoxaluria may also increase risk
Hypocitraturia increases risk because citrate forms complexes with calcium making it more soluble. Stones are radio-opaque
(though less than calcium phosphate stones). Hyperuricosuria may cause uric acid stones to which calcium oxalate binds
Percentage of all calculi: 85%

Stone type Urine acidity Mean


urine pH
Calcium phosphate Normal- alkaline >5.5
12
Cystine: Inherited recessive disorder of transmembrane cystine Calcium oxalate Variable 6
transport leading to decreased absorption of cystine from intestine Uric acid Acid 5.5
and renal tubule. Multiple stones may form. Relatively radiodense Struvate Alkaline >7.2
because they contain sulphur. Percentage of all calculi 1%. Uric Cystine Normal 6.5
acid: Uric acid is a product of purine metabolism. May precipitate
when urinary pH low. May be caused by diseases with extensive tissue breakdown e.g. malignancy. More common in children with
inborn errors of metabolism. Radiolucent. Percentage of all calculi: 5-10%. Calcium phosphate: May occur in renal tubular
acidosis, high urinary pH increases supersaturation of urine with calcium and phosphate. Renal tubular acidosis types 1 and 3
increase risk of stone formation (types 2 and 4 do not). Radio-opaque stones (composition similar to bone). Percentage of all calculi
10%. Struvite: Stones formed from magnesium, ammonium and phosphate. Occur as a result of urease producing bacteria (and are
thus associated with chronic infections). Under the alkaline conditions produced, the crystals can precipitate. Slightly radio-opaque.
Percentage of all calculi: 2-20%>

Effect of urinary pH on stone formation: Urine pH will show individual variation (from pH 5-7). Post prandially the pH falls as
purine metabolism will produce uric acid. Then the urine becomes more alkaline (alkaline tide). When the stone is not available for
analysis the pH of urine may help to determine which stone was present.

30. A 64 year old man presents to the clinic with right upper quadrant discomfort. He has never attended the hospital
previously and is usually well. He has just retired from full time employment as a machinist in a PVC factory. CT scanning
shows a large irregular tumour in the right lobe of his liver. Which of the following lesions is the most likely?
A. Liposarcoma
B. Angiosarcoma
C. Hamartoma
D. Hyatid liver disease
E. Benign angioma
Answer: B
Angiosarcoma of the liver is a rare tumour. However, it is linked to working with vinyl chloride, as in this case. Although modern
factories minimise the exposure to this agent, this has not always been the case.

Occupational cancers accounted for 5.3% cancer deaths in 2005. In men the main cancers include: Mesothelioma. Bladder cancer.
Non melanoma skin cancer. Lung cancer. Sino nasal cancer. Occupations with high levels of occupational tumours include:
Construction industry. Working with coal tar and pitch. Mining. Metalworkers. Working with asbestos (accounts for 98% of all
mesotheliomas). Working in rubber industry. Shift work has been linked to breast cancer in women (Health and safety executive
report RR595).
The latency between exposure and disease is typically 15 years for solid tumours and 20 for leukaemia. Many occupational cancers
are otherwise rare. For example sino nasal cancer is an uncommon tumour, 50% will be SCC. They are linked to conditions such as
wood dust exposure and unlike lung cancer is not strongly linked to cigarette smoking. Another typical occupational tumour is
angiosarcoma of the liver which is linked to working with vinyl chloride. Again in the non occupational context this is an extremely
rare sporadic tumour.

31. A 32 year old man is involved in a house fire and sustains extensive partial thickness burns to his torso and thigh. Two weeks
post operatively he develops oedema of both lower legs. The most likely cause of this is:
A. Iliofemoral deep vein thrombosis
B. Venous obstruction due to scarring
C. Hypoalbuminaemia
D. Excessive administration of intravenous fluids
E. None of the above
Answer: C
Loss of plasma proteins is the most common cause of oedema developing in this time frame.

Extensive burns: Haemolysis due to damage of erythrocytes by heat and microangiopathy. Loss of capillary membrane integrity
causing plasma leakage into interstitial space. Extravasation of fluids from the burn site causing hypovolaemic shock (up to 48h
after injury)- decreased blood volume and increased haematocrit. Protein loss. Secondary infection e.g. Staphylococcus aureus.
ARDS. Risk of Curlings ulcer (acute peptic stress ulcers). Danger of full thickness circumferential burns in an extremity as these
may develop compartment syndrome
Healing: Superficial burns: keratinocytes migrate to form a new layer over the burn site. Full thickness burns: dermal scarring.
Usually need keratinocytes from skin grafts to provide optimal coverage.

32. What is the diagnostic marker for carcinoid syndrome?


A. B-HCG
13
B. Histamine
C. Chromogranin A
D. 5-Hydroxyindoleacetic acid
E. 5-Hydroxytryptamine
Answer: D
Urinary measurement of 5- HIAA is an important part of clinical follow up.

Carcinoid syndrome: Carcinoid tumours secrete serotonin. Originate in neuroendocrine cells mainly in the intestine (midgut-distal
ileum/appendix). Can occur in the rectum, bronchi. Hormonal symptoms mainly occur when disease spreads outside the bowel
Clinical features: Onset: years. Flushing face. Palpitations. Tricuspid stenosis causing dyspnea. Asthma. Severe diarrhoea
(secretory, persists despite fasting). Investigation: 5-HIAA in a 24-hour urine collection. Scintigraphy. CT scan. Treatment:
Octreotide. Surgical removal.

33. A 42 year old man from Southern India presents with chronic swelling of both lower legs, they are brawny and indurated
with marked skin tophic changes. Which of the following organisms is the most likely origin of this disease process?
A. Loa loa
B. Wuchereria bancrofti
C. Trypanosoma cruzi
D. Trypanosoma gambiense
E. None of the above
Answer: B
W. Bancrofti is the commonest cause of filariasis leading to lymphatic obstruction. Infection with Loa loa typically occurs in the
African sub continent and usually results in generalised sub cutaneous infections without lymphatic obstruction. Trypanosomal
infections would not produce this clinical picture.

Wuchereria bancrofti: Parasitic filarial nematode. Accounts for 90% of cases of filariasis. Usually diagnosed by blood smears.
Usually transmitted by mosquitos. Treatment is with diethylcarbamazine.

34. A 45 year old lady has recently undergone a thyroidectomy for treatment of medullary thyroid cancer. Which of the
following tumour markers is used clinically to screen for recurrence?
a. Free T3
b. Thyroglobulin
c. Calcitonin
d. Free T4
e. Thyroid stimulating hormone
Answer: C
Calcitonin is clinically utilised to screen for medullary thyroid cancer recurrence. Thyroid function testing does not form part of
either diagnosis or follow up from a malignancy perspective. However, routine assessment of TSH may be needed in patients on
thyroxine.

Papillary carcinoma: Commonest sub-type. Accurately diagnosed on fine needle aspiration cytology. Histologically they may
demonstrate psammoma bodies (areas of calcification) and so called 'orphan Annie' nuclei. They typically metastasise via the
lymphatics and thus laterally located apparently ectopic thyroid tissue is usually a metastasis from a well differentiated papillary
carcinoma.
Follicular carcinoma: Are less common than papillary lesions. Like papillary tumours they may present as a discrete nodule.
Although they appear to be well encapsulated macroscopically there invasion on microscopic evaluation. Lymph node metastases
are uncommon and these tumours tend to spread haematogenously. This translates into a higher mortality rate. Follicular lesions
cannot be accurately diagnosed on fine needle aspiration cytology and thus all follicular FNA's will require at least a hemi
thyroidectomy.
Anaplastic carcinoma: Less common and tend to occur in elderly females. Disease is usually advanced at presentation and often
only palliative decompression and radiotherapy can be offered.
Medullary carcinoma: These are tumours of the parafollicular cells ( C Cells) and are of neural crest origin. The serum calcitonin
may be elevated which is of use when monitoring for recurrence. . They may be familial and occur as part of the MEN -2A disease
spectrum. Spread may be either lymphatic or haematogenous and as these tumours are not derived primarily from thyroid cells they
are not responsive to radioiodine.
Lymphoma: These respond well to radiotherapy. Radical surgery is unnecessary once the disease has been diagnosed on biopsy
material. Such biopsy material is not generated by an FNA and thus a core biopsy has to be obtained (with care!).

14
35. A 22 year old man is kicked in the head during a rugby match. He is temporarily concussed, but then regains consciousness.
Half an hour later he develops slurred speech, ataxia and loses consciousnesses. On arrival in hospital he is intubated and
ventilated. A CT Scan is performed which shows an extradural haematoma. What is the most likely cause?
A. Basilar artery laceration
B. Middle meningeal artery laceration
C. Laceration of the sigmoid sinus
D. Laceration of the anterior cerebral artery
E. Laceration of the middle cerebral artery
Answer: B
The most likely vessel from those in the list to cause an acute extra dural haemorrhage is the middle meningeal artery. The anterior
and middle cerebral arteries may cause acute sub dural haemorrhage. Acute sub dural haemorrhages usually take slightly longer to
evolve than acute extra dural haemorrhages.

Middle meningeal artery: Middle meningeal artery is typically the third branch of the first part of the maxillary artery, one of the
two terminal branches of the external carotid artery. After branching off the maxillary artery in the infratemporal fossa, it runs
through the foramen spinosum to supply the dura mater (the outermost meninges) . The middle meningeal artery is the largest of the
three (paired) arteries which supply the meninges, the others being the anterior meningeal artery and the posterior meningeal artery.
The middle meningeal artery runs beneath the pterion. It is vulnerable to injury at this point, where the skull is thin. Rupture of the
artery may give rise to an extra dural hematoma. In the dry cranium, the middle meningeal, which runs within the dura mater
surrounding the brain, makes a deep indention in the calvarium. The middle meningeal artery is intimately associated with the
auriculotemporal nerve which wraps around the artery making the two easily identifiable in the dissection of human cadavers and
also easily damaged in surgery.

36. Which of the following is not characteristic of a granuloma?


A. Altered macrophages
B. Fused macrophages
C. Epithelioid cells
D. Mixture of chronic inflammatory cells
E. Polymorphnuclear leucocytes, cellular debris and fibrin
Answer: E
These are typical components of an abscess cavity. Polymorphonuclear leucocytes may be found in a granuloma if there is a focus
of suppuration.

Chronic inflammation: it may occur secondary to acute inflammation.In most cases chronic inflammation occurs as a primary
process. These may be broadly viewed as being one of three main processes: Persisting infection with certain organisms such as
Mycobacterium tuberculosis which results in delayed type hypersensitivity reactions and inflammation. Prolonged exposure to non-
biodegradable substances such as silica or suture materials which may induce an inflammatory response. Autoimmune conditions
involving antibodies formed against host antigens.

Acute vs. Chronic inflammation: Acute inflammation: Infiltration of neutrophils. Changes to existing vascular structure and
increased permeability of endothelial cells. Process may resolve with: Suppuration. Complete resolution. Abscess formation.
Progression to chronic inflammation. Healing by fibrosis. Chronic inflammation: Angiogenesis predominates. Macrophages,
plasma cells and lymphocytes predominate. Healing by fibrosis is the main result

Granulomatous inflammation: A granuloma consists of a microscopic aggregation of macrophages (with epithelial type
arrangement =epitheliod). Large giant cells may be found at the periphery of granulomas.

Mediators: Growth factors released by activated macrophages include agents such as interferon and fibroblast growth factor (plus
many more). Some of these such as interferons may have systemic features resulting in systemic symptoms and signs, which may be
present in individuals with long standing chronic inflammation.

37. A 42 year old man presents with a painless lump in the left testicle that he noticed on self examination. Clinically there is a
firm nodule in the left testicle, ultrasound appearances show an irregular mass lesion. His serum AFP and HCG levels are
both within normal limits. What is the most likely diagnosis?
A. Yolk sack tumour
B. Seminoma
C. Testicular teratoma
D. Epididymo-orchitis
15
E. Adenomatoid tumour
Answer: B
This mans age, presenting symptoms and normal tumour markers make a seminoma the most likely diagnosis. Epididymo-orchitis
does not produce irregular mass lesions which are painless.

Testicular cancer: Testicular cancer is the most common malignancy in men aged 20-30 years. Around 95% of cases of testicular
cancer are germ-cell tumours. Germ cell tumours may essentially be divided into Seminoma and Non seminomatous germ cell
tumours

Seminoma: Key features: Commonest subtype (50%). Average age at diagnosis = 40. Even advanced disease associated with 5
year survival of 73% AFP usually normal. Tumour markers: HCG elevated in 10% seminomas. Lactate dehydrogenase; elevated
in 10-20% seminomas (but also in many other conditions). Pathology: Sheet like lobular patterns of cells with substantial fibrous
component. Fibrous septa contain lymphocytic inclusions and granulomas may be seen.

Non seminomatous germ cell tumours (42%): Teratoma, Yolk sac tumour, Choriocarcinoma and Mixed germ cell tumours
(10%): Key features Younger age at presentation =20-30 years. Advanced disease carries worse prognosis (48% at 5 years).
Retroperitoneal lymph node dissection may be needed for residual disease after chemotherapy. Tumour markers: AFP elevated in
up to 70% of cases. HCG elevated in up to 40% of cases. Other markers rarely helpful. Pathology: Heterogenous texture with
occasional ectopic tissue such as hair

Risk factors for testicular cancer: Cryptorchidism. Infertility. Family history. Klinefelter's syndrome. Mumps orchitis. Features:
A painless lump is the most common presenting symptom. Pain may also be present in a minority of men. Other possible features
include hydrocele, gynaecomastia. Diagnosis: Ultrasound is first-line. CT scanning of the chest/ abdomen and pelvis is used for
staging. Tumour markers (see above) should be measured. Management: Orchidectomy (Inguinal approach). Chemotherapy and
radiotherapy may be given depending on staging. Abdominal lesions >1cm following chemotherapy may require retroperitoneal
lymph node dissection.. Prognosis is generally excellent: 5 year survival for seminomas is around 95% if Stage I. 5 year survival for
teratomas is around 85% if Stage I

Benign disease
Epididymo-orchitis: Acute epididymitis is an acute inflammation of the epididymis, often involving the testis and usually caused
by bacterial infection. Infection spreads from the urethra or bladder. In men <35 years, gonorrhoea or chlamydia are the usual
infections. Amiodarone is a recognised non infective cause of epididymitis, which resolves on stopping the drug.. Tenderness is
usually confined to the epididymis, which may facilitate differentiating it from torsion where pain usually affects the entire testis.
Testicular torsion: Twist of the spermatic cord resulting in testicular ischaemia and necrosis. Most common in males aged between
10 and 30 (peak incidence 13-15 years). Pain is usually severe and of sudden onset.. Cremasteric reflex is lost and elevation of the
testis does not ease the pain. Treatment is with surgical exploration. If a torted testis is identified then both testis should be fixed as
the condition of bell clapper testis is often bilateral.
38. A baby is born by normal vaginal delivery at 39 weeks gestation. Initially all appears well and then the clinical staff become
concerned because the baby develops recurrent episodes of cyanosis. These are worse during feeding and improve
dramatically when the baby cries. The most likely underlying diagnosis is:
A. Choanal atresia
B. Oesophageal reflux
C. Tetralogy of Fallot
D. Oesophageal atresia
E. Congenital diaphragmatic hernia
Answer: A
In Choanal atresia the episodes of cyanosis are usually worst during feeding. Improvement may be seen when the baby cries as the
oropharyngeal airway is used.

Choanal atresia: Congenital disorder with an incidence of 1 in 7000 births. Posterior nasal airway occluded by soft tissue or bone.
Associated with other congenital malformations e.g. coloboma. Babies with unilateral disease may go unnoticed. Babies with bilateral
disease will present early in life as they are obligate nasal breathers. Treatment is with fenestration procedures designed to restore
patency.

39. A 28 year old lady presents with a pigmented lesion on her calf. Excisional biopsy confirms a diagnosis of melanoma
measuring 1cm in diameter with a Breslow thickness of 0.5mm. The lesion is close <1 mm to all resection margins. Which of
the following surgical resection margins is acceptable for this lesion?
A. 5 cm
B. 1 cm

16
C. 0.5 cm
D. 2 cm
E. 3 cm
Answer: B
The main diagnostic features (major criteria): Change in size. Change in shape. Change in colour
Secondary features (minor criteria): Diameter >6mm. Inflammation. Oozing or bleeding. Altered sensation
Treatment: Suspicious lesions should undergo excision biopsy. The lesion should be removed in completely as incision biopsy can
make subsequent histopathological assessment difficult. Once the diagnosis is confirmed the pathology report should be reviewed to
determine whether further re-exicision of margins is required (see below):
Margins of excision-Related to Breslow thickness: Lesions 0-1mm thick:1cm. Lesions 1-2mm thick:1- 2cm (Depending upon site
and pathological features). Lesions 2-4mm thick:2-3 cm (Depending upon site and pathological features). Lesions >4 mm thick:3cm
Further treatments such as sentinel lymph node mapping, isolated limb perfusion and block dissection of regional lymph node groups
should be selectively applied.

40. A 20 year old man is involved in a road traffic accident. Following the incident he is unable to extend his wrist. However,
this improves over the following weeks. Which type of injury is he most likely to have sustained?
A. Radial nerve neurotmesis
B. Radial nerve neuropraxia
C. Axillary nerve axonotmesis
D. Ulnar nerve neuropraxia
E. Ulnar nerve axonotmesis
Answer: B
Transient loss of function makes neuropraxia the most likely injury. The wrist extensors are innervated by the radial nerve making
this the most likely site of injury.
Neuropraxia: Nerve intact but electrical conduction is affected. Myelin sheath integrity is preserved. Full recovery. Autonomic
function preserved. Wallerian degeneration does not occur

41. A 53 year old lady has undergone a bilateral breast augmentation procedure many years previously. The implants are tense
and uncomfortable and are removed. During their removal the surgeon encounters a dense membrane surrounding the
implants, it has a coarse granular appearance. The tissue is sent for histology and it demonstrates fibrosis with the presence
of calcification. The underlying process responsible for these changes is:
A. Hyperplasia
B. Dysplasia
C. Metastatic calcification
D. Dystrophic calcification
E. Necrosis
Answer: D
Breast implants often become surrounded by a pseudocapsule and this may secondarily then be subjected to a process of dystrophic
calcification.

Pathological calcification: Dystrophic calcification:Deposition of calcium deposits in tissues that have undergone, degeneration,
damage or disease in the presence of normal serum calcium levels. Metastatic calcification:Deposition of calcium deposits in tissues
that are otherwise normal in the presence of increased serum calcium levels

42. A 4 year old girl presents with symptoms of right sided loin pain, lethargy and haematuria. On examination she is pyrexial
and has a large mass in the right upper quadrant. The most likely underlying diagnosis is:
A. Perinephric abscess
B. Nephroblastoma
C. Renal cortical adenoma
D. Grawitz tumour
E. Squamous cell carcinoma of the kidney
Answer: B
In a child of this age, with the symptoms described a nephroblastoma is the most likely diagnosis. A perinephric abscess is most
unlikely. If an abscess were to occur it would be confined to Gertotas fascia in the first instance, and hence anterior extension would
be unlikely.

17
Nephroblastoma (Wilms tumours): Usually present in first 4 years of life. May often present as a mass associated with haematuria
(pyrexia may occur in 50%). Often metastasise early (usually to lung). Treated by nephrectomy. Younger children have better
prognosis (<1 year of age =80% overall 5 year survival)

Theme: Thyroid neoplasms

A. Follicular carcinoma
B. Anaplastic carcinoma
C. Medullary carcinoma
D. Papillary carcinoma
E. Lymphoma
F. Hashimotos thyroiditis
G. Graves disease
For the following histological descriptions please select the most likely underlying thyroid neoplasm. Each option may be used
once, more than once or not at all.

44. A 22 year old female undergoes a thyroidectomy. The resected specimen shows a non encapsulated tumour with papillary
projections and pale empty nuclei.
Answer: Papillary carcinoma
The presence of papillary structures together with the cytoplasmic features described is strongly suggestive of papillary carcinoma.
They are seldom encapsulated.

45. A thyroidectomy specimen from a 43 year old lady shows a mass with prominent oxyphil cells and scanty thyroid colloid.
Answer Follicular carcinoma
Hurthle cell tumours are a variant of follicular neoplasms in which oxyphil cells predominate. They have a poorer prognosis than
conventional follicular neoplasms

46. A 32 year old lady undergoes a thyroidectomy for a mild goitre. The resected specimen shows an intense lymphocytic
infiltrate with acinar destruction and fibrosis.
Answer: Hashimotos thyroiditis
Lymphocytic infiltrates and fibrosis are typically seen in Hashimotos thyroiditis. In Lymphoma only dense lymphatic type tissue is
usually present.

Thyroid neoplasms
Lesion:Common features
Follicular adenoma: Usually present as a solitary thyroid nodule. Malignancy can only be excluded on formal histological
assessment
Papillary carcinoma: Usually contain a mixture of papillary and colloidal filled follicles. Histologically tumour has papillary
projections and pale empty nuclei. Seldom encapsulated. Lymph node metastasis predominate. Haematogenous metastasis rare.
Account for 60% of thyroid cancers
Follicular carcinoma:May appear macroscopically encapsulated, microscopically capsular invasion is seen. Without this finding
the lesion is a follicular adenoma.Vascular invasion predominates. Multifocal disease rare. Account for 20% of all thyroid cancers
Anaplastic carcinoma: Most common in elderly females. Local invasion is a common feature. Account for 10% of thyroid cancers.
Treatment is by resection where possible, palliation may be achieved through isthmusectomy and radiotherapy. Chemotherapy is
ineffective.
Medullary carcinoma:Tumours of the parafollicular cells (C Cells). C cells derived from neural crest and not thyroid tissue. Serum
calcitonin levels often raised. Familial genetic disease accounts for up to 20% cases. Both lymphatic and haematogenous metastasis
are recognised, nodal disease is associated with a very poor prognosis.

47. A 43 year old lady with hypertension is suspected of having a phaeochromocytoma. Which of the following investigations is
most likely to be beneficial in this situation?
A. Dexamethasone suppression test
B. Urinary 5-Hydroxyindoleacetic Acid (5-HIAA)
C. Histamine provocation test
D. Tyramine provocation test
E. Urinary vanillymandelic acid measurements
Answer: E
18
Urinary VMA measurements are not completely specific but constitute first line assessment. Stimulation tests of any sort are not justified
in first line assessments.

Phaeochromocytoma: Neuroendocrine tumour of the chromaffin cells of the adrenal medulla. Hypertension and hyperglycaemia are
often found: 10% of cases are bilateral. 10% occur in children. 11% are malignant (higher when tumour is located outside the adrenal).
10% will not be hypertensive. Familial cases are usually linked to the Multiple endocrine neoplasia syndromes (considered under its
own heading). Most tumours are unilateral (often right sided) and smaller than 10cm. Diagnosis: Urine analysis of vanillymandelic
acid (VMA) is often used (false positives may occur e.g. in patients eating vanilla ice cream!). Blood testing for plasma metanephrine
levels. CT and MRI scanning are both used to localise the lesion.

Treatment: Patients require medical therapy first. An irreversible alpha adrenoreceptor blocker should be given, although minority
may prefer reversible bockade(1). Labetolol may be co-administered for cardiac chronotropic control. Isolated beta blockade should
not be considered as it will lead to unopposed alpha activity. These patients are often volume depleted and will often require moderate
volumes of intra venous normal saline perioperatively. Once medically optimised the phaeochromocytoma should be removed. Most
adrenalectomies can now be performed using a laparoscopic approach(2). The adrenals are highly vascular structures and removal can
be complicated by catastrophic haemorrhage in the hands of the inexperienced. This is particularly true of right sided resections where
the IVC is perilously close. Should the IVC be damaged a laparotomy will be necessary and the defect enclosed within a Satinsky
style vascular clamp and the defect closed with prolene sutures. Attempting to interfere with the IVC using any instruments other than
vascular clamps will result in vessel trauma and make a bad situation much worse.

Incidental adrenal lesions


Adrenal lesions may be identified on CT scanning performed for other reasons(3). Factors suggesting benign disease on CT
include(4): Size less than 3cm. Homogeneous texture. Lipid rich tissue. Thin wall to lesion

All patients with incidental lesions should be managed jointly with an endocrinologist and full work up as described above. Patients
with functioning lesions or those with adverse radiological features (Particularly size >3cm) should proceed to surgery.

48. A 46 year old lady presents with symptoms of diarrhoea, weight loss of 10 Kg and a skin rash of erythematous blisters
involving the abdomen and buttocks. The blisters have an irregular border and both intact and ruptured vesicles. What is
the most likely diagnosis?
A. Colonic adenocarcinoma
B. Pancreatic adenocarcinoma
C. Tropical sprue
D. Glucagonoma
E. Insulinoma
Answer: D
Glucagonoma Is strongly associated with necrolytic migratory erythema.

Glucagonoma: Rare pancreatic tumours arising from the alpha cells of the pancreas.Glucagon levels markedly elevated. Symptoms
include diarrhoea, weight loss and necrolytic migratory erythema. A serum level of glucagon >1000pg/ml usually suggests the diagnosis,
imaging with CT scanning is also required. Treatment is with surgical resection.

49. A 56 year old man presents with symptoms of neuropathic facial pain and some weakness of the muscles of facial expression
on the right side. On examination he has a hard mass approximately 6cm anterior to the right external auditory meatus.
What is the most likely diagnosis?
A. Pleomorphic adenoma
B. Adenocarcinoma
C. Mucoepidermoid carcinoma
D. Adenoid cystic carcinoma
E. Lymphoma
Answer: D
The patient is most likely to have a malignant lesion within the parotid. Of the malignancies listed; adenoid cystic carcinoma has the
greatest tendency to perineural invasion.

Parotid gland malignancy: Most parotid neoplasms (80%) are benign lesions. Most commonly present with painless mass in cheek
region. Up to 30% may present with pain, when this is associated with a discrete mass lesion in the parotid it usually indicates
perineural invasion. Perineural invasion is very unlikely to occur in association with benign lesions. 80% of patients with facial nerve
weakness caused by parotid malignancies will have nodal metastasis and a 5 year survival of 25%
19
Types of malignancy
Mucoepidermoid carcinoma: 30% of all parotid malignancies. Usually low potential for local invasiveness and metastasis (depends
mainly on grade)
Adenoid cystic carcinoma:Unpredictable growth patter. Tendency for perineural spread. Nerve growth may display skip lesions
resulting in incomplete excision. Distant metastasis more common (visceral rather than nodal spread). 5 year survival 35%
Mixed tumours: Often a malignancy occurring in a previously benign parotid lesion
Acinic cell carcinoma:Intermediate grade malignancy. May show perineural invasion. Low potential for distant metastasis. 5 year
survival 80%
Adenocarcinoma:Develops from secretory portion of gland. Risk of regional nodal and distant metastasis. 5 year survival depends
upon stage at presentation, may be up to 75% with small lesions with no nodal involvement
Lymphoma:Large rubbery lesion, may occur in association with Warthins tumours. Diagnosis should be based on regional nodal
biopsy rather than parotid resection Treatment is with chemotherapy (and radiotherapy)

50. A 20 year old African lady undergoes an open appendicectomy. She is reviewed for an unrelated problem 8 months later. On
abdominal inspection the wound site is covered by shiny dark protuberant scar tissue that projects beyond the limits of the
skin incision. Which of the following is the most likely underlying process?
A. Hypertrophic scar
B. Keloid scar
C. Marjolins ulcer
D. Repeated episodes of wound sepsis
E. Mycosis fungoides
Answer: B
Keloid scars extend beyond the limits of the incision. Mycosis fungoides is a cutaneous T cell lymphoma.

Wound healing: Surgical wounds are either incisional or excisional and either clean, clean contaminated or dirty. Although the stages
of wound healing are broadly similar their contributions will vary according to the wound type.
The main stages of wound healing include:
Haemostasis: Vasospasm in adjacent vessels, platelet plug formation and generation of fibrin rich clot.
Inflammation: Neutrophils migrate into wound (function impaired in diabetes). Growth factors released, including basic fibroblast
growth factor and vascular endothelial growth factor. Fibroblasts replicate within the adjacent matrix and migrate into wound.
Macrophages and fibroblasts couple matrix regeneration and clot substitution.
Regeneration: Platelet derived growth factor and transformation growth factors stimulate fibroblasts and epithelial cells. Fibroblasts
produce a collagen network. Angiogenesis occurs and wound resembles granulation tissue.
Remodeling: Longest phase of the healing process and may last up to one year (or longer). During this phase fibroblasts become
differentiated (myofibroblasts) and these facilitate wound contraction.. Collagen fibres are remodeled. Microvessels regress leaving a
pale scar.
The above description represents an idealised scenario. A number of diseases may distort this process. It is obvious that one of the key
events is the establishing well vascularised tissue. At a local level angiogenesis occurs, but if arterial inflow and venous return are
compromised then healing may be impaired, or simply nor occur at all. The results of vascular compromise are all too evidence in
those with peripheral vascular disease or those poorly constructed bowel anastomoses.
Conditions such as jaundice will impair fibroblast synthetic function and overall immunity with a detrimental effect in most parts of
healing.
Problems with scars: Hypertrophic scars: Excessive amounts of collagen within a scar. Nodules may be present histologically
containing randomly arranged fibrils within and parallel fibres on the surface. The tissue itself is confined to the extent of the wound
itself and is usually the result of a full thickness dermal injury. They may go on to develop contractures. Keloid scars: Excessive
amounts of collagen within a scar. Typically a keloid scar will pass beyond the boundaries of the original injury. They do not contain
nodules and may occur following even trivial injury. They do not regress over time and may recur following removal.
Drugs which impair wound healing: Non steroidal anti inflammatory drugs. Steroids. Immunosupressive agents. Anti neoplastic
drugs
Closure: Delayed primary closure is the anatomically precise closure that is delayed for a few days but before granulation tissue
becomes macroscopically evident. Secondary closure refers to either spontaneous closure or to surgical closure after granulation
tissue has formed.

51. The pathogenicity of the tubercle bacillus is due to which of the following?
A. Necrosis caused by expanding granulomas
B. Ability to multiply within fibroblasts
C. Delayed hypersensitivity reaction against bacteria
D. Effect of antibody response
E. Direct toxic effect on host cells

20
Answer: C
Mycobacteria stimulate a specific T cell response of cell mediated immunity. This is effective in reducing the infection, the delayed
hypersensitivity also damages tissues. Necrosis occurs in TB but is usually within the granuloma.

Tuberculosis pathology: Is a form of primary chronic inflammation, caused by the inability of macrophages to kill the
Mycobacterium tuberculosis. The macrophages often migrate to regional lymph nodes, the lung lesion plus affected lymph nodes is
referred to as a Ghon complex.This leads to the formation of a granuloma which is a collection of epithelioid histiocytes. There is
the presence of caseous necrosis in the centre. The inflammatory response is mediated by a type 4 hypersensitivity reaction. In
healthy individuals the disease may be contained, in the immunocompromised disseminated (miliary TB) may occur.

Diagnosis: Waxy membrane of mycobacteria prevents binding with normal stains. Ziehl - Neelsen staining is typically used. Culture
based methods take far longer.

52. A 45 year old women with a thyroid carcinoma undergoes a total thyroidectomy. The post operative histology report shows
a final diagnosis of medullary type thyroid cancer. Which of the tests below is most likely to be of clinical use in screening
for disease recurrence?
A. Serum CA 19-9 Levels
B. Serum thyroglobulin levels
C. Serum PTH levels
D. Serum calcitonin levels
E. Serum TSH levels
Answer: D

Medullary thyroid cancers often secrete calcitonin and monitoring the serum levels of this hormone is useful in detecting sub clinical
recurrence.

53. A 15 year old boy undergoes an emergency splenectomy for trauma. He makes a full recovery and is discharged home. Eight
weeks post operatively the general practitioner performs a full blood count with a blood film. Which of the following is most
likely to be present?
A. Myofibroblasts
B. Howell-Jolly bodies
C. Multinucleate giant cells
D. Reed Sternberg Cells
E. None of the above
Answer: B
Post splenectomy blood film features: Howell- Jolly bodies. Pappenheimer bodies. Target cells. Irregular contracted erythrocytes
As the filtration function is the spleen is no longer present Howell-Jolly bodies are found.

Post splenectomy blood film changes: The loss of splenic tissue results in the inability to readily remove immature or abnormal
red blood cells from the circulation. The red cell count does not alter significantly. However, cytoplasmic inclusions may be seen
e.g. Howell-Jolly bodies. In the first few days after splenectomy target cells, siderocytes and reticulocytes will appear in the
circulation. Immediately following splenectomy a granulocytosis (mainly composed of neutrophils) is seen, this is replaced by a
lymphocytosis and monocytosis over the following weeks.
The platelet count is usually increased and this may be persistent, oral antiplatelet agents may be needed in some patients.

54. A 43 year old women is identified as being a carrier of a BRCA 1 mutation. Apart from breast cancer, which of the following
malignancies is she at greatest risk of developing?
A. Colonic cancer
B. Ovarian cancer
C. Follicular carcinoma of the thyroid
D. Pituitary adenoma
E. Phaeochromocytoma
Answer: B
BRCA 1 mutation patients are 55% more likely to get ovarian cancer. Those with BRCA 2 are 25% more likely. The risk of developing
other malignancies is slightly increased but not to the same extent, and not enough to justify screening.

Genetics and surgical disease: Some of the more commonly occurring genetic conditions occurring in surgical patients are presented
here.

21
Li-Fraumeni Syndrome: Autosomal dominant. Consists of germline mutations to p53 tumour suppressor gene. High incidence of
malignancies particularly sarcomas and leukaemias. Diagnosed when: *Individual develops sarcoma under 45 years. *First degree
relative diagnosed with any cancer below age 45 years and another family member develops malignancy under 45 years or sarcoma at
any age
BRCA 1 and 2: Carried on chromosome 17. Linked to developing breast cancer (60%) risk. Associated risk of developing ovarian
cancer (55% with BRCA 1 and 25% with BRCA2).
Lynch Syndrome: Autosomal dominant . Develop colonic cancer and endometrial cancer at young age. 80% of affected individuals
with get colonic and or endometrial cancer. High risk individuals may be identified using the Amsterdam criteria
Amsterdam criteria: Three or more family members with a confirmed diagnosis of colorectal cancer, one of whom is a first degree
(parent, child, sibling) relative of the other two. Two successive affected generations. One or more colon cancers diagnosed under age
50 years. Familial adenomatous polyposis (FAP) has been excluded.
Gardners syndrome: Autosomal dominant familial colorectal polyposis. Multiple colonic polyps. Extra colonic diseases include:
skull osteoma, thyroid cancer and epidermoid cysts. Desmoid tumours are seen in 15%. Mutation of APC gene located on
chromosome 5. Due to colonic polyps most patients will undergo colectomy to reduce risk of colorectal cancer. Now considered a
variant of familial adenomatous polyposis coli

55. A 53 year old man is due to undergo a splenectomy as a treatment for refractory haemolytic anaemia. The underlying
pathological basis for haemolytic anaemia is thought to be a Type 2 hypersensitivity response. Which of the following
mechanisms best describes this process
A. Deposition of immune complexes
B. Cell mediated immune response
C. IgE mediated response
D. Formation of autoantibodies against cell surface antigens
E. None of the above
Answer: D
Mnemonic for the reactions and the mediators involved
ACID EGG-T : Type 1 Anaphylactic. Type 2 Cytotoxic. Type 3 Immune complex. Type 4 Delayed type. EGG T (mediators): IgE.
IgG. IgG. T cells. Type 2 hypersensitivity reactions (which includes haemolytic anaemia) are associated with formation of antibody
against cell surface antigens.

56. A 25 year old man is injured in a road traffic accident. His right tibia is fractured and is managed by fasciotomies and
application of an external fixator. Over the next 48 hours his serum creatinine rises and urine is sent for microscopy, muddy
brown casts are identified. What is the most likely underlying diagnosis?
A. Acute interstitial nephritis
B. Acute tubular necrosis
C. Glomerulonephritis
D. IgA Nephropathy
E. Thin basement membrane disease
Answer: B
This patient is likely to have had compartment syndrome (tibial fracture + fasciotomies) which may produce myoglobinuria. The
presence of worsening renal function, together with muddy brown casts is strongly suggestive of acute tubular necrosis. Acute interstitial
nephritis usually arises from drug toxicity and does not usually produce urinary muddy brown casts. Thin basement membrane disease is
an autosomal dominant condition that causes persistent microscopic haematuria, but not worsening renal function.

Acute Renal Failure: Final pathway is tubular cell death. Renal medulla is a relatively hypoxic environment making it susceptible to
renal tubular hypoxia. Renovascular autoregulation maintains renal blood flow across a range of arterial pressures. Estimates of GFR are
best indices of level of renal function. Useful clinical estimates can be obtained by considering serum creatinine, age, race, gender and
body size. eGFR calculations such as the Cockcroft and Gault equation are less reliable in populations with high GFR's. Nephrotoxic
stimuli such as aminoglycosides and radiological contrast media induce apoptosis. Myoglobinuria and haemolysis result in necrosis.
Overlap exists and proinflammatory cytokines play and important role in potentiating ongoing damage. Post-operative renal failure is
more likely to occur in patients who are elderly, have peripheral vascular disease, high BMI, have COPD, receive vasopressors, are on
nephrotoxic medication or undergo emergency surgery. Avoiding hypotension will reduce risk of renal tubular damage. There is no
evidence that administration of ACE inhibitors or dopamine reduces the incidence of post-operative renal failure.

57. A 56 year old man has undergone a radical nephrectomy. The pathologist bisects the kidney and identifies a pink fleshy
tumour in the renal pelvis. What is the most likely disease?
A. Renal cell carcinoma
B. Transitional cell carcinoma
C. Angiomyolipoma
22
D. Phaeochromocytoma
E. Renal adenoma
Ansewr: B
Most renal tumours are yellow or brown in colour. TCC's are one of the few tumours to appear pink.
The finding of a TCC in the renal pelvis mandates a nephroureterectomy.

Renal cell carcinoma: Most present with haematuria (50%). Common renal tumour (85% cases). Paraneoplastic features include
hypertension and polycythaemia. Most commonly has haematogenous mestastasis. Treatment:Usually radical or partial ephrectomy

Nephroblastoma:Rare childhood tumour. It accounts for 80% of all genitourinary malignancies in those under the age of 15 years.
Up to 90% will have a mass. 50% will be hypertensive Diagnostic work up includes ultrasound and CT scanning. Treatment:Surgical
resection combined with chemotherapy (usually vincristine, actinomycin D and doxorubicin

Neuroblastoma: Most common extracranial tumour of childhood. 80% occur in those under 4 years of age. Tumour of neural crest
origin (up to 50% occur in the adrenal gland). The tumour is usually calcified and may be diagnosed using MIBG scanning. Staging is
with CT:Surgical resection, radiotherapy and chemotherapy

Transitional cell carcinoma: Accounts for 90% of upper urinary tract tumour, but only 10% of renal tumours. Males affected 3x
more than females. Occupational exposure to industrial dyes and rubber chemicals may increase risk. Up to 80% present with painless
haematuria. Diagnosis and staging is with CT IVU. Treatment: Radical nephroureterectomy

Angiomyolipoma:80% of these hamartoma type lesions occur sporadically, the remainder are seen in those with tuberous sclerosis.
Tumour is composed of blood vessels, smooth muscle and fat. Massive bleeding may occur in 10% of cases. Treatment: 50% of
patients with lesions >4cm will have symptoms and will require surgical resection

58. A 65 year old lady presents with a lesion affecting her right breast. On examination she has a weeping, crusting lesion
overling the right nipple, the areolar region is not involved. There is no palpable mass lesion in the breast, there is a palpable
axillary lymph node. The patients general practitioner has tried treating the lesion with 1% hydrocortisone cream, with no
success. What is the most likely diagnosis?
A. Infection with Staphylococcus aureus
B. Pagets disease of the nipple
C. Phyllodes tumour
D. Nipple eczema
E. Basal cell carcinoma
Answer: B
A weeping, crusty lesion such as this is most likely to represent Pagets disease of the nipple (especially since the areolar region is
spared). Although no mass lesion is palpable, a proportion of patients will still have an underlying invasive malignancy (hence the
lymphadenopathy).

Pagets disease of the nipple: Pagets disease is an eczematoid change of the nipple associated with an underlying breast malignancy
and it is present in 1-2% of patients with breast cancer. In half of these patients, it is associated with an underlying mass lesion and
90% of such patients will have an invasive carcinoma. 30% of patients without a mass lesion will still be found to have an
underlying carcinoma. The remainder will have carcinoma in situ.
Pagets disease differs from eczema of the nipple in that it involves the nipple primarily and only latterly spreads to the areolar (the
opposite occurs in eczema). Diagnosis is made by punch biopsy, mammography and ultrasound of the breast.
Treatment will depend on the underlying lesion.

59. A 73 year old man presents with haemoptysis and is suspected of suffering from lung cancer. On examination he has an
enlarged supraclavicular lymph node. Which of the following features is most likely to be present on histological
examination?
A. Increased mitoses
B. Apoptosis
C. Barr Bodies
D. Multinucleate giant cells
E. Granuloma

23
Answer: A
Increased mitoses are commonly seen in association with malignant transformation of cells. Apoptosis is not a common feature of
metastatic cancer. Barr Bodies are formed during X chromosome inactivation in female somatic cells.
Histopathology of malignancy: Abnormal tissue architecture. Coarse chromatin. Invasion of basement membrane*. Abnormal
mitoses. Angiogenesis. De-differentiation. Areas of necrosis. Nuclear pleomorphism

*= Those features that distinguish invasive malignancy from in situ disease

60. Which of the following pathological explanations best describes the initial pathological processes occurring in an abdominal
aortic aneurysm in an otherwise well 65 year old, hypertensive male?
A. Loss of elastic fibres from the adventitia
B. Loss of collagen from the adventitia
C. Loss of collagen from the media
D. Loss of elastic fibres from the media
E. Decreased matrix metalloproteinases in the adventitia
Answer: D
In established aneurysmal disease there is dilation of all layers of the arterial wall and loss of both elastin and collagen. The primary
event is loss of elastic fibres with subsequent degradation of collagen fibres.

Pathology of abdominal aortic aneurysm: Abdominal aortic aneurysms occur primarily as a result of the failure of elastic proteins
within the extracellular matrix. Anuerysms typically represent dilation of all layers of the arterial wall. Most aneurysms are caused
by degenerative disease. After the age of 50 years the normal diameter of the infrarenal aorta is 1.5cm in females and 1.7cm in
males. Diameters of 3cm and greater, are considered aneurysmal. The pathophysiology involved in the development of aneurysms is
complex and the primary event is loss of the intima with loss of elastic fibres from the media. This process is associated with, and
potentiated by, increased proteolytic activity and lymphocytic infiltration.

Major risk factors for the development of aneurysms include smoking and hypertension. Rare but important causes include syphilis
and connective tissues diseases such as Ehlers Danlos type 1 and Marfans syndrome.

61. A 28 year old lady has a malignant melanoma removed from her calf. Which of the following pathological criteria carries
the greatest prognostic weighting?
A. Vascular invasion
B. Abnormal mitoses
C. Breslow thickness
D. Perineural invasion
E. Lymphocytic infiltrates
Answer: C
The Breslow thickness has considerable prognostic importance. Lymphocytic infiltrates may be associated with an improved
prognosis, but do not carry nearly the same weight as increased thickness.

62. A 34 year old lady undergoes an elective cholecystectomy for attacks of recurrent cholecystitis due to gallstones. Microscopic
assessment of the gallbladder is most likely to show which of the following?
A. Dysplasia of the fundus
B. Widespread necrosis
C. Ashoff-Rokitansky sinuses
D. Metaplasia of the fundus
E. None of the above
Answer: C
Aschoff-Rokitansky sinuses are the result of hyperplasia and herniation of epithelial cells through the fibromuscular layer of the
gallbladder wall. They may be macroscopic or microscopic. Ashoff-Rokitansky sinuses may be identified in cases of chronic
cholecystitis and gallstones. Although gallstones may predispose to the development of gallbladder cancer the actual incidence of
dysplasia and metaplastic change is rare. In the elective setting described above necrosis would be rare.

Gallbladder: Fibromuscular sac with capacity of 50ml. Columnar epithelium. Relations of the gallbladder: Anterior: Liver.
Posterior: Covered by peritoneum. Transverse colon. 1st part of the duodenum. Laterally: Right lobe of liver. Medially: Quadrate
lobe of liver. Arterial supply: Cystic artery (branch of Right hepatic artery). Venous drainage: Cystic vein. Nerve supply:
Sympathetic- mid thoracic spinal cord, Parasympathetic- anterior vagal trunk
Common bile duct: Origin: Confluence of cystic and common hepatic ducts. Relations at origin: Medially - Hepatic artery.
Posteriorly- Portal vein. Relations distally:Duodenum – anteriorly. Pancreas - medially and laterally Right renal vein – posteriorly.
Arterial supply: Branches of hepatic artery and retroduodenal branches of gastroduodenal artery.
24
Calot's triangle: Medially: Common hepatic duct. Inferiorly:Cystic duct. Superiorly:Inferior edge of liver. Contents:Cystic artery

63. Which of the following are not true of Li-Fraumeni syndrome?


A. It consists of mutations to the p53 tumour suppressor gene
B. Is likely to be present in a teenager presenting with a liposarcoma
C. It has an autosomal dominant inheritance pattern
D. Affected individuals are unlikely to develop acute myeloid leukaemia
E. Adrenal malignancies are more common than in normal population
Answer: D

They are at high risk of developing leukaemia.

64. A 35 year old type 1 diabetic presents with difficulty mobilising and back pain radiating to the thigh. He has a temperature
of 39 oC and has pain on extension of the hip. He is diagnosed with an iliopsoas abscess. Which of the following statements is
false in relation to his diagnosis?
A. Staphylococcus aureus is the most likely primary cause
B. Recurrence occurs in 60% cases
C. More common in males
D. Crohn's is the most likely secondary cause
E. CT guided drainage is preferable first line management
Answer: B
Classical features include: a limp, back pain and fever. Recurrence rates are about 15-20%. Staphylococcus is the commonest
primary cause, others include Streptococcus and E.coli. Management is ideally by CT guided drainage.

Iliopsoas abscess: Collection of pus in iliopsoas compartment (iliopsoas and iliacus): Causes: Primary: Haematogenous spread of
bacteria. Staphylococcus aureus: most common. Secondary: Crohn's (commonest cause in this category). Diverticulitis, Colorectal
cancer. UTI, GU cancers. Vertebral osteomyelitis. Femoral catheter, lithotripsy. Endocarditis. Note the mortality rate can be up to
19-20% in secondary iliopsoas abscesses compared with 2.4% in primary abscesses. Clinical features: Fever. Back/flank pain.
Limp. Weight loss

Clinical examination: Patient in the supine position with the knee flexed and the hip mildly externally rotated. Specific tests to
diagnose iliopsoas inflammation: Place hand proximal to the patient's ipsilateral knee and ask patient to lift thigh against your hand.
This will cause pain due to contraction of the psoas muscle. Lie the patient on the normal side and hyperextend the affected hip. In
inflammation this should elicit pain as the psoas muscle is stretched.

Investigation: CT is gold standard

Management: Antibiotics. Percutaneous drainage. Surgery is indicated if: 1. Failure of percutaneous drainage. 2. Presence of an
another intra-abdominal pathology which requires surgery

Surgical approach: The authors technique for draining these collections is given here. Review the CT scans and plan surgical
approach. An extraperitoneal approach is important. The collection usually extends inferiorly and can be accessed from an incision
at a level of L4 on the affected side. GA. Transverse laterally placed incision. Incise external oblique. Split the subsequent muscle
layers.
As you approach the peritoneum use blunt dissection to pass laterally around it. Remember the ureter and gonadal veins lie posterior
at this level. Eventually you will enter the abscess cavity, a large amount of pus is usually released at this point. Drain the area with
suction and washout with saline. Place a corrugated drain well into the abscess cavity. If you have made a small skin incision it is
reasonable to bring the drain up through the skin wound. Otherwise place a lateral exit site and close the skin and external oblique. If
you do this ensure that you use interrupted sutures. Anchor the drain with strong securely tied silk sutures (it is extremely tiresome if
it falls out!)

65. Which of the following statements relating to parathyroid neoplasms is incorrect?


A. 15% of cases are due to parathyroid carcinoma
B. 80% of cases are due to parathyroid adenomas
C. Parathyroid adenomas are often encapsulated
D. 10% of parathyroid adenomas develop in ectopically located glands

25
E. 85% of cases of primary hyperparathyroidism are due to solitary adenomas
Answer: A
Parathyroid carcinomas account for up to 5% of tumours. Adenomas are often encapsulated .Lesions that are fibrotic and densely
adherent to the gland may be a carcinoma. 85% cases of primary hyperparathyroidism are due to a single adenoma and this is the
reason some surgeons favour a focussed parathyroidectomy.

Parathyroid glands and disorders of calcium metabolism

Primary hyperparathyroidism: PTH (Elevated). Ca2+ (Elevated). Phosphate (Low). Serum Calcium: Creatinine clearance ratio >
0.01. Clinical features: May be asymptomatic if mild. Recurrent abdominal pain (pancreatitis, renal colic). Changes to emotional
or cognitive state. Causes: Most cases due to solitary adenoma (80%), multifocal disease occurs in 10-15% and parathyroid
carcinoma in 1% or less. Management: Indications for surgery: Elevated serum Calcium > 1mg/dL above normal. Hypercalciuria
> 400mg/day. Creatinine clearance < 30% compared with normal. Episode of life threatening hypercalcaemia. Nephrolithiasis. Age
< 50 years. Neuromuscular symptoms. Reduction in bone mineral density of the femoral neck, lumbar spine, or distal radius of more
than 2.5 standard deviations below peak bone mass (T score lower than -2.5)

Secondary hyperparathyroidism: PTH (Elevated). Ca2+ (Low or normal). Phosphate (Elevated). Vitamin D levels (Low). Clinical
features: May have few symptoms. Eventually may develop bone disease, osteitis fibrosa cystica and soft tissue calcifications.
Causes: Parathyroid gland hyperplasia occurs as a result of low calcium, almost always in a setting of chronic renal failure.
Management: Usually managed with medical therapy. Indications for surgery in secondary (renal) hyperparathyroidism: Bone
pain. Persistent pruritus. Soft tissue calcifications

Tertiary hyperparathyroidism: Ca2+ (Normal or high). PTH (Elevated). Phosphate levels (Decreased or Normal). Vitamin D
(Normal or decreased). Alkaline phosphatase (Elevated). Clinical features: Metastatic calcification. Bone pain and / or fracture.
Nephrolithiasis. Pancreatitis. Causes: Occurs as a result of ongoing hyperplasia of the parathyroid glands after correction of
underlying renal disorder, hyperplasia of all 4 glands is usually the cause. Management: Usually treatment is surgical. The
presence of an autonomously functioning parathyroid gland may require surgery. If the culprit gland can be identified then it should
be excised. Otherwise total parathyroidectomy and re-implantation of part of the gland may be required.

Differential diagnoses
It is important to consider the rare but relatively benign condition of benign familial hypocalciuric hypercalcaemia, caused by an
autosomal dominant genetic disorder. Diagnosis is usually made by genetic testing and concordant biochemistry (Serum Calcium:
Creatinine clearance ratio <0.01-distinguished from primary hyperparathyroidism).

66. A 20 year old girl presents with a thyroid cancer, she is otherwise well with no significant family history. On examination she
has a nodule in the left lobe of the thyroid with a small discrete mass separate from the gland itself. Which of the following is
the most likely cause?
A. Follicular carcinoma
B. Anaplastic carcinoma
C. Medullary carcinoma
D. Papillary carcinoma
E. B Cell Lymphoma

Answer: D
Papillary carcinoma is the most common subtype and may cause lymph node metastasis (mass separate from the gland itself) that is
rare with follicular tumours. Anaplastic carcinoma would cause more local symptoms and would be rare in this age group.

67. A 28 year old lady is breast feeding her first child. She presents with discomfort of the right breast. Clinical examination
demonstrates erythema and an area that is fluctuant. Aspiration and culture of the fluid is most likely to demonstrate
infection with which of the following organisms?
A. Clostridium perfringens
B. Staphylococcus aureus
C. Streptococcus pyogenes
D. Staphylococcus epidermidis
E. Actinomycosis
Answer: B
Staphylococcus aureus is the commonest cause. The infants mouth is usually the source as it damages the nipple areolar complex
allowing entry of bacteria.

26
Breast abscess: In lactational women Staphylococcus aureus is the most common cause. Typical presentation is with a tender mass
in a lactating women. There is often tenderness and pain and a fluctuant mass. Diagnosis and treatment is performed using USS and
associated drainage of the abscess cavity. Antibiotics should also be administered. Where there is necrotic skin overlying the
abscess, the patient should undergo surgery

68. An 18 year old rock climber falls onto his left arm and sustains a large haematoma of the left upper arm. Unfortunately the
wound associated with the injury is neglected and it becomes infected. Which of the following changes is least likely to
occur?
A. Axillary lymphadenopathy
B. Leucopenia
C. Tenderness
D. Mild pyrexia
E. Local formation of yellow pus
Answer: B
Leucopenia would be unusual and should prompt a search for another cause.

Theme: Tumour markers

A. Invasive ductal carcinoma of the breast


B. Prostate cancer
C. Gastric cancer
D. Ovarian cancer
E. Colorectal cancer
F. Pancreatic adenocarcinoma
G. Seminoma testicular cancer
H. Non-seminomatous testicular cancer
I. Hepatocellular carcinoma

For each tumour marker please select the most likely underlying malignancy. Each option may be used once, more than once or not
at all.

69. Raised beta-human chorionic gonadotropin with a raised alpha-feto protein level
Answer: Non-seminomatous testicular cancer
A raised alpha-feto protein level excludes a seminoma

70. Elevated CA 19-9


Answer: Pancreatic adenocarcinoma

71. Raised alpha-feto protein level in a 54-year-old woman


Answer: Hepatocellular carcinoma

Tumour markers may be divided into: monoclonal antibodies against carbohydrate or glycoprotein tumour antigens. Tumour
antigens. enzymes (alkaline phosphatase, neurone specific enolase). Hormones (e.g. calcitonin, ADH)
It should be noted that tumour markers usually have a low specificity
Monoclonal antibodies: CA 125: Ovarian cancer. CA 19-9: Pancreatic cancer. CA 15-3: Breast cancer. NB: The breast cancer
tumour marker is not specific or sensitive enough to be used routinely.
Tumour antigens: Prostate specific antigen (PSA): Prostatic carcinoma. Alpha-feto protein (AFP): Hepatocellular carcinoma,
teratoma. Carcinoembryonic antigen (CEA):Colorectal cancer

Theme: Benign breast lesions

A. Fibroadenoma
B. Breast abscess
C. Cyst of Montgomery's gland
D. Galactocele
E. Lipoma
F. Duct ectasia
G. Intraductal papilloma
H. Fat necrosis
27
What is the most likely diagnosis for the scenario given? Each option may be used once, more than once or not at all.

72. A 64 year old obese female presents with a breast lump. She was hit on the breast by a cricket ball when playing with her
grandson.
Answer: Fat necrosis
An obese, post menopausal woman, with a history of trauma points towards fat necrosis. Trauma causes inflammation of fat cells,
leading to formation of a lump. Mammography will be needed to differentiate it from breast disease.

73. A 21 year old female notices a bloody discharge from the nipple. She is otherwise well. On examination there are no discrete
lesions to feel and mammography shows dense breast tissue but no mass lesion.
Answer: Intraductal papilloma
Intraductal papillomata are the commonest cause of blood stained nipple discharge in younger women. There is seldom any palpable
mass. An ultrasound is required and possibly a galactogram.

74. A 18 year old female notices a non tender mobile breast lump. Clinically there is a smooth lump which is not tethered to the
skin.
Answer: Fibroadenoma
Also called a breast 'mouse' due to its mobility. It is a benign condition arising from the breast lobule. May enlarge in pregnancy.

Benign Breast lesions


Fibroadenoma: Develop from a whole lobule. Mobile, firm breast lumps. 12% of all breast masses. Over a 2 year period up to 30%
will get smaller. No increase in risk of malignancy. Treatment: If >3cm surgical excision is usual, Phyllodes tumours should be
widely excised (mastectomy if the lesion is large)
Breast cyst: 7% of all Western females will present with a breast cyst. Usually presents as a smooth discrete lump (may be
fluctuant). Small increased risk of breast cancer (especially if younger). Treatment: Cysts should be aspirated, those which are
blood stained or persistantly refill should be biopsied or excised
Sclerosing adenosis, (radial scars and complex sclerosing lesions):Usually presents as a breast lump or breast pain. Causes
mammographic changes which may mimic carcinoma. Cause distortion of the distal lobular unit, without hyperplasia (complex
lesions will show hyperplasia). Considered a disorder of involution, no increase in malignancy risk. Treatment: Lesions should be
biopsied, excision is not mandatory
Epithelial hyperplasia:Variable clinical presentation ranging from generalised lumpiness through to discrete lump. Disorder
consists of increased cellularity of terminal lobular unit, atypical features may be present. Atypical features and family history of
breast cancer confers greatly increased risk of malignancy. Treatment :If no atypical features then conservative, those with atypical
features require either close monitoring or surgical resection
Fat necrosis: Up to 40% cases usually have a traumatic aetiology. Physical features usually mimic carcinoma. Mass may increase
in size initially. Treatment: Imaging and core biopsy
Duct papilloma: Usually present with nipple discharge. Large papillomas may present with a mass. The discharge usually
originates from a single duct. Treatment: No increase risk of malignancy: Microdochectomy

75. A 17 year old man is identified as having a Meckels diverticulum. From which of the following embryological structures is it
derived?
A. Foregut
B. Hindgut
C. Uranchus
D. Cloaca
E. Vitello-intestinal duct
Answer: E
Rule of 2's: 2% of population. 2 inches (5cm) long. 2 feet (60 cm) from the ileocaecal valve. 2 x's more common in men. 2 tissue
types involved

The Meckels diverticulum is a persistence of the vitello-intestinal duct. Meckel's diverticulum: Congenital abnormality resulting in
incomplete obliteration of the vitello-intestinal duct. Normally, in the foetus, there is an attachment between the vitello-intestinal
duct and the yolk sac.This disappears at 6 weeks gestation. The tip is free in majority of cases. Associated with enterocystomas,
umbilical sinuses, and omphaloileal fistulas. Arterial supply: omphalomesenteric artery. 2% of population, 2 inches long, 2 feet
from the ileocaecal valve. Typically lined by ileal mucosa but ectopic gastric mucosa can occur, with the risk of peptic ulceration.
Pancreatic and jejunal mucosa can also occur. Clinical: Normally asymptomatic and an incidental finding. Complications are the
result of obstruction, ectopic tissue, or inflammation. Removal if narrow neck or symptomatic. Options are between wedge excision
or formal small bowel resection and anastomosis.
28
Theme: Rectal bleeding

A. Solitary rectal ulcer syndrome


B. Haemorroidal disease
C. Fissure in ano
D. Fistula in ano
E. Anal cancer
F. Ulcerative colitis

Please select the most likely diagnosis for the scenario given. Each option may be used once, more than once or not at all.

76. A 22 year old man presents with a 6 day history of passage of bloody diarrhoea with passage of mucous and slime. He is
passing an average of 8 to 9 bowel movements per day. On digital rectal examination there is no discrete abnormality to feel,
but there is some blood stained mucous on the glove.
Answer: Ulcerative colitis
The passage of bloody diarrhoea together with mucous and a short history makes this a likely first presentation of inflammatory
bowel disease.

77. A 17 year old man presents with a 2 week history of significant pain on defecation accompanied by the presence of a small
amount of blood which is noticed on toilet paper.
Answer: Fissure in ano
Young patients with painful rectal bleeding may have a fissure. Treatment is with stool softeners and either GTN or Diltiazem
initially.

78. A 24 year old women presents with a long history of obstructed defecation and chronic constipation. She often strains to
open her bowels for long periods and occasionally notices that she has passed a small amount of blood. On examination she
has an indurated area located anteriorly approximately 3cm proximal to the anal verge.
Answer: Solitary rectal ulcer syndrome
Solitary rectal ulcers are associated with chronic constipation and straining. It will need to be biopsied to exclude malignancy (the
histological appearances are characteristic). Diagnostic work up should include endoscopy and probably defecating proctogram and
ano-rectal manometry studies.

Rectal bleeding: is a common cause for patients to be referred to the surgical clinic. In the clinical history it is useful to try and
localise the anatomical source of the blood. Bright red blood is usually of rectal anal canal origin, whilst dark red blood is more
suggestive of a proximally sited bleeding source. Blood which has entered the GI tract from a gastro-duodenal source will typically
resemble malaena due to the effects of the digestive enzymes on the blood itself.

In the table below we give some typical bleeding scenarios together with physical examination findings and causation.
Type of
Cause Features in history Examination findings
bleeding
Painful bleeding that occurs post defecation Muco-epithelial defect usually in the
Fissure in Bright red
in small volumes. Usually antecedent midline posteriorly (anterior fissures more
ano rectal bleeding
features of constipation likely to be due to underlying disease)
Post defecation bleeding noted both on
toilet paper and drips into pan. May be Normal colon and rectum. Proctoscopy may
Bright red
Haemorroids alteration of bowel habit and history of show internal haemorrhoids. Internal
rectal bleeding
straining. No blood mixed with stool. No haemorrhoids are usually impalpable.
local pain.
Bleeding that is accompanied by other Perineal inspection may show fissures or
Crohns Bright red or symptoms such as altered bowel habit, fistulae. Proctoscopy may demonstrate
disease mixed blood malaise, history of fissures (especially indurated mucosa and possibly strictures.
anterior) and abscesses. Skip lesions may be noted at colonoscopy.
Bright red
Proctitis is the most marked finding. Peri
Ulcerative bleeding often Diarrhoea, weight loss, nocturnal
anal disease is usually absent. Colonoscopy
colitis mixed with incontinence, passage of mucous PR.
will show continuous mucosal lesion.
stool

29
Usually obvious mucosal abnormality.
Bright red Lesion may be fixed or mobile depending
Alteration of bowel habit. Tenesmus may
Rectal cancer blood mixed upon disease extent. Surrounding mucosa
be present. Symptoms of metastatic disease.
volumes often normal, although polyps may be
present.

Investigation: All patients presenting with rectal bleeding require digital rectal examination and procto-sigmoidoscopy as a minimal
baseline. Remember that haemorrhoids are typically impalpable and to attribute bleeding to these in the absence of accurate internal
inspection is unsatisfactory. In young patients with no other concerning features in the history a carefully performed sigmoidoscopy
that demonstrates clear haemorrhoidal disease may be sufficient. If clear views cannot be obtained then patients require bowel
preparation with an enema and a flexible sigmoidscopy performed. In those presenting with features of altered bowel habit or
suspicion of inflammatory bowel disease a colonoscopy is the best test. Patients with excessive pain who are suspected of having a
fissure may require an examination under general or local anaesthesia. In young patients with external stigmata of fissure and a
compatible history it is acceptable to treat medically and defer internal examination until the fissure is healed. If the fissure fails to
heal then internal examination becomes necessary along the lines suggested above to exclude internal disease.

Special tests: In patients with a malignancy of the rectum the staging investigations comprise an MRI of the rectum to identify
circumferential resection margin compromise and to identify mesorectal nodal disease. In addition to this CT scanning of the chest
abdomen and pelvis is necessary to stage for more distant disease. Some centres will still stage the mesorectum with endo rectal
ultrasound but this is becoming far less common. Patients with fissure in ano who are being considered for surgical sphincterotomy
and are females who have an obstetric history should probably have ano rectal manometry testing performed together with endo anal
ultrasound. As this service is not universally available it is not mandatory but in the absence of such information there are
continence issues that may arise following sphincterotomy.

Disease Management
Fissure in ano GTN ointment 0.2% or diltiazem cream applied topically is the usual first line treatment. Botulinum toxin
for those who fail to respond. Internal sphincterotomy for those who fail with botox, can be considered at the
botox stage in males.
Haemorroids Lifestyle advice, for small internal haemorrhoids can consider injection sclerotherapy or rubber band
ligation. For external haemorrhoids consider haemorrhoidectomy. Modern options include HALO procedure
and stapled haemorrhoidectomy.
Inflammatory Medical management- although surgery may be needed for fistulating Crohns (setons).
bowel disease
Rectal cancer Anterior resection or abdomino-perineal excision of the colon and rectum. Total mesorectal excision is now
standard of care. Most resections below the peritoneal reflection will require defunctioning ileostomy. Most
patients will require preoperative radiotherapy.

79. Which of the following is the most common childhood brain tumour?
A. Glioblastoma multiforme
B. Astrocytoma
C. Medulloblastoma
D. Ependymoma
E. Meningioma
Answer: C
Glioblastoma multiforme is rare in childhood. In contrast, medulloblastoma (more correctly termed primitive neuroectodermal
tumours) is the commonest brain tumour in children, and the 2nd commonest malignant solid neoplasm in children.

CNS tumours: 60% = Glioma and metastatic disease. 20% = Meningioma. 10% = Pituitary lesions. In paediatric practice
medulloblastomas (neuroectodermal tumours) are the commonest lesion, they are very rare in adults. Tumours arising in right
temporal and frontal lobe may reach considerable size before becoming symptomatic. Whereas tumours in the speech and visual
areas will typically produce early symptoms. Diagnosis : MRI Scanning provides the best resolution. Treatment: Usually surgery,
even if tumour cannot be completely resected conditions such as rising ICP can be addressed with tumour debulking and survival
and quality of life prolonged. Curative surgery can usually be undertaken with lesions such as meningiomas. Gliomas have a marked
propensity to invade normal brain and resection of these lesions is nearly always incomplete.

30
80. A keen surgical trainee is about to embark on her first hemi arthroplasty for a fractured neck of femur. In the anaesthetic
room the patient is given 1.2g intravenous co-amoxiclav. There is a possible history of penicillin allergy but the patient is
demented and the history is not checked. The patient then develops severe respiratory compromise and haemodynamic
collapse. Which of the following pathological processes accounts for this event?
A. Binding of the drug to circulating IgG class antibodies
B. Recognition of the drug by IgE receptors on mast cells
C. Drug initiated formation of hapten-protein complexes
D. Binding of the drug to circulating IgM class antibodies
E. None of the above
Answer: B
Anaphylactic shock: Antigen recognised by IgE molecules on the surface of mast cells resulting in rapid degranulation with release
of histamine and other inflammatory cytokines. This is a case of anaphylactic shock. In anaphylaxis the mast cells degranulate.
Anaphylactic shock: Suspect if there has been exposure to an allergen

Management: Remove allergen. ABCD. Drugs: Adrenaline 1:1000 0.5ml INTRAMUSCULARLY (not IV). Repeat after 5 mins if
no response. Then Chlorpheniramine 10mg IV. Then Hydrocortisone 100-200mg IV

81. Which of the following hepatobiliary disorders are most classically associated with ulcerative colitis?
A. Gallstones
B. Primary sclerosing cholangitis
C. Bile duct stones
D. Liver hamartomas
E. Hepatocellular carcinoma
Answer: B

Primary sclerosing cholangitis is an idiopathic inflammation of the bile ducts. It may result in episodes of cholestasis and cholangitis
and ultimately result in the need for liver transplantation. It carries a 10% risk of malignant transformation. Crohns disease is
associated with gallstones due to impaired entero-hepatic circulation. Apart from PSC, ulcerative colitis does not increase the risk of
other liver lesions.

Ulcerative colitis: Ulcerative colitis is a form of inflammatory bowel disease. Inflammation always starts at rectum, never spreads
beyond ileocaecal valve and is continuous. The peak incidence of ulcerative colitis is in people aged 15-25 years and in those aged
55-65 years. It is less common in smokers. The initial presentation is usually following insidious and intermittent symptoms.
Features include: bloody diarrhea. Urgency. Tenesmus. Abdominal pain, particularly in the left lower quadrant.

Extra-intestinal features: questions regarding the 'extra-intestinal' features of inflammatory bowel disease are common. Extra-
intestinal features include sclerosing cholangitis, iritis and ankylosing spondylitis. Related to disease activity: Common to both
Crohn’s disease and ulcerative colitis: Arthritis: pauciarticular, asymmetric. Erythema nodosum. Episcleritis. Osteoporosis. Note:
Arthritis is the most common extra-intestinal feature in both CD and UC
Episcleritis is more common in CD. Unrelated to disease activity: Common to both Crohn’s disease and ulcerative colitis:
Arthritis: polyarticular, symmetric. Uveitis. Pyoderma gangrenosum. Clubbing. Primary sclerosing cholangitis. Note: Primary
sclerosing cholangitis is much more common in UC. Uveitis is more common in UC

Pathology: Red, raw mucosa, bleeds easily. No inflammation beyond submucosa (unless fulminant disease). Widespread superficial
ulceration with preservation of adjacent mucosa which has the appearance of polyps ('pseudopolyps'). Inflammatory cell infiltrate in
lamina propria. Neutrophils migrate through the walls of glands to form crypt abscesses. Depletion of goblet cells and mucin from
gland epithelium. Granulomas are infrequent. Barium enema: loss of haustrations. superficial ulceration, 'pseudopolyps'. long
standing disease: colon is narrow and short -'drainpipe colon'. Endoscopy: Superficial inflammation of the colonic and rectal
mucosa. Continuous disease from rectum proximally. Superifical ulceration, mucosal islands, loss of vascular definition and
continuous ulceration pattern. Management: Patients with long term disease are at increased risk of development of malignancy.
Acute exacerbations are generally managed with steroids, in chronic patients agents such as azathioprine and infliximab may be
used. Individuals with medically unresponsive disease usually require surgery- in the acute phase a sub total colectomy and end
ileostomy. In the longer term a proctectomy will be required. An ileoanal pouch is an option for selected patients

82. Which of the following is not associated with thrombosis?


A. Endothelial cell damage
B. Use of tourniquets in surgery
31
C. Formation of platelet aggregates
D. Thrombocytopenia
E. Carcinoma of the stomach
Answer: D

All the other options either act directly to promote thrombosis e.g. endothelial cell damage or via changes in consistency or flow of

Cause Factors affected Disorder APTT PT Bleeding time


Heparin Prevents activation factors 2,9,10,11 Haemophilia Increased Normal Normal
Warfarin Affects synthesis of factors 2,7,9,10 von Willebrand's Increased Normal Increased
DIC Factors 1,2,5,8,11 disease
Liver disease Factors 1,2,5,7,9,10 Vitamin K deficiency Increased Increased Normal
blood.

83. A 16 year old boy develops a painful swelling of his distal femur. An osteoblastic sarcoma is diagnosed. To which of the
following sites is this lesion most likely to metastasise?
A. Inguinal lymph nodes
B. Common iliac lymph nodes
C. Liver
D. Brain
E. Lung
Answer: E
Sarcomas in which Lymphatic Metastasis is seen:

'RACE For MS': R: Rhabdomyosarcoma. A: Angiosarcoma. C: Clear cell sarcoma. E: Epithelial cell sarcoma. For: Fibrosarcoma
M: Malignant fibrous histiocytoma. S: Synovial cell sarcoma Or 'SCARE': Synovial sarcoma. Clear cell sarcoma. Angiosarcoma.
Rhabdomyosarcoma. Epithelioid sarcoma. Sarcomas often metastasise via the haematogenous route and the lung is a common site
for sarcoma metastasis. The liver and brain are often spared (at least initially). A smaller number may develop lymphatic metastasis
(see above).
Sarcomas: Malignant tumours of mesenchymal origin. Types: May be either bone or soft tissue in origin. Bone sarcoma include:
Osteosarcoma. Ewings sarcoma (although non bony sites recognised). Chrondrosarcoma - originate from Chondrocytes. Soft tissue
sarcoma are a far more heterogeneous group and include: Liposarcoma-adipocytes. Rhabdomyosarcoma-striated muscle.
Leiomyosarcoma-smooth muscle. Synovial sarcomas- close to joints (cell of origin not known but not synovium). Malignant fibrous
histiocytoma is a sarcoma that may arise in both soft tissue and bone. Features: Certain features of a mass or swelling should raise
suspicion for a sarcoma these include: Large >5cm soft tissue mass. Deep tissue location or intra muscular location. Rapid growth.
Painful lump. Assessment: Imaging of suspicious masses should utilise a combination of MRI, CT and USS. Blind biopsy should
not be performed prior to imaging and where required should be done in such a way that the biopsy tract can be subsequently
included in any resection.

Ewings sarcoma: Commoner in males. Incidence of 0.3 / 1, 000, 000. Onset typically between 10 and 20 years of age. Location by
femoral diaphysis is commonest site. Histologically it is a small round tumour. Blood borne metastasis is common and
chemotherapy is often combined with surgery

Osteosarcoma: Mesenchymal cells with osteoblastic differentiation. 20% of all primary bone tumours. Incidence of 5 per
1,000,000. Peak age 15-30, commoner in males. Limb preserving surgery may be possible and many patients will receive
chemotherapy

Liposarcoma: Malignancy of adipocytes. Rare approximately 2.5 per 1,000,000. They are the second most common soft tissue
sarcoma. Typically located in deep locations such as retroperitoneum. Affect older age group usually >40 years of age. May be well
differentiated and thus slow growing although may undergo dedifferentiation and disease progression. Many tumours will have a
pseudocapsule that can misleadingly allow surgeons to feel that they can 'shell out' these lesions. In reality tumour may invade at the
edge of the pseudocapsule and result in local recurrence if this strategy is adopted. Usually resistant to radiotherapy although this is
often used in a palliative setting

Malignant Fibrous Histiocytoma: Tumour with large number of histiocytes. Most common sarcoma in adults. Also described as
undifferentiated pleomorphic sarcoma NOS (i.e. Cell of origin is not known). Four major subtypes are recognised: storiform-

32
pleomorphic (70% cases), myxoid (less aggressive), giant cell and inflammatory. Treatment is usually with surgical resection and
adjuvant radiotherapy as this reduces the likelihood of local recurrence

84. Infection with which of the following micro-organisms may result in a clinical picture resembling achalasia of the
oesphagus?
A. Epstein Barr virus
B. Wuchereria Bancrofti
C. Candida Spp
D. Trypanosoma Cruzi
E. Helicobacter Pylori
Answer: D
Infection with Trypanosoma Cruzi may result in destruction of the ganglion cells of the myenteric plexus, resulting in a clinical
picture similar to achalasia.

Trypanosoma Cruzi: Protozoan. Causes Chagas disease. Carried by bugs which infect the skin whilst feeding. Penetrate through
open wounds and mucous membranes. Intracellular proliferation. Major infective sites include CNS, intestinal myenteric plexus,
spleen, lymph nodes and cardiac muscle. Chronic disease is irreversible, nifurtimox is used to treat acute infection

85. A 45-year-old man presents to surgical outpatients with a long history of recurrent abdominal pain and vomiting. He is
noted to have a peripheral motor neuropathy on examination. What is the most likely diagnosis?
A. Huntington's disease
B. Myeloma
C. Acute intermittent porphyria
D. Lawrence-Moon-Biedl syndrome
E. Friedreich's ataxia
Answer: C
Neurological signs combined with abdominal pain is acute intermittent porphyria or lead poisoning until proven otherwise.

Acute intermittent porphyria: Acute intermittent porphyria (AIP) is a rare autosomal dominant condition caused by a defect in
porphobilinogen deaminase, an enzyme involved in the biosynthesis of haem. The results in the toxic accumulation of delta
aminolaevulinic acid and porphobilinogen. It characteristically presents with abdominal and neuropsychiatric symptoms in 20-40
year olds. AIP is more common in females (5:1). Features: Abdominal: abdominal pain, vomiting. Neurological: motor neuropathy.
Psychiatric: e.g. Depression. Hypertension and tachycardia common. Diagnosis: Classically urine turns deep red on standing.
Raised urinary porphobilinogen (elevated between attacks and to a greater extent during acute attacks). Assay of red cells for
porphobilinogen deaminase. Raised serum levels of delta aminolaevulinic acid and porphobilinogen

86. A 56 year old man presents with episodic facial pain and discomfort whilst eating. He has suffered from halitosis recently
and he frequently complains of a dry mouth. He has a smooth swelling underneath his right mandible. What is the most
likely underlying diagnosis?
A. Stone impacted in Whartons duct
B. Stone impacted in Stensens duct
C. Benign adenoma of the submandibular gland
D. Adenocarcinoma of the submandibular gland
E. Squamous cell carcinoma of the submandibular gland
Answer: A
The symptoms are typical for sialolithiasis. The stones most commonly form in the submandibular gland and therefore may occlude
Whartons duct. Stensens duct drains the parotid gland.

Submandibular glands- disease: Physiology: The submandibular glands secrete approximately 800- 1000ml saliva per day. They
typically produce mixed seromucinous secretions. When paraympathetic activity is dominant the secretions will be more serous.
The parasympathetic fibres are derived from the chorda tympani nerves and the submandibular ganglion, they travel to the glands
via the lingual nerves.

Sialolithiasis: 80% of all salivary gland calculi occur in the submandibular gland. 70% of the these calculi are radio-opaque. Stones
are usually composed of calcium phosphate or calcium carbonate. Patients typically develop colicky pain and post prandial swelling
of the gland. Investigation involves sialography to demonstrate the site of obstruction and associated other stones. Stones impacted
in the distal aspect of Whartons duct may be removed orally, other stones and chronic inflammation will usually require gland
excision.

33
Sialadenitis: Usually occurs as a result of Staphylococcus aureus infection. Pus may be seen leaking from the duct, erythema may
also be noted. Development of a sub mandibular abscess is a serious complication as it may spread through the other deep fascial
spaces and occlude the airway

Submandibular tumours: Only 8% of salivary gland tumours affect the sub mandibular gland. Of these 50% are malignant
(usually adenoid cystic carcinoma). Diagnosis usually involves fine needle aspiration cytology. Imaging is with CT and MRI. In
view of the high prevalence of malignancy, all masses of the submandibular glands should generally be excised.

87. Which of the following cellular types or features is not seen in sarcoidosis?
A. Reed Sternberg Cells
B. T lymphocytes
C. Macrophages
D. Asteroid bodies
E. B lymphocytes
Answer: A
Reed Sternberg cells are seen in Hodgkins disease. All of the other cell types are seen in sarcoid.

88. Which of the following diseases is not considered a risk factor for gastric cancer?
A. Polya gastrectomy for antral ulcer
B. Atrophic gastritis
C. Intestinal metaplasia of columnar type at the gastric cardia
D. Patient with polyp showing medium grade dysplasia
E. Long term therapy with H2 blockers
Answer: E
Although some acid lowering procedures increase the risk of gastric cancer the use of H 2 blockers does not, at the present time,
seem to increase the risk.

89. A 56 year old man is diagnosed as having a glioma. From which of the following cell types do these tumours usually
originate?
A. Astrocytes
B. Oligodendrocytes
C. Ependymal cells
D. Squamous cells
E. Neuroglial cells
Answer: E
Gliomas originate from glial (otherwise known as neuroglial) cells. These serve a structural function in the CNS. The tumours
produced may resemble a number of CNS cell types. Tumours are therefore named according to the cells they resemble rather than
the origin. Where this is not possible they are termed gliomas.

Glioma: a tumour that is typically found in the CNS. These tumours arise from glial cells. They are sub categorised according to the
cell type they most closely resemble.

Glioma sub types:Ependymomas- Ependymal cells. Astocytomas- Astrocytes (including glioblastoma). Oligodendrogliomas-
Oligodendrocytes. Mixed- e.g. oligoastrocytomas. Gliomas are categorised as being either high or low grade lesions (the former has
the worse prognosis). They may be either supra or infra tentorial. Their symptoms will typically reflect their site of origin.
Glioblastoma multiforme has the worst prognosis and few patients will survive beyond 12 months.

90. A 78 year old man presents with unilateral deafness which has been present for the past 3 months. On examination Webers
test localises to the contralateral side and a CT scan of his head shows a thickened calvarium with areas of sclerosis and
radiolucency. His blood tests show an elevated alkaline phosphatase, normal serum calcium and normal PTH levels. Which
of the following is the most likely underlying diagnosis?
A. Multiple myeloma with skull involvement
B. Osteoporosis
C. Pagets disease with skull involvement
D. Lung cancer with skull metastasis
E. Osteopetrosis with skull involvement
Answer: C

34
Of the conditions listed Pagets disease is the most likely diagnosis (skull vault expansion and sensorineural hearing loss). Multiple
myeloma would typically result in multiple areas of radiolucency and usually raised calcium in this setting. Osteopetrosis is a
recognised cause of the features described. However, it is a rare inherited disorder and usually presents in children in young adults.
Presentation at this stage with no prior symptoms would be extremely rare and therefore this is not the most likely diagnosis.

Pagets disease: a disease of increased but uncontrolled bone turnover and is characterised by architecturally abnormal bones. It is
thought to be primarily a disorder of osteoclasts, with excessive osteoclastic resorption followed by increased osteoblastic activity
causing areas of sclerosis and deformity. Paget's disease is common (UK prevalence 5%) but symptomatic in only 1 in 20 patients
Predisposing factors: increasing age. Male sex. Northern latitude. Family history. Clinical features: bone pain (e.g. pelvis, lumbar
spine, femur). Classical, untreated features: bowing of tibia, bossing of skull. Raised alkaline phosphatase (alp) - calcium* and
phosphate are typically normal. Skull x-ray: thickened vault, osteoporosis circumscripta. Indications for treatment: bone pain,
skull or long bone deformity, fracture, periarticular Paget's. bisphosphonate (either oral risedronate or IV zoledronate). Calcitonin is
less commonly used now. Complications: deafness (cranial nerve entrapment). Bone sarcoma (1% if affected for > 10 years).
Fractures. Skull thickening. High-output cardiac failure.
*usually normal in this condition but hypercalcaemia may occur with prolonged immobilisation

Theme: Genetic causes of cancer

A. Multiple endocrine neoplasia type I


B. Multiple endocrine neoplasia type II
C. Gardner's syndrome
D. Lynch Syndrome
E. Kartagener's syndrome
F. Von Recklinghausen's disease

Please select the most likely condition for the disease process described. Each option may be used once, more than once or not at all

91. A 5 year old boy presents with recurrent episodes of sinusitis. The casualty staff are surprised to find his liver lying in the
left upper quadrant of the abdomen
Answer: Kartagener's syndrome
This is a case of Kartagener's syndrome. The primary problem is of immotile cilia syndrome. When associated with situs inversus
Kartagener's syndrome is diagnosed.

92. A 22 year old man presents with carcinoma of the caecum. His brother died from colorectal cancer aged 25 and his father
died from the disease aged 30.
Answer: Lynch Syndrome
This is a case of Lynch syndrome HNPCC. It is transmitted in an autosomal dominant fashion.

93. A tall 32 year old lady presents with a diffuse neck swelling a carcinoma of the thyroid medullary type is diagnosed.
Answer: Multiple endocrine neoplasia type II
This is a case MEN type IIb. It is associated with phaeochromocytomas and is transmitted in an autosomal dominant pattern if
inherited. All MEN II tend to have medullary carcinoma of the thyroid as a presenting feature

94. A 45 year old man presents with symptoms of urinary colic. In the history he has suffered from recurrent episodes of frank
haematuria over the past week or so. On examination he has a left loin mass and a varicocele. The most likely diagnosis is:
A. Renal adenocarcinoma
B. Renal cortical adenoma
C. Squamous cell carcinoma of the renal pelvis
D. Retroperitoneal fibrosis
E. Nephroblastoma
Answer: A
Renal adenocarcinoma are the most common renal malignancy and account for 75% cases.
Patients may develop frank haematuria and have episodes of clot colic.A Grawitz tumour is an eponymous name for Renal
Adenocarcinoma. May metastasise to bone.

Renal cell carcinoma


Renal cell carcinoma is an adenocarcinoma of the renal cortex and is believed to arise from the proximal convoluted tubule. They are
usually solid lesions, up to 20% may be multifocal, 20% may be calcified and 20% may have either a cystic component or be wholly
35
cystic. They are often circumscribed by a pseudocapsule of compressed normal renal tissue. Spread may occur either by direct
extension into the adrenal gland, renal vein or surrounding fascia. More distant disease usually occurs via the haematogenous route to
lung, bone or brain. Renal cell carcinoma comprise up to 85% of all renal malignancies. Males are more commonly affected than
females and sporadic tumours typically affect patients in their sixth decade. Patients may present with a variety of symptoms
including; haematuria (50%), loin pain (40%), mass (30%) and up to 25% may have symptoms of metastasis.Less than 10% have the
classic triad of haematuria, pain and mass.

Investigation Many cases will present as haematuria and be discovered during diagnostic work up. Benign renal tumours are rare, so
renal masses should be investigated with multislice CT scanning. Some units will add and arterial and venous phase to the scan to
demonstrate vascularity and evidence of caval ingrowth. CT scanning of the chest and abdomen to detect distant disease should also
be undertaken. Routine bone scanning is not indicated in the absence of symptoms. Biopsy should not be performed when a
nephrectomy is planned but is mandatory before any ablative therapies are undertaken. Assessment of the functioning of the contra
lateral kidney.

Management: T1 lesions may be managed by partial nephrectomy and this gives equivalent oncological results to total radical
nephrectomy. Partial nephrectomy may also be performed when there is inadequate reserve in the remaining kidney. For T2 lesions
and above a radical nephrectomy is standard practice and this may be performed via a laparoscopic or open approach. Preoperative
embolisation is not indicated nor is resection of uninvolved adrenal glands. During surgery early venous control is mandatory to avoid
shedding of tumour cells into the circulation. Patients with completely resected disease do not benefit from adjuvant therapy with
either chemotherapy or biological agents. These should not be administered outside the setting of clinical trials. Patients with
transitional cell cancer will require a nephroureterectomy with disconnection of the ureter at the bladder.

95. A 63 year old man finds that he has to stop walking after 100 yards due to bilateral calf pain. He finds that bending forwards
and walking up hill helps. He is able to ride a bike without any pain. What is the most likely underlying cause?
A. Lumbar canal stenosis
B. Diabetic neuropathy
C. Aorto-iliac occlusion
D. Occlusion of the superficial femoral artery
E. Pelvic rheumatoid arthritis
Answer: A
The positional nature of the pain and the fact that improves with walking uphill makes an underlying vascular aetiology far less
likely.

Lumbar spinal stenosis


Lumbar spinal stenosis is a condition in which the central canal is narrowed by tumour, disk prolapse or other similar degenerative
changes. Patients may present with a combination of back pain, neuropathic pain and symptoms mimicking claudication. One of the
main features that may help to differentiate it from true claudication in the history is the positional element to the pain. Sitting is
better than standing and patients may find it easier to walk uphill rather than downhill. The neurogenic claudication type history
makes lumbar spinal stenosis a likely underlying diagnosis, the absence of such symptoms makes it far less likely.
Pathology: Degenerative disease is the commonest underlying cause. Degeneration is believed to begin in the intervertebral disk
where biochemical changes such as cell death and loss of proteoglycan and water content lead to progressive disk bulging and
collapse. This process leads to an increased stress transfer to the posterior facet joints, which accelerates cartilaginous degeneration,
hypertrophy, and osteophyte formation; this is associated with thickening and distortion of the ligamentum flavum. The combination
of the ventral disk bulging, osteophyte formation at the dorsal facet, and ligamentum flavum hyptertrophy combine to
circumferentially narrow the spinal canal and the space available for the neural elements. The compression of the nerve roots of the
cauda equina leads to the characteristic clinical signs and symptoms of lumbar spinal stenosis.
Diagnosis: MRI scanning is the best modality for demonstrating the canal narrowing. Historically a bicycle test was used as true
vascular claudicants could not complete the test.Treatment: Laminectomy

96. A 73 year old lady is admitted for a laparoscopic cholecystectomy. During her pre-operative assessment it is noted that she is
receiving furosemide for the treatment of hypertension. Approximately what proportion of the sodium that is filtered at the
glomerulus will be subsequently excreted?
A. Up to 25%
B. Upt to 75%
C. Between 3 and 5%
D. <2%
E. Between 1 and 2%
Answer: A

36
The loop diuretics can lead to marked increases in the amount of sodium excreted. They act in the medullary and cortical aspects of
the thick ascending limb of the loop of Henle. This results in a decreased medullary osmolal gradient and increases free water
excretion (as well as loss of sodium). Because loop diuretics result in the loss of both sodium and water they are less frequently
associated with hyponatraemia than thiazide diuretics (these latter agents act in the cortex and do not affect urine concentrating
ability).

Diuretic agents: The diuretic drugs are divided into three major classes, which are distinguished according to the site at which they
impair sodium reabsorption: loop diuretics in the thick ascending loop of Henle, thiazide type diuretics in the distal tubule and
connecting segment; and potassium sparing diuretics in the aldosterone - sensitive principal cells in the cortical collecting tubule.
In the kidney, sodium is reabsorbed through Na +/ K+ ATPase pumps located on the basolateral membrane. These pumps return
reabsorbed sodium to the circulation and maintain low intracellular sodium levels. This latter effect ensures a constant concentration
gradient.

Physiological effects of commonly used diuretics


Site of action Diuretic Carrier or channel Percentage of filtered sodium
inhibited excreted
Ascending limb of loop of Henle Frusemide Na+/K+ 2Cl - carrier Upt to 25%
Distal tubule and connecting Thiazides Na+Cl- carrier Between 3 and 5%
segment
Cortical collecting tubule Spironolactone Na+ channel Between 1 and 2%

97. A 59 year old man presents with recurrent episodes of urinary sepsis. In his history he mentions that he has suffered from
recurrent attacks of left iliac fossa pain over the past few months. He has also notices bubbles in his urine. He undergoes a
CT scan which shows a large inflammatory mass in the left iliac fossa. No other abnormality is detected. The most likely
diagnosis is:
A. Ulcerative colitis
B. Crohns disease
C. Mesenteric ischaemia
D. Diverticular disease
E. Rectal cancer
Answer: D
Recurrent attacks of diverticulitis may cause the development of local abscesses which may erode into the bladder resulting in
urinary sepsis and pneumaturia. This would be an unusual presentation from Crohns disease and rectal cancer would be more
distally sited and generally evidence of extra colonic disease would be present if the case were malignant and this advanced.

Diverticular disease: Diverticular disease is a common surgical problem. It consists of herniation of colonic mucosa through the
muscular wall of the colon. The usual site is between the taenia coli which vessels pierce the muscle to supply the mucosa.
Symptoms: Altered bowel habit Bleeding. Abdominal pain. Complications: Diverticulitis. Haemorrhage. Development of fistula
Perforation and faecal peritonitis. Perforation and development of abscess. Development of diverticular phlegmon
Diagnosis: Patients presenting in clinic will typically undergo either a colonoscopy or barium enema as part of their diagnostic work
up. Both tests will identify diverticular disease. It can be far more difficult to confidently exclude cancer, particularly in diverticular
strictures. Acutely unwell surgical patients should be investigated in a systematic way. Plain abdominal films and an erect chest x-
ray will identify perforation. An abdominal CT scan with oral and intravenous contrast will help to identify whether acute
inflammation is present but also the presence of local complications such as abscess formation.
Severity Classification- Hinchey: I: Para-colonic abscess. II: Pelvic abscess. III: Purulent peritonitis. IV: Faecal peritonitis
Treatment: Increase dietary fibre intake. Mild attacks of diverticulitis may be managed conservatively with antibiotics.. Peri
colonic abscesses should be drained either surgically or radiologically. Recurrent episodes of acute diverticulitis requiring
hospitalisation are a relative indication for a segmental resection. Hinchey IV perforations (generalised faecal peritonitis) will
require a resection and usually a stoma. This group have a very high risk of post operative complications and usually require HDU
admission. Less severe perforations may be managed by laparoscopic washout and drain insertion.

98. A 78 year old man is referred to the clinic by his general practitioner. For many years he noticed a smooth swelling
approximately 5cm anterior to the tragus of his right ear. Apart from being a heavy smoker he has no co-morbidities. What
is the most likely diagnosis?
A. Pleomorphic adenoma
B. Liposarcoma
C. Warthins tumour
D. Adenocarcinoma

37
E. None of the above
Answer: C
Warthins tumours are most common in elderly smokers. They have a relatively benign and indolent course. They are usually well
circumscribed as illustrated below:

Parotid gland clinical: Causes of bilateral parotid enlargement : Mumps: Associated with meningoencephalitis, pancreatitis,
orchitis, or deafness. Parotitis. Sialectasis - especially if related to eating. Sjogren's syndrome: dry eyes or mouth, connective tissue
disease. Sarcoidosis. Tuberculosis. Alcoholism. Myxoedema. Cushing's disease. Diabetes/insulin resistance. Liver cirrhosis. Gout.
Bulimia nervosa. Drugs. Severe dehydration. Malnutrition

Causes of unilateral parotid enlargement: Salivary calculus. Tumour

Parotid gland tumours: Pleomorphic adenomas are the most common. Incisional biopsy of parotid masses is not recommended, so
superficial parotidectomy is the usual procedure of choice. Signs of facial nerve palsy and a parotid mass should raise suspicion of
malignancy. Warthins tumours are relatively benign lesions that are slow growing and occur most commonly in elderly male
smokers.. Adenoid cystic carcinoma have a tendency for perineural invasion.

Theme: Lung cancer

A. Adenocarcinoma
B. Small cell lung cancer
C. Large cell lung cancer
D. Squamous cell carcinoma

Please select the most likely lung cancer variant for the scenario described. Each option may be used once, more than once or not at
all.

99. A 73 year old heavy smoker presents with haemoptysis. On examination he is cachectic and shows evidence of clubbing.
Imaging shows a main bronchial tumour with massive mediastinal lymphadenopathy together with widespread visceral
metastases.
Answer: Small cell lung cancer
Small cell carcinoma is associated with disseminated disease at presentation in the majority of cases. Most cases occur in the main
airways and paraneoplastic features are common.

100. A 68 year old female who has never smoked presents with a mass at the periphery of her right lung.
Answer: Adenocarcinoma
Adenocarcinomas are the most common tumour type present in never smokers. They are usually located at the periphery.

101. An 85 year old man presents with a cough and haemoptysis. He has a modest smoking history of 15 pack years. He is found
to have a tumour located in the right main bronchus, with no evidence of metastatic disease. He decides no undergo any
treatment and he remains well for a further 12 months before developing symptomatic metastasis.
Answer: Squamous cell carcinoma
Squamous cell carcinomas are reported to be more slow growing and are typically centrally located. Small cell carcinomas are
usually centrally located. However, small cell carcinomas would seldom be associated with a survival of a year without treatment.

Lung cancers may be classified according to histological subtypes. The main distinction is between small cell and non small cell
lung cancer. Non small cell lung cancer is the most common variant and accounts for 80% of all lung cancers.
Non small cell lung cancer: These share common features of prognosis and management. They comprise the following tumours:
Squamous cell carcinoma (25% cases). Adenocarcinoma (40% cases). Large cell carcinoma (10% cases)
Paraneoplastic features and early disease dissemination are less likely than with small cell lung carcinoma. Adenocarcinoma is the
most common lung cancer type encountered in never smokers.

Small cell lung carcinoma: Small cell lung carcinomas are comprised of cells with a neuro endocrine differentiation. The
neuroendocrine hormones may be released from these cells with a wide range of paraneoplastic associations. These tumours are
strongly associated with smoking and will typically arise in the larger airways. They disseminate early in the course of the disease
and although they are usually chemosensitive this seldom results in long lasting remissions.
38
102. Which of the following cells is not found on a blood film post splenectomy?
A. Pappenheimer bodies
B. Stipple cells
C. Erythrocyte containing siderotic granules
D. Howell-Jolly bodies
E. Target cells
Answer: B

Stipple cells are found in lead poisoning/haemoglobinopathies.


Blood film in hyposplenism:
Howell-Jolly bodies. Pappenheimer bodies. Poikilocytes (Target cells). Erythrocyte containing siderotic granules. Heinz bodies

103. A 45 year old man with long standing ulcerative colitis and rectal dysplasia presents with a DALM lesion in the rectum.
What is the most appropriate management option?
A. Snare polypectomy
B. Repeat endoscopy in 2 years
C. Discharge
D. Anterior resection
E. Panproctocolectomy
Answer: E
DALM lesions complicating ulcerative colitis should be managed with panproctocolectomy. An anterior resection is inadequate
since it will only remove the rectum and ulcerative colitis affects the entire colon. Since many will be associated with invasion a
snare polypectomy is not sufficient either.

Colonic lesions – DALM: The term DALM lesion refers to a Dysplasia Associated Lesion or Mass. They may complicate dysplasia
occurring in patients with longstanding ulcerative colitis. They have a high incidence of invasive foci.When they complicate
longstanding ulcerative colitis, they should be treated by panproctocolectomy.

104. Which of the metastatic bone tumours described below is at the greatest risk of pathological fracture ?
A. Proximal humeral lesion from a prostate cancer
B. Vertebral body lesions from a prostate cancer
C. Peritrochanteric lesion from a carcinoma of the breast
D. Proximal humeral lesion from a carcinoma of the breast
E. Peritrochanteric lesion from a prostate cancer
Ansewr: C
Peritrochanteric lesions have the greatest risks of fracture (due to loading). The lesions from breast cancer are usually lytic and
therefore at higher risk rather than the sclerotic lesions from prostate cancer.

Metastatic bone disease- risk of fracture: Metastatic bone tumours may be described as blastic, lytic or mixed. Osteoblastic
metastatic disease has the lowest risk of spontaneous fracture when compared to osteolytic lesions of a similar size.
Lesions affecting the peritrochanteric region are most prone to spontaneous fracture (because of loading forces at that site).
The factors are incorportated into the Mirel Scoring system to stratify the risk of spontaneous fracture for bone metastasis of varying
types.

Mirel Scoring system


Score points Site Radiographic appearance Width of bone involved Pain
1 Upper extremity Blastic Less than 1/3 Mild
2 Lower extremity Mixed 1/3 to 2/3 Moderate
3 Peritrochanteric Lytic More than 2/3 Aggravated by function

Depending upon the score the treatment should be as follows:

Score Risk of fracture Treatment


9 or greater Impending (33%) Prophylactic fixation
8 Borderline Consider fixation
7 or less Not impending (4%) Non operative management

39
105. A 63 year old male presents with several episodes of haematuria. He suffers from COPD secondary to long term smoking.
Which is the most likely underlying cause?
A. Renal cortical adenoma
B. Renal adenocarcinoma
C. Nephroblastoma
D. Transitional cell carcinoma of the bladder
E. Adenocarcinoma of the bladder
Answer: D

TCC is the most common subtype and is strongly linked to smoking. The important point to note in this question is the term most
likely as renal adenocarcinoma may produce similar symptoms but is less likely.

Bladder cancer is the second most common urological cancer. It most commonly affects males aged between 50 and 80 years of
age. Those who are current, or previous (within 20 years), smokers have a 2-5 fold increased risk of the disease. Occupational
exposure to hydrocarbons such as alanine increases the risk. Although rare in the UK, chronic bladder inflammation arising from
Schistosomiasis infection remains a common cause of squamous cell carcinomas, in those countries where the disease is endemic.
Benign tumours of the bladder including inverted urothelial papilloma and nephrogenic adenoma are uncommon.

Bladder malignancies: Transitional cell carcinoma (>90% of cases). Squamous cell carcinoma ( 1-7% -except in regions affected
by schistosomiasis). Adenocarcinoma (2%). Transitional cell carcinomas may arise as solitary lesions, or may be multifocal, owing
to the effect of "field change" within the urothelium. Up to 70% of TCC's will have a papillary growth pattern. These tumours are
usually superficial in location and accordingly have a better prognosis. The remaining tumours show either mixed papillary and
solid growth or pure solid growths. These tumours are typically more prone to local invasion and may be of higher grade, the
prognosis is therefore worse. Those with T3 disease or worse have a 30% (or higher) risk of regional or distant lymph node
metastasis.

Staging Stage Description


Most will undergo a cystoscopy and biopsies or TURBT, this T0 No evidence of tumour
provides histological diagnosis and information relating to depth of Ta Non invasive papillary carcinoma
invasion. Locoregional spread is best determined using pelvic MRI T1 Tumour invades sub epithelial connective tissue
and distant disease CT scanning. Nodes of uncertain significance T2a Inner half of detrusor invaded
may be investigated using PET CT. T2b Outer half of detrusor invaded
T3 Tumour extends to perivesical fat
Presentation T4a Invasion of uterus, prostate or bowel
Most patients (85%) will present with painless, macroscopic T4b Invasion of other abdominal organs
haematuria. In those patients with incidental microscopic
N0 No nodal disease
haematuria, up to 10% of females aged over 50 will be found to
N1 Single lymph node metastasis (up to 2cm)
have a malignancy (once infection excluded).
N2 Single node >2cm or multiple nodes up to 5cm
N3 Nodes over 5cm
Treatment
Those with superficial lesions may be managed using TURBT in M1 Distant disease
isolation. Those with recurrences or higher grade/ risk on histology may be offered intravesical chemotherapy. Those with T2
disease are usually offered either surgery (radical cystectomy and ileal conduit) or radical radiotherapy.
Prognosis: T1: 90%. T2: 60%. T3: 35%. T4a: 10-25%. Any T, N1-N2: 30%

Theme: Neck lumps

A. Cystic hygroma
B. Bartonella infection
C. Mycobacterium tuberculosis infection
D. Branchial cyst
E. Thyroglossal cyst
F. Pharyngeal pouch
G. Follicular thyroid cyst
H. Parathyroid adenoma
I. None of the above

Please select the most likely underlying disease process for the scenario given. Each option may be used once, more than once or
not at all.
40
106. A 25 year old cat lover presents with symptoms of abdominal pain, lethargy and sweats. These have been present for the
past two weeks. On examination she has lymphadenopathy in the posterior triangle.
Answer: Bartonella infection
Bartonella infection may occur following a cat scratch. The organism is intracellular. Generalised systemic symptoms may occur for
a week or so prior to clinical presentation.

107. A 25 year old lady presents with an swelling located at the anterior border of the sternocleidomastoid muscle. The swelling is
intermittent and on examination it is soft and fluctuant.
Answer: Branchial cyst
Branchial cysts are remnants of the branchial cleft. They may become infected.

108. A 38 year old lady presents with a mass in the midline of the neck immediately below the hyoid bone. It moves upwards on
tongue protrusion.
Answer: Thyroglossal cyst
Thyroglossal cysts are usually located in the midline and are linked to the foramen caecum and will thus move upwards on tongue
protrusion.

109. A 22 year old man presents with a discharging area on his lower back. On examination there is an epithelial defect located
6cm proximal to the tip of his coccyx and located in the midline. There are two further defects located about 2cm superiorly
in the same position. He is extremely hirsute. What is the most likely diagnosis?
A. Pre sacral tumour
B. Sacrococcygeal teratoma
C. Pilonidal sinus
D. Fistula in ano
E. Occult spina bifida
Answer: C
Pilonidal sinuses are extremely common in hirsute individuals and typically present as midline sinuses in the natal cleft.

Pilonidal sinus: Occur as a result of hair debris creating sinuses in the skin (Bascom theory). Usually in the natal cleft of male
patients after puberty. It is more common in Caucasians related to their hair type and growth patterns. The opening of the sinus is
lined by squamous epithelium, but most of its wall consists of granulation tissue. Up to 50 cases of squamous cell carcinoma has
been described in patients with chronic pilonidal sinus disease. Hairs become trapped within the sinus. Clinically the sinus presents
when acute inflammation occurs, leading to an abscess. Patients may describe cycles of being asymptomatic and periods of pain and
discharge from the sinus. Treatment is difficult and opinions differ. Definitive treatment should never be undertaken when acute
infection or abscess is present as this will result in failure. Definitive treatments include the Bascom procedure with excision of the
pits and obliteration of the underlying cavity. The Karydakis procedure involves wide excision of the natal cleft such that the surface
is recontoured once the wound is closed. This avoids the shearing forces that break off the hairs and has reasonable results.

110. A 43 year old man from Greece presents with colicky right upper quadrant pain, jaundice and a pruritic, erythematous rash
over his body. He is initially treated with ciprofloxacin, but does not improve. What is the most likely diagnosis?
A. Infection with Wucheria bancrofti
B. Infection with Echinococcus granulosus
C. Type III hypersentivity reaction
D. Allergy to ciprofloxacin
E. Common bile duct stones
Answer: B
Infection with Echinococcus granulosus will typically produce a type I hypersensitivity reaction which is characterised by a
urticarial rash. With biliary rupture a classical triad of biliary colic, jaundice and urticaria occurs. Whilst jaundice and biliary colic
may be a feature of CBD stones they do not produce an urticarial rash. Antibiotic sensitivity with ciprofloxacin may produce
jaundice and a rash, however it was not present at the outset and does not cause biliary colic.

Hydatid cysts: Hydatid cysts are endemic in Mediterranean and Middle Eastern countries. They are caused by the tapeworm
parasite Echinococcus granulosus. An outer fibrous capsule is formed containing multiple small daughter cysts. These cysts are
allergens which precipitate a type 1 hypersensitivity reaction.

Clinical features are as follows: Up to 90% cysts occur in the liver and lungs. Can be asymtomatic, or symptomatic if cysts > 5cm in
41
diameter. Morbidity caused by cyst bursting, infection and organ dysfunction (biliary, bronchial, renal and cerebrospinal fluid
outflow obstruction). In biliary ruputure there may be the classical triad of; biliary colic, jaundice, and urticarial. CT is the best
investigation to differentiate hydatid cysts from amoebic and pyogenic cysts. Surgery is the mainstay of treatment (the cyst walls
must not be ruptured during removal and the contents sterilised first).

111. A 22 year old lady presents with an episode of renal colic and following investigation is suspected of suffering from MEN IIa.
Which of the following abnormalities of the parathyroid glands are most often found in this condition?
A. Hypertrophy
B. Hyperplasia
C. Adenoma
D. Carcinoma
E. Metaplasia
Answer: B
MEN IIa: Medullary thyroid cancer. Hyperparathyroidism (usually hyperplasia). Phaeochromocytoma
In MEN IIa the commonest lesion is medullary thyroid cancer, with regards to the parathyroid glands the most common lesion is
hyperplasia. In MEN I a parathyroid adenoma is the most common lesion.

Multiple Endocrine NeoplasiaMultiple endocrine neoplasia (MEN) is inherited as an autosomal dominant disorder.
The table below summarises the three main types of MEN:

MEN type I: Mnemonic 'three P's': Parathyroid (95%): Parathyroid adenoma. Pituitary (70%): Prolactinoma/ACTH/Growth
Hormone secreting adenoma. Pancreas (50%): Islet cell tumours/Zollinger Ellison syndrome. Also: Adrenal (adenoma) and thyroid
(adenoma). MENIN gene (chromosome 11). Most common presentation = hypercalcaemia.

MEN type IIa: Phaeochromocytoma. Medullary thyroid cancer (70%). Hyperparathyroidism (60%). RET oncogene (chromosome
10).

MEN type IIb: Same as MEN IIa with addition of: Marfanoid body habitus. Mucosal neuromas. RET oncogene (chromosome 10)

115. A male infant is born prematurely at 34 weeks gestation by emergency cesarean section. He initially appears to be
stable. However, over the ensuing 24 hours he develops worsening neurological function. Which of the following processes is
most likely to have occurred?

A. Extra dural haemorrhage


B. Sub dural haemorrhage
C. Sub arachnoid haemorrhage
D. Intraventricular haemorrhage
E. Arteriovenous malformation
Answer: D
Intraventricular haemorrhage
Intraventricular haemorrhage is a haemorrhage that occurs into the ventricular system of the brain. It is relatively rare in adult
surgical practice and when it does occur, it is typically associated with severe head injuries. In premature neonates it may occur
spontaneously. The blood may clot and occlude CSF flow, hydrocephalus may result.
In neonatal practice the vast majority of IVH occur in the first 72 hours after birth, the aetiology is not well understood and it is
suggested to occur as a result of birth trauma combined with cellular hypoxia, together the with the delicate neonatal CNS.
TreatmentIs largely supportive, therapies such as intraventricular thrombolysis and prophylactic CSF drainage have been trialled
and not demonstrated to show benefit. Hydrocephalus and rising ICP is an indication for shunting.

112. A 22 year old man is admitted to hospital with a lower respiratory chest infection. He had a splenectomy after being
involved in a car accident. What is the most likely infective organism?
A. Haemophilus influenzae
B. Staphylococcus aureus
C. Rhinovirus
D. Mycobacterium tuberculosis
E. Moraxella catarrhalis
Answer: A
Organisms causing post splenectomy sepsis: Streptococcus pneumonia. Haemophilus influenza. Meningococci

42
Encapsulated organisms carry the greatest pathogenic risk following splenectomy. The effects of sepsis following splenectomy are
variable. This may be the result of small isolated fragments of splenic tissue that retain some function following splenectomy. These
may implant spontaneously following splenic rupture (in trauma) or be surgically implanted at the time of splenectomy.

Post splenectomy sepsis: The loss of splenic function renders individuals at increased risk of fulminant sepsis. Young children are
at the highest risk, especially in the first 2 years following surgery. Surgery for trauma is associated with a lower risk than when
splenectomy is performed as a treatment for haematological disorders.
Infection with encapsulated organisms poses the greatest risk, these organisms may be opsonised, but this then goes undetected at an
immunological level due to loss of the spleen. Prophylactic vaccinations are usually administered to reduce the risk of
pneumococcal septicaemia. Since the vaccine only covers up to 80% of pneumococcal infections, patients will usually recieve long
term, low dose penicillin prophylaxis in addition to vaccination.

113. A 24 year old man presents with symptoms of malaise, weight loss and lymphadenopathy. A lymph node biopsy is performed
and the subsequent histology report states that there is evidence of granuloma formation and central necrosis. What is the
most likely underlying cause?
A. Non Hodgkins lymphoma
B. Churg Strauss syndrome
C. Epstein Barr Virus infection
D. Rheumatoid nodule
E. Infection with Mycobacterium tuberculosis
Answer: E
These histological features are typically seen in TB. Necrosis occurring in granulomas is usually indicative of an underlying
infective cause. Churg Strauss syndrome is a form of vasculitis, which is the usual histological finding. Granulomas are reported in
the condition, but it is rare for them to demonstrate necrosis.

114. A 20 year old man develops acute appendicitis, his appendix is removed and he makes a full recovery. Which of the
following pathological processes is least likely to be present in the acutely inflamed tissues?
A. Altered Starlings forces.
B. Seqestration of neurophils
C. Formation of fluid exudate
D. Formation of granulomas
E. None of the above
Answer: D
Neutrophil polymorphs=Acute inflammation. Granuloma = Chronic inflammation.

Acute inflammation: 3 phases: 1. Changes in blood vessel and flow: flush, flare, wheal. 2. Fluid exudates (rich in protein i.e. Ig,
coagulation factors) produced via increased vascular permeability. 3. Cellular exudates mainly containing neutrophil polymorphs
pass into extravascular space.

Neutrophils are then transported to tissues via: a. Margination of neutrophils to the peripheral plasmatic of the vessel rather than
the central axial stream. b. Pavementing: Adhesion of neutrophils to endothelial cells in venules at site of acute inflammation. c.
Emigration: neutrophils pass between endothelial cells into the tissue

Theme: Liver lesions

A. Haemangioma
B. Hepatocellular carcinoma
C. Hepatic metastasis
D. Polycystic liver disease
E. Simple liver cyst
F. Hyatid cyst
G. Amoebic abscess
H. Mesenchymal hamartoma

Please select the most likely liver lesion for the scenario given. Each option may be used once, more than once or not at all.

115. A 42 year old lady has suffered from hepatitis C for many years and has also developed cirrhosis. On routine follow up, an
ultrasound has demonstrated a 2.5cm lesion in the right lobe of the liver.
43
Answer: Hepatocellular carcinoma
In patients with cirrhosis the presence of a lesion >2cm is highly suggestive of malignancy. The diagnosis is virtually confirmed if
the AFP is >400ng/mL.

116. A 25 year old man from the far east presents with a fever and right upper quadrant pain. As part of his investigations a CT
scan shows an ill defined lesion in the right lobe of the liver.
Answer: Amoebic abscess
Amoebic abscesses will tend to present in a similar fashion to other pyogenic liver abscesses. They should be considered in any
individual presenting from a region where Entamoeba histiolytica is endemic. Treatment with metronidazole usually produces a
marked clinical response.
117. A 42 year old lady presents with right upper quadrant pain and a sensation of abdominal fullness. An ultrasound scan
demonstrates a 6.5 cm hyperechoic lesion in the right lobe of the liver. Serum AFP is normal.
Answer: Haemangioma
A large hyperechoic lesion in the presence of normal AFP is likely to be a haemangioma. An HCC of equivalent size will almost
always result in rise in AFP.

118. Which of the following disorders is associated with massive splenomegaly?


A. Acute lymphoblastic leukaemia
B. Acute myeloblastic leukaemia
C. Acute myelomonocytic leukaemia
D. Acute monoblastic leukaemia
E. Chronic granulocytic leukaemia
Answer: E
Chronic leukaemia is more likely to be associated with splenomegaly than acute leukaemia.

Spleen: Embryology: derived from mesenchymal tissue. Shape: orange segment. Position: below 9th-12th ribs. Weight: 75-150g
Relations: Superiorly- diaphragm. Anteriorly- gastric impression. Posteriorly- kidney. Inferiorly- colon. Hilum: tail of pancreas and
splenic vessels (splenic artery divides here, branches pass to the white pulp transporting plasma). Forms apex of lesser sac
(containing short gastric vessels). Contents: White pulp: immune function. Contains central trabecular artery. The germinal centres
are supplied by arterioles called penicilliary radicles. Red pulp: filters abnormal red blood cells. Function: Filtration of abnormal
blood cells and foreign bodies such as bacteria. Immunity: IgM. Production of properdin, and tuftsin which help target fungi and
bacteria for phagocytosis. Haematopoiesis: up to 5th month gestation or in haematological disorders. Pooling: storage of 40%
platelets. Iron reutilization. Storage red blood cells-animals, not humans. Storage monocytes

Disorders of the spleen: Massive splenomegaly. Myelofibrosis. Chronic myeloid leukaemia. Visceral leishmaniasis (kala-azar).
Malaria. Gaucher's syndrome

Other causes (as above plus): Portal hypertension e.g. secondary to cirrhosis. Lymphoproliferative disease e.g. CLL, Hodgkin's.
Haemolytic anaemia. Infection: hepatitis, glandular fever. Infective endocarditis. Sickle-cell*, thalassaemia. Rheumatoid arthritis
(Felty's syndrome)
*the majority of adults patients with sickle-cell will have an atrophied spleen due to repeated infarction

119. Causes of primary chronic inflammation do not include which of the following?
A. Sarcoidosis
B. Tuberculosis
C. Ulcerative colitis
D. Prostheses
E. Chronic cholecystitis
Answer: E
Chronic cholecystitis is caused by recurrent episodes of acute inflammation.
Prosthetic implants may be the site of primary chronic inflammation. A common example clinically is breast implants which may
become encapsulated. The subsequent fibrosis then results in distortion and may be painful.

120. A 30 year old man is trapped in a house fire and sustains 30% partial and full thickness burns to his torso and limbs. Three
days following admission he has a brisk haematemesis. Which of the following is the most likely explanation for this event?
A. Dieulafoy lesion
B. Curlings ulcers
44
C. Mallory Weiss tear
D. Depletion of platelets
E. Depletion of clotting factors
Answer: B
Stress ulcers in burns patients are referred to as Curlings ulcers and may cause haematemesis.

Burns: Types of burn


Type of burn Skin layers affected Skin appearance Blanching Management
Epidermal/Superficial Epidermis Red, moist Yes
Superficial partial Epidermis and part of Pale, dry Yes Normally heals with no
thickness papillary dermis affected intervention
Deep partial thickness Epidermis, whole papillary Mottled red No Needs surgical intervention
dermis affected colour (depending on site)
Full thickness Whole skin layer and Dry, leathery hard No Burns centre
subcutaneous tissue affected wound

Depth of burn assessment: Bleeding on needle prick. Sensation. Appearance. Blanching to pressure

Percentage burn estimation: Lund Browder chart: most accurate even in children. Wallace rule of nines. Palmar surface: surface
area palm = 0.8% burn. >15% body surface area burns in adults needs urgent burn fluid resuscitation

Transfer to burn centre if: Need burn shock resuscitation. Face/hands/genitals affected. Deep partial thickness or full thickness
burns. Significant electrical/chemical burns

Escharotomies: Indicated in circumferential full thickness burns to the torso or limbs. Careful division of the encasing band of burn
tissue will potentially improve ventilation (if the burn involves the torso), or relieve compartment syndrome and oedema (where a
limb is involved)
Hettiaratchy S & Papini R. Initial management of a major burn: assessment and resuscitation. BMJ 2004;329:101-103

Theme: Adrenal gland disorders

A. Nelsons syndrome
B. Conns syndrome
C. Cushings syndrome
D. Benign incidental adenoma
E. Malignant adrenal adenoma
F. Waterhouse- Friderichsen syndrome
G. Metastatic lesion
H. Walker - Warburg syndrome
I. Phaeochromocytoma

Please select the most appropriate adrenal disorder for the scenario given. Each disorder may be selected once, more than once or
not at all.

121. A 19 year old lady is admitted to ITU with severe meningococcal sepsis. She is on maximal inotropic support and a CT scan
of her chest and abdomen is performed. The adrenal glands show evidence of diffuse haemorrhage.
Answer: Waterhouse- Friderichsen syndrome
This is often a pre-terminal event and is associated with profound sepsis and coagulopathy.

122. A 34 year old lady is admitted with recurrent episodes of non-specific abdominal pain. On each admission all blood
investigations are normal, as are her observations. On this admission a CT scan was performed. This demonstrates a 1.5cm
nodule in the right adrenal gland. This is associated with a lipid rich core. Urinary VMA is within normal limits. Other
hormonal studies are normal.
Answer: Benign incidental adenoma
This is typical for a benign adenoma.Benign adenomas often have a lipid rich core that is readily identifiable on CT scanning. In
addition the nodules are often well circumscribed.

45
123. A 38 year old man is noted to have a blood pressure of 175/110 on routine screening. On examination there are no physical
abnormalities of note. CT scanning shows a left sided adrenal mass. Plasma metanephrines are elevated.
Answer: Phaeochromocytoma
Hypertension in a young patient without any obvious cause should be investigated. Urinary VMA and plasma metanephrines are
typically elevated.

124. A 15 year old boy is admitted with colicky abdominal pain of 6 hours duration. On examination he has a soft abdomen, on
systemic examination he has brownish spots around his mouth, feet and hands. His mother underwent surgery for
intussusception, aged 12, and has similar lesions. What is the most likely underlying diagnosis?
A. Li Fraumeni syndrome
B. Peutz-Jeghers syndrome
C. Addisons disease
D. McCune -Albright syndrome
E. Appendicitis
Answer: B

This is most likely to be Peutz-Jeghers syndrome. Addisons and McCune Albright syndrome may produce similar skin changes but
the intussusception resulting from polyps combined with the autosomal inheritance pattern makes this the most likely diagnosis.

Peutz-Jeghers syndrome is an autosomal dominant condition characterised by numerous benign hamartomatous polyps in the
gastrointestinal tract. It is also associated with pigmented freckles on the lips, face, palms and soles. Around 50% of patients will
have died from a gastrointestinal tract cancer by the age of 60 years. Genetics: Autosomal dominant. Responsible gene encodes
serine threonine kinase LKB1 or STK11. Features: Hamartomatous polyps in GI tract (mainly small bowel). Pigmented lesions on
lips, oral mucosa, face, palms and soles. Intestinal obstruction e.g. intussusception (which may lead to diagnosis). Gastrointestinal
bleeding. Management: Conservative unless complications develop

125. Which of the following is not included in Multiple Endocrine Neoplasia Type 2b?
A. Phaeochromocytoma
B. Visceral ganglioneuromas
C. Thyroid medullary carcinoma
D. Zollinger Ellison syndrome
E. Marfanoid features
Answer: D
MEN IIB: Medullary thyroid cancer. Phaeochromocytoma. Mucosal neuroma. Marfanoid appearance

126. Which virus is associated with Kaposi's sarcoma?


A. Human herpes virus 8
B. Human papillomavirus 16
C. Human T-lymphotropic virus 1
D. Epstein-Barr virus
E. Human papillomavirus 18
Answer: A
Oncoviruses: Viruses which cause cancer. These may be detected on blood test and prevented by vaccine

These are the main types of oncoviruses and their diseases: Epstein-Barr virus: Burkitt's lymphoma, Hodgkin's lymphoma, Post
transfusion lymphoma, Nasopharyngeal carcinoma. Human papillomavirus 16/18: Cervical cancer, Anal cancer, Penile cancer,
Vulval cancer, Oropharyneal cancer. Human herpes virus 8 :Kaposi's sarcoma. Hepatitis B virus :Hepatocellular carcinoma.
Hepatitis C virus: Hepatocellular carcinoma. Human T-lymphotropic virus 1: Tropical spastic paraparesis
Adult T cell leukaemia

127. Which of the following is not a feature of Wallerian Degeneration?


A. May result from an axonotmesis
B. May occur in either the central or peripheral nervous systems
C. The axon remains excitable throughout the whole process
D. The distal neuronal stump is affected
E. Is a component of the healing process following neuronal injury
Answer: C
The axon loses its excitability once the process is established.
46
Wallerian degeneration: Is the process that occurs when a nerve is cut or crushed. It occurs when the part of the axon separated
from the neuron's cell nucleus degenerates. It usually begins 24 hours following neuronal injury and the distal axon remains
excitable up until this time. The degeneration of the axon is following by breakdown of the myelin sheath, a process that occurs by
infiltration of the site with macrophages. Eventually regeneration of the nerve may occur although recovery will depend on the
extent and manner of injury.

128. A 45 year old woman complains of painful tingling in her fingers. The pain is relieved by hanging the arm over the side of
the bed. She has a positive Tinel's sign. Which of the following is most likely to contribute to her diagnosis?
A. Methotrexate use
B. Crohn's disease
C. Hyperthyroidism
D. Tuberculosis
E. Rheumatoid arthritis
Answer: E
This woman has a diagnosis of carpal tunnel syndrome. Rheumatological disorders are a common cause. Clinical examination
should focus on identifying stigmata of rheumatoid arthritis, such as rheumatoid nodules, vasculitic lesions and
metacarpophalangeal joint arthritis.

Carpal tunnel syndrome is caused by compression of median nerve in the carpal tunnel. History: pain/pins and needles in thumb,
index, middle finger e.g. at night. Patient flicks hand to obtain relief. Examination: weakness of thumb abduction. Wasting of
thenar eminence (NOT hypothenar). Tinel's sign: tapping causes paraesthesia. Phalen's sign: flexion of wrist causes symptoms.
Causes of carpal tunnel syndrome: MEDIAN TRAP Mnemonic: Myxoedema. Edema premenstrually. Diabetes. Idiopathic.
Agromegaly. Neoplasm. Trauma. Rheumatoid arthritis. Amyloidosis. Pregnancy.
Management: Non surgical treatment: May resolve spontaneously. Avoid precipitants and reassurance. Night-time splints. Local
steroid injections. Surgery Complete division of the flexor retinaculum and decompression of the tunnel (successful in
approximately 80% of patients)

129. Which of the following is not an oncogene?


A. ras
B. myc
C. sis
D. Ki 67
E. erb-B
Answer: D
Ki 67 is a nuclear proliferation marker (used in immunohistochemistry). Although, Ki67 positivity is a marker of malignancy, it is not
itself, an oncogene.

Oncogenes: Oncogenes are cancer promoting genes that are derived from normal genes (proto-oncogenes). Proto-oncogenes play an
important physiological role in cellular growth. They are implicated in the development of up to 20% of human cancers. Proto-
oncogenes may become oncogenes via the following processes: Mutation (point mutation). Chromosomal translocation. Increased
protein expression. Only one mutated copy of the gene is needed for cancer to occur - a dominant effect. Classification of oncogenes:
Growth factors e.g. Sis. Transcription factors e.g. Myc. Receptor tyrosine kinase e.g. RET. Cytoplasmic tyrosine kinase e.g. Src.
Regulatory GTPases e.g. Ras

Tumour supressor genes: Tumour supressor genes restrict or repress cellular proliferation in normal cells. Their inactivation through
mutation or germ line incorporation is implicated in renal, colonic, breast, bladder and many other cancers. One of the best known
tumour supressor genes is p53. p53 gene offers protection by causing apoptosis of damaged cells. Other well known genes include
BRCA 1 and 2.

130. A 30 year old male presents with a painless swelling of the testis. Histologically the stroma has lymphocytic infiltrate.
The most likely diagnosis is :
A. Differentiated teratoma
B. Malignant undifferentiated teratoma
C. Classical seminoma
D. Spermatocytic seminoma
E. Anaplastic seminoma
Answer: C

47
Seminoma is the commonest type of testicular tumour and is more common in males aged between 30-40 years. Classical seminoma is
the commonest subtype and histology shows lymphocytic stromal infiltrate. Other subtypes include:
1. Spermatocytic: tumour cells resemble spermatocytes. Excellent prognosis. 2. Anaplastic. 3. Syncytiotrophoblast giant cells: beta
hCG present in cells. A teratoma is common in males aged 20-30 years.

131. A 48 year old women presents with recurrent loin pain and fevers. Investigation reveals a staghorn calculus of the left
kidney. Infection with which of the following organisms is most likely?
A. Staphylococcus saprophyticus
B. Proteus mirabilis
C. Klebsiella
D. E-Coli
E. Staphylococcus epidermidis
Answer: B
Infection with Proteus mirabilis accounts for 90% of all proteus infections. It has a urease producing enzyme. This will tend to favor
urinary alkalinisation which is a relative per-requisite for the formation of staghorn calculi.

132. Causes of granulomatous disease do not include:


A. Amiodarone
B. Allopurinol
C. Sulphonamides
D. Beryllium
E. Wegener's granulomatosis
Answer: A
Allopurinol and sulphonamides cause hepatic granulomas.

Theme: Facial nerve palsy

A. Adenoid cystic carcinoma


B. Cerebrovascular accident
C. Petrous temporal fracture
D. Warthins tumour
E. Sarcoidosis
F. Pleomorphic adenoma
G. Cholesteatoma

Please select the most likely cause of facial nerve palsy for the scenario given. Each option may be used once, more than once or not
at all.

133. A 22 year old man presents with symptoms of lethargy and bilateral facial nerve palsy. On examination he has
bilateral parotid gland enlargement.
Answer: Sarcoidosis
Facial nerve palsy is the commonest neurological manifestation of sarcoid. It usually resolves. The absence of ear discharge
or discrete lesion on palpation is against the other causes.

134. A 21 year old man presents with a unilateral facial nerve palsy after being hit in the head. On examination he has a
right sided facial nerve palsy and a watery discharge from his nose.
Answer: Petrous temporal fracture
Nasal discharge of clear fluid and recent head injury makes a basal skull fracture the most likely underlying diagnosis.

135. A 43 year old lady presents with symptoms of chronic ear discharge and a right sided facial nerve palsy. On
examination she has foul smelling fluid draining from her right ear and a complete right sided facial nerve palsy.
Answer: Cholesteatoma
Foul smelling ear discharge and facial nerve weakness is likely to be due to cholesteatoma. The presence of a neurological
deficit is a sinister feature.

Facial nerve palsy: Sarcoid: Facial nerve palsy is the most frequent neurological manifestation of sarcoid. Affects right and left side
with equal frequency, may be bilateral. Typically resolves in up to 80% of cases. Cholesteatoma: Destructive and expanding growth
48
of keratinised squamous epithelium. Patients often complain of chronic ear discharge. Infection with Pseudomonas may occur
resulting in foul smell to discharge. Aquired lesions usually arise from the Pars flaccida region of the tympanic membrane. Surgical
removal and mastoidectomy may be needed. Recurrence rates of 20% may be seen following surgery. Basal skull fracture: History
of head injury. Presence of features such as Battles sign on examination. Clinical presence of CSF leak strongly supports diagnosis.
Assessment is by CT and MRI scan. Prophylactic antibiotics should be given in cases of CSF leak

136. A 59 year old lady is referred from the NHS breast screening program. A recent mammogram is reported as showing
linear, branching microcalcification with coarse granules. Which disease process is the most likely underlying cause of
these appearances?
A. Invasive lobular cancer
B. Lobular carcinoma in situ
C. Cribriform type ductal carcinoma in situ
D. Comedo type ductal carcinoma in situ
E. Fibroadenosis
Answer: D
Comedo type DCIS is usually associated with microcalcifications. Cribriform lesions are usually multifocal but less likely to form
microcalcifications. Lobular cancers and in situ lesions rarely form microcalcifications and are difficult to detect using mammography.

Breast cancer - In situ disease: Breast cancer that has yet to invade the basement membrane is referred to as in situ disease. Both
ductal and lobular in situ variants are recognised. Ductal carcinoma in situ: Sub types include; comedo, cribriform, micropapillary
and solid. Comdeo DCIS is most likely to form microcalcifications. Cribriform and micropapillary are most likely to be multifocal.
Most lesions are mixed (composed of multiple subtypes). High nuclear grade DCIS is associated with more malignant characteristics
(loss of p53, increase erbB2 expression). Local excision of low nuclear grade DCIS will usually produce satisfactory outcomes.
Multifocal lesions, large and high nuclear grade lesions will usually require mastectomy. Lobular carcinoma in situ: Much rarer than
DCIS. Does not form microcalcifications. Usually single growth pattern. When an invasive component is found it is less likely to be
associated with axillary nodal metastasis than with DCIS. Low grade LCIS is usually treated by monitoring rather than excision

137. In patients with an annular pancreas where is the most likely site of obstruction?
A. The first part of the duodenum
B. The second part of the duodenum
C. The fourth part of the duodenum
D. The third part of the duodenum
E. The duodeno-jejunal flexure

Answer: B
The pancreas develops from two foregut outgrowths (ventral and dorsal). During rotation the ventral bud and adjacent gallbladder and
bile duct lie together and fuse. When the pancreas fails to rotate normally it can compress the duodenum with development of
obstruction. Usually occurring as a result of associated duodenal malformation. The second part of the duodenum is the commonest
site.

Pancreas: The pancreas is a retroperitoneal organ and lies posterior to the stomach. It may be accessed surgically by dividing the
peritoneal reflection that connects the greater omentum to the transverse colon. The pancreatic head sits in the curvature of the
duodenum. It's tail lies close to the hilum of the spleen, a site of potential injury during splenectomy.

Relations: Posterior to the pancreas


Pancreatic head: Inferior vena cava. Common bile duct. Right and left renal veins. Superior mesenteric vein and artery. Pancreatic
neck: Superior mesenteric vein, portal vein. Pancreatic body: Left renal vein: Crus of diaphragm. Psoas muscle. Adrenal gland.
Kidney. Aorta. Pancreatic tail: Left kidney

Anterior to the pancreas

Pancreatic head: 1st part of the duodenum. Pylorus. Gastroduodenal artery. SMA and SMV(uncinate process). Pancreatic body:
Stomach. Duodenojejunal flexure. Pancreatic tail: Splenic hilum. Superior to the pancreas: Coeliac trunk and its branches common
hepatic artery and splenic artery. Grooves of the head of the pancreas: 2nd and 3rd part of the duodenum. Arterial supply: Head:
pancreaticoduodenal artery. Rest: splenic artery. Venous drainage: Head: superior mesenteric vein. Body and tail: splenic vein.
Ampulla of Vater: Merge of pancreatic duct and common bile duct. Is an important landmark, halfway along the second part of the
duodenum, that marks the anatomical transition from foregut to midgut (also the site of transition between regions supplied by coeliac
trunk and SMA).
49
Theme: Chest pain

A. Achalasia
B. Pulmonary embolus
C. Dissection of thoracic aorta
D. Boerhaaves syndrome
E. Gastro-oesophageal reflux
F. Carcinoma of the oesophagus
G. Oesophageal candidiasis

Please select the most likely cause for chest pain for the scenario given. Each option may be used once, more than once or not at all.

138. A 43 year old man who has a long term history of alcohol misuse is admitted with a history of an attack of vomiting
after an episode of binge drinking. After vomiting he developed sudden onset left sided chest pain, which is pleuritic in
nature. On examination he is profoundly septic and drowsy with severe epigastric tenderness and left sided chest pain.
Answer: Boerhaaves syndrome
In patients with Boerhaaves the rupture is often on the left side. The story here is typical. All patients should have a contrast
study to confirm the diagnosis and the affected site prior to thoracotomy.

139. A 22 year old man is admitted with severe retrosternal chest pain and recurrent episodes of dysphagia. These occur
sporadically and often resolve spontaneously. On examination there are no physical abnormalities and the patient
seems well.
Answer: Achalasia
Achalasia may produce severe chest pain and many older patients may undergo cardiac investigations prior to endoscopy.
Endoscopic injection with botulinum toxin is a popular treatment (although the benefit is not long lasting). Cardiomyotomy is
a more durable alternative.

140. An obese 53 year old man presents with symptoms of recurrent retrosternal discomfort and dyspepsia. This is
typically worse at night after eating a large meal. On examination there is no physical abnormality to find.
Answer: Gastro-oesophageal reflux

Patients with GORD often have symptoms that are worse at night. In this age group an Upper GI endoscopy should probably
be performed.

Surgical chest pain

Dissection of thoracic aorta: Tearing interscapular pain. Discrepancy in arterial blood pressures taken in both arms. May show
mediastinal widening on chest x-ray

Diffuse oesophageal spasm: Spectrum of oesophageal motility disorders. Caused by uncoordinated contractions of oesphageal
muscles. May show "nutcracker oesophagus" on barium swallow. Symptoms include dysphagia, retrosternal discomfort and dyspepsia

Gastro-oesphageal reflux: Common cause of retrosternal discomfort. Usually associated with symptoms of regurgitation,
odynophagia and dyspepsia. Symptoms usually well controlled with PPI therapy. Risk factors include obesity, smoking and excess
alcohol consumption

Boerhaaves syndrome: Spontaneous rupture of the oesophagus. Caused by episodes of repeated vomiting often in association with
alcohol excess. Typically there is an episode of repetitive vomiting followed by severe chest and epigastric pain. Diagnosis is by CT
and contrast studies. Treatment is surgical; during first 12 hours primary repair, beyond this usually creation of controlled fistula with
a T Tube, delay beyond 24 hours is associated with fulminent mediastinitis and is usually fatal.

Achalasia: Difficulty swallowing, dysphagia to both liquids and solids and sometimes chest pain. Usually caused by failure of distal
oesphageal inhibitory neurons. Diagnosis is by pH and manometry studies together with contrast swallow and endoscopy. Treatment
is with either botulinum toxin, pneumatic dilatation or cardiomyotomy

50
141. A 32 year old woman presents with an episode of haemoptysis and is found to have metastatic tumour present within
the parenchyma of the lungs. This is biopsied and subsequent histology shows clear cells. What is the most likely
primary site?
A. Kidney
B. Breast
C. Liver
D. Adrenal
E. Bone
Answer: A
Clear cell tumours are a sub type of renal cell cancer it is associated with specific genetic changes localised to chromosome 3.

142. A laceration of the wrist produces a median nerve transaction. The wound is clean and seen immediately after injury.
Collateral soft tissue damage is absent. The patient asks what the prognosis is. You indicate that the nerve should
regrow at approximately:
A. 0.1 mm per day
B. 1 mm per day
C. 5 mm per day
D. 1 cm per day
E. None of the above
Answer: B
Transaction of a peripheral nerve results in hemorrhage and in retraction of the several nerve ends. Almost immediately, degeneration
of the axon distal to the injury begins. Degeneration also occurs in the proximal fragment back to the first node of Ranvier.
Phagocytosis of the degenerated axonal fragments leaves neurilemmal sheath with empty cylindrical spaces where the axons were.
Several days following the injury, axons from the proximal fragment begin to regrow. If they make contact with the distal
neurilemmal sheath, regrowth occurs at about the rate of 1 mm/day. However, if associated trauma, fracture, infection, or separation of
neurilemmal sheath ends precludes contact between axons, growth is haphazard and a traumatic neuroma is formed. When neural
transaction is associated with widespread soft tissue damage and hemorrhage (with increased probability of infection), many surgeons
choose to delay reapproximation of the severed nerve end for 3 to 4 weeks.

Nerve injury: There are 3 types of nerve injury: Neuropraxia: Nerve intact but electrical conduction is affected. Full recovery.
Autonomic function preserved. Wallerian degeneration does not occur. Axonotmesis: Axon is damaged and the myelin sheath is
preserved. The connective tissue framework is not affected. Wallerian degeneration occurs. Neurotmesis: Disruption of the axon,
myelin sheath and surrounding connective tissue. Wallerian degeneration occurs. Wallerian Degeneration: Axonal degeneration
distal to the site of injury. Typically begins 24-36 hours following injury. Axons are excitable prior to degeneration occurring. Myelin
sheath degenerates and is phagocytosed by tissue macrophages.

Nerve repair: Neuronal repair may only occur physiologically where nerves are in direct contact. Where a large defect is present the
process of nerve regeneration is hampered and may not occur at all or result in the formation of a neuroma. Where nerve regrowth
occurs it typically occurs at a rate of 1mm per day.

156. Which of the following statements relating to gastric cancer is untrue?

A. It is associated with chronic helicobacter pylori infection


B. 5% of gastric malignancies are due to lymphoma
C. In the Lauren classification the diffuse type of adenocarcinoma typically presents as a large exophytic growth in the
antrum
D. Smoking is a risk factor
E. It is associated with acanthosis nigricans
Answer: C
The Lauren classification describes a diffuse type of adenocarcinoma (Linitis plastica type lesion) and an intestinal type. The diffuse
type is often deeply infiltrative and may be difficult to detect on endoscopy. Barium meal appearances can be characteristic.

157. Which of the following statements relating to Gardners syndrome variant of familial adenomatous polyposis coli is false?

A. It is an autosomal dominant condition


B. Patients may develop retroperitoneal desmoid tumours
C. The vast majority of the polyps are benign and thus the risk of colorectal cancer is small
D. Patients are at increased risk of thyroid cancer
51
E. It is characterised by a mutation in the APC gene
Answer: C
The multiple polyps increase the risk of malignancy and most patients should undergo a colectomy.

158. A 60-year-old man presents with lower urinary tract symptoms and is offered a PSA test. Which one of the following
could interfere with the PSA level?

A. Vigorous exercise in the past 48 hours


B. Poorly controlled diabetes mellitus
C. Drinking more than 4 units of alcohol in the past 48 hours
D. Smoking
E. Recent cholecystectomy
Answer: A
PSA testing: Prostate specific antigen (PSA) is a serine protease enzyme produced by normal and malignant prostate epithelial cells.
It has become an important tumour marker but much controversy still exists regarding its usefulness as a screening tool. The NHS
Prostate Cancer Risk Management Programme (PCRMP) has published updated guidelines in 2009 on how to handle requests for PSA
testing in asymptomatic men. A recent European trial (ERSPC) showed a statistically significant reduction in the rate of death prostate
cancer by 20% in men aged 55 to 69 years but this was associated with a high risk of over-diagnosis and over-treatment. Having
reviewed this and other data the National Screening Committee have decided not to introduce a prostate cancer screening programme
yet but rather allow men to make an informed choice. Age-adjusted upper limits for PSA were recommended by the PCRMP*:
PSA level by age (ng/ml): 50-59 years:3.0; 60-69 years:4.0; > 70 years:5.0

PSA levels may also be raised by**: benign prostatic hyperplasia (BPH). Prostatitis and urinary tract infection (NICE recommend to
postpone the PSA test for at least 1 month after treatment). Ejaculation (ideally not in the previous 48 hours). Vgorous exercise
(ideally not in the previous 48 hours). Urinary retention. Instrumentation of the urinary tract

Poor specificity and sensitivity: around 33% of men with a PSA of 4-10 ng/ml will be found to have prostate cancer. With a PSA of
10-20 ng/ml this rises to 60% of men. Around 20% with prostate cancer have a normal PSA. Various methods are used to try and add
greater meaning to a PSA level including age-adjusted upper limits and monitoring change in PSA level with time (PSA velocity or
PSA doubling time)
*aide memoire for upper PSA limit: (age - 20) / 10
**whether digital rectal examination actually causes a rise in PSA levels is a matter of debate

159. Which of the symptoms below is least typical of pancreatic cancer?

A. Painless jaundice
B. Hyperamylasaemia
C. Hyperglycaemia
D. Weight loss
E. Classical Courvoisier syndrome
Answer: B
Raised serum amylase is relatively uncommon. The typical Courvoisier syndrome typically occurs in 20% and hyperglycaemia occurs
in 15-20%.

Pancreatic cancer: Adenocarcinoma. Risk factors: Smoking, diabetes, Adenoma, Familial adenomatous polyposis. Mainly occur in
the head of the pancreas (70%). Spread locally and metastasizes to the liver. Carcinoma of the pancreas should be differentiated from
other periampullary tumours with better prognosis. Clinical features: Weight loss. Painless jaundice. Epigastric discomfort (pain
usually due to invasion of the coeliac plexus is a late feature). Pancreatitis. Trousseau's sign: migratory superficial thrombophlebitis.
Investigations: USS: May miss small lesions. CT Scanning (pancreatic protocol). If unresectable on CT then no further staging
needed. PET/CT for those with operable disease on CT alone. ERCP/ MRI for bile duct assessment. Staging laparoscopy to exclude
peritoneal disease. Management: Head of pancreas: Whipple's resection (SE dumping and ulcers). Newer techniques include pylorus
preservation and SMA/ SMV resection. Carcinoma body and tail: poor prognosis, distal pancreatectomy if operable. Usually adjuvent
chemotherapy for resectable disease. ERCP and stent for jaundice and palliation. Surgical bypass may be needed for duodenal
obstruction.

160. A 53 year old man presents with dyspepsia. An upper GI endoscopy is performed and Helicobacter pylori is identified. A
duodenal ulcer is present in the first part of the duodenum. Duodenal biopsies are taken and demonstrate epithelium that
resembles cells of the gastric antrum. Which of the following is the most likely explanation for this process?

52
A. Hyperplasia of the crypts of Lieberkhun
B. Duodenal metaplasia
C. Duodenal dysplasia
D. Duodenal carcinoma
E. Hyptertrophy of Brunners glands
Answer: B
Metaplasia = cell type conversion

The process involved is metaplasia. During metaplasia there is no direct carcinogenesis, however the persistent presence of
precipitants of metaplasia will lead to malignant changes in cells.
Metaplastic changes in the duodenal cap are frequently seen in association with H-Pylori induced ulcers. It typically resolves after
ulcer healing and eradication therapy.

Metaplasia: Definition: reversible change of differentiated cells to another cell type. May represent an adaptive substitution of cells
that are sensitive to stress by cell types better able to withstand the adverse environment. Can be a normal physiological response
(ossification of cartilage to form bone). Most common epithelial metaplasia occurs with transformation of columnar cells to squamous
cells (smoking causes ciliated columnar cells to be replaced by squamous epithelial cells; Schistosomiasis). Metaplasia from
squamous to columnar cells occurs in Barrett oesophagus. If the metaplastic stimulus is removed, the cells will return to their original
pattern of differentiation. However, if the stimulus is not removed then progression to dysplasia may occur. Not considered directly
carcinogenic, however the factors which predispose to metaplasia, if persistent may induce malignant transformation. The
pathogenesis involves a reprogramming of stem cells that are known to exist in normal tissues, or of undifferentiated mesenchymal
cells present in connective tissue. In a metaplastic change, these precursor cells differentiate along a new pathway.

161. A 22 year old lady presents with symptoms and signs of hyperthyroidism. Her diagnostic work up results in a diagnosis of
Graves disease. Which of the following best describes the pathophysiology of the condition?

A. Formation of IgG antibodies to the TSH receptors on the thyroid gland


B. Formation of IgG antibodies to the TRH receptors on the anterior pituitary
C. Formation of IgM antibodies to the TSH receptors on the thyroid gland
D. Formation of IgA antibodies to the TSH receptors on the thyroid gland
E. Formation of IgM antibodies to the TRH receptors on the anterior pituitary
Answer: A
Usually IgG antibodies are formed against the TSH receptors on the thyroid gland. Which is why the TSH level is often very low in
Graves disease.

Hormones of the thyroid gland: Triiodothyronine T3: Major hormone active in target cells. Thyroxine T4: Most prevalent form in
plasma, less biologically active than T3. Calcitonin: Lowers plasma calcium

Synthesis and secretion of thyroid hormones: Thyroid actively concentrates iodide to twenty five times the plasma concentration.
Iodide is oxidised by peroxidase in the follicular cells to atomic iodine which then iodinates tyrosine residues contained in
thyroglobulin. Iodinated tyrosine residues in thyroglobulin undergo coupling to either T3 or T4. Process is stimulated by TSH, which
stimulates secretion of thyroid hormones. The normal thyroid has approximately 3 month reserves of thyroid hormones.

LATS and Graves disease: In Graves disease patients develop IgG antibodies to the TSH receptors on the thyroid gland. This results
in chronic and long term stimulation of the gland with release of thyroid hormones. The typically situation is raised thyroid hormones
and low TSH. Thyroid receptor autoantibodies should be checked in individuals presenting with hyperthyroidism as they are present
in up to 85% cases.

Theme: Breast disease

A. Tuberculosis
B. Actinomycosis
C. Duct ectasia
D. Fibroadenoma
E. Fat necrosis
F. Intraductal papilloma
53
G. Breast abscess

What is the most likely diagnosis for each scenario given? Each diagnosis may be used once, more than once or not at all.

162. A 32 year old Indian woman presents with a tender breast lump. She has a 2 month old child. Clinically there is a tender,
fluctuant mass of the breast.
Answer: Breast abscess
This lady is likely to be breast feeding and is at risk of mastitis. This may lead to an abscess if not treated. Staphylococcus aureus
is usually the causative organism.

163. A 53 year old lady presents with a green nipple discharge. On examination she has discharge originating from multiple
ducts and associated nipple inversion.
Answer: Duct ectasia
Duct ectasia is common during the period of breast involution that occurs during the menopausal period. As the ducts shorten they
may contain insipiated material. The discharge will often discharge from several ducts.

164. A 52 year old lady presents with an episode of nipple discharge. It is usually clear in nature. On examination the discharge
is seen to originate from a single duct and although it appears clear, when the discharge is tested with a labstix it is shown
to contain blood. Imaging and examination shows no obvious mass lesion,
Answer: Intraductal papilloma
Intraductal papilloma usually cause single duct discharge. The fluid is often clear, although it may be blood stained. If the fluid is
tested with a labstix (little point in routiine practice) then it will usually contain small amounts of blood. A microdocechtomy may
be performed.

Non malignant breast disease: Duct ectasia: Mammary duct ectasia may be seen in up to 25% of normal female breasts. Patients
usually present with nipple discharge, which may be from single or multiple ducts (usually present age >50 years). The discharge is
often thick and green. Duct ectasia is a normal varient of breast involution and is not the same condition as periductal mastitis
Periductal mastitis: Present at younger age than duct ectasia. May present with features of inflammation, abscess or mammary duct
fistula. Stongly associated with smoking. Usually treated with antibiotics, abscess will require drainage
Intraductal papilloma: Growth of papilloma in a single duct. Usually presents with clear or blood stained discharge originating from
a single duct. No increase in risk of malignancy
Breast abscess: Lactational mastitis is common. Infection is usually with Staphylococcus aureus. On examination there is usually a
tender fluctuant mass. Treatment is with antibiotics and ultrasound guided aspiration. Overlying skin necrosis is an indication for
surgical debridement, which may be complicated by the development of a subsequent mammary duct fistula.
Tuberculosis: Rare in western countries, usually secondary TB. Affects women later in child bearing period. Chronic breast or
axillary sinus is present in up to 50% cases. Diagnosis is by biopsy culture and histology

165. A 45 year old man has widespread metastatic adenocarcinoma of the colon. Which of the following tumour markers is most
likely to be elevated?
A. CA19-9
B. Carcinoembryonic antigen
C. Alpha Feto Protein
D. CA 125
E. Beta BCG
Answer: B
Screening for colonic cancer using CEA is not justified
Carcinoembryonic antigen is elevated in colonic cancer, typically in relation to disease extent with highest serum levels noted in
metastatic disease. It is falsely elevated in a number of non-malignant disease states such as cirrhosis and colitis and for this reason it
has no role in monitoring colitics for colonic cancer[1].

Colorectal cancer screening and diagnosis: Overview: Most cancers develop from adenomatous polyps. Screening for colorectal
cancer has been shown to reduce mortality by 16%. The NHS now has a national screening programme offering screening every 2
years to all men and women aged 60 to 69 years. Patients aged over 70 years may request screening. Eligible patients are sent faecal
occult blood (FOB) tests through the post. Patients with abnormal results are offered a colonoscopy. At colonoscopy, approximately: 5
out of 10 patients will have a normal exam. 4 out of 10 patients will be found to have polyps which may be removed due to their
premalignant potential. 1 out of 10 patients will be found to have cancer

54
Diagnosis: Essentially the following patients need referral: Altered bowel habit for more than six weeks. New onset of rectal bleeding.
Symptoms of tenesmus. Colonoscopy is the gold standard, provided it is complete and good mucosal visualisation is achieved. Other
options include double contrast barium enema and CT colonography.
Staging: Once a malignant diagnosis is made patients with colonic cancer will be staged using chest / abdomen and pelvic CT.
Patients with rectal cancer will also undergo evaluation of the mesorectum with pelvic MRI scanning.
For examination purposes the Dukes and TNM systems are preferred.
Tumour markers
Carcinoembryonic antigen (CEA) is the main tumour marker in colorectal cancer. Not all tumours secrete this, and it may be raised in
conditions such as IBD. However, absolute levels do correlate (roughly) with disease burden and whilst this marker may not be used
extensively in follow up, it can be useful for investigation of patients with cancer of unknown primary.

166. Which of the following does not cause hypercalcaemia?

A. Thiazides
B. DiGeorge syndrome
C. Vitamin A
D. Rhabdomylosis
E. Sarcoidosis
Answer: B

VITAMINS TRAP: Vitamins A & D, Immobilization, Thyrotoxicosis, Addison's disease, Milk-alkali syndrome, Inflammatory
disorders, Neoplastic diseases, Sarcoidosis, Thiazides and other drugs, Rhabdomyolysis, AIDS, Paget's disease, Parenteral nutrition,
Parathyroid disease.

The parathyroid glands don't form as a result of this syndrome.

Hypercalcaemia: Main causes: Malignancy. Primary hyperparathyroidism


Less common: Sarcoidosis (extrarenal synthesis of calcitriol ). Thiazides, lithium. Immobilisation. Pagets disease. Vitamin A/D
toxicity. Thyrotoxicosis. MEN. Milk alkali syndrome

Clinical features: “Stones, bones, abdominal moans, and psychic groans”


Theme: Diseases affecting the great vessels

A. Aortic coarctation
B. Cervical rib
C. Takayasu's arteritis
D. Subclavian steal syndrome
E. Patent ductus arteriosus
F. Aortic dissection

Please select the most likely underlying cause for the symptoms described. Each option may be used once, more than once or not at
all.

166. A 24 year old lady from Western India presents with symptoms of lethargy and dizziness, worse on turning her head. On
examination her systolic blood pressure is 176/128. Her pulses are impalpable at all peripheral sites. Auscultation of her
chest reveals a systolic heart murmur.
Answer: Takayasu's arteritis
Takayasu's arteritis most commonly affects young Asian females. Pulseless peripheries are a classical finding. The CNS symptoms
may be variable.

167. A 48 year old man notices that he is becoming increasingly dizzy when he plays squash, in addition he has also developed
cramping pain in his left arm. One day he is inflating his car tyre with a hand held pump, he collapses and is brought to
hospital.
Answer: Subclavian steal syndrome
Subclavian steal syndrome is associated with a stenosis or occlusion of the subclavian artery, proximal to the origin of the vertebral
artery. As a result the increased metabolic needs of the arm then cause retrograde flow and symptoms of CNS vascular
insufficiency.

55
168. shows evidence of rib notching. Auscultation of his chest reveals a systolic murmur which is loudest at the posterior aspect
of the fourth intercostal space.
Answer: Aortic coarctation
Coarctation of the aorta may occur due to the remnant of the ductus arteriosus acting as a fibrous constrictive band of the aorta.
Weak arm pulses may be seen, radiofemoral delay is the classical physical finding. Collateral flow through the intercostal vessels
may produce notching of the ribs, if the disease is long standing.

Vascular disease
Aortic dissection: Chest pain (anterior chest pain- ascending aorta, back pain - descending aorta). Widening of aorta on chest x-ray.
Diagnosis made by CT scanning. Treatment is either medical (Type B disease) or surgical (Type A disease)

Cervical rib: Supernumery fibrous band arising from seventh cervical vertebra. Incidence of 1 in 500. May cause thoracic outlet
syndrome. Treatment involves surgical division of rib

Subclavian steal syndrome: Due to proximal stenotic lesion of the subclavian artery. Results in retrograte flow through vertebral or
internal thoracic arteries. The result is that decrease in cerebral blood flow may occur and produce syncopal symptoms. A duplex scan
and/ or angiogram will delineate the lesion and allow treatment to be planned

Takayasu's arteritis: Large vessel granulomatous vasculitis. Results in intimal narrowing. Most commonly affects young asian
females. Patients present with features of mild systemic illness, followed by pulseless phase with symptoms of vascular insufficiency.
Treatment is with systemic steroids

Patent ductus arteriosus: Ductus arteriosus is a normal foetal vessel that closes spontaneously after birth. Results in high pressure,
oxygenated blood entering the pulmonary circuit. Untreated patients develop symptoms of congestive cardiac failure

Coarctation of the aorta: Aortic stenosis at the site of the ductus arteriosus insertion. Most common in boys and girls with Turners
syndrome. Patients may present with symptoms of arterial insufficiency, such as syncope and claudication. Blood pressure mismatch
may be seen, as may mismatch of pulse pressure in the upper and lower limbs. Treatment is either with angioplasty or surgical
resection (the former is the most common)

169. A 25 year old man is found to have carcinoid syndrome. Which of the following hormones is released by carcinoids?
A. Serotonin
B. Dopamine
C. Nor adrenaline
D. Adrenaline
E. Aldosterone
Answer: A
Rule of thirds:1/3 multiple. 1/3 small bowel. 1/3 metastasize. 1/3 second tumour

Carcinoids secrete serotonin. Carcinoid syndrome will only occur in the presence of liver metastasis as the hormone released from
primary lesions will typically be metabolised by the liver.

170. Which one of the following genes protects against neoplasms?


A. sis
B. p53
C. ras
D. myc
E. src
Answer: B
p53 is a tumour supressor gene and located on chromosome 17. It plays an important role in causing cells that are undergoing
neoplastic changes to enter an apoptotic pathway.

171. A 55 year old man with a long history of achalasia is successfully treated by a Hellers Cardiomyotomy. Several years later
he develops an oesophageal malignancy. Which of the following lesions is most likely to be present?
A. Adenocarcinoma
B. Gastrointestinal stromal tumour
C. Leiomyosarcoma

56
D. Rhabdomyosarcoma
E. Squamous cell carcinoma
Answer: E
Achalasia is a rare condition. However, even once treated there is an increased risk of malignancy. When it does occur it is most likely
to be of squamous cell type.

Oesophageal cancer: Incidence is increasing. In most cases in the Western world this increase is accounted for by a rise in the
number of cases of adenocarcinoma. In the UK adenocarcinomas account for 65% of cases. Barretts oesophagus is a major risk factor
for most cases of oesophageal adenocarcinoma. In other regions of the world squamous cancer is more common and is linked to
smoking, alcohol intake, diets rich in nitrosamines and achalasia. Surveillance of Barretts is important as it imparts a 30 fold increase
in cancer risk and if invasive malignancy is diagnosed early then survival may approach 85% at 5 years.
Diagnosis: Upper GI endoscopy is the first line test. Contrast swallow may be of benefit in classifying benign motility disorders but
has no place in the assessment of tumours. Staging is initially undertaken with CT scanning of the chest, abdomen and pelvis. If overt
metastatic disease is identified using this modality then further complex imaging is unnecessary. If CT does not show metastatic
disease, then local stage may be more accurately assessed by use of endoscopic ultrasound. Staging laparoscopy is performed to detect
occult peritoneal disease. PET CT is performed in those with negative laparoscopy. Thoracoscopy is not routinely performed.
Treatment: Operable disease is best managed by surgical resection. The most standard procedure is an Ivor- Lewis type
oesophagectomy. This procedure involves the mobilisation of the stomach and division of the oesophageal hiatus. The abdomen is
closed and a right sided thoracotomy performed. The stomach is brought into the chest and the oesophagus mobilised further. An
intrathoracic oesophagogastric anastomosis is constructed. Alternative surgical strategies include a transhiatal resection (for distal
lesions), a left thoraco-abdominal resection (difficult access due to thoracic aorta) and a total oesophagectomy (McKeown) with a
cervical oesophagogastric anastomosis.
The biggest surgical challenge is that of anastomotic leak, with an intrathoracic anastomosis this will result in mediastinitis. With high
mortality. The McKeown technique has an intrinsically lower systemic insult in the event of anastmotic leakage.
In addition to surgical resection many patients will be treated with adjuvent chemotherapy.

172. Which of the following genes is not implicated in the adenoma-carcinoma sequence in colorectal cancer?
A. src
B. c-myc
C. APC
D. p53
E. K-ras
Answer: A
Other genes involved are: MCC, DCC, c-yes, bcl-2

Colorectal cancer: Annually about 150,000 new cases are diagnosed and 50,000 deaths from the disease. About 75% will have
sporadic disease and 25% will have a family history. Colorectal tumours comprise a spectrum of disease ranging from adenomas
through to polyp cancers and frank malignancy. Polyps may be categorised into: neoplastic polyps, adenomatous polyps and non
neoplastic polyps. The majority of adenomas are polypoidal lesions, although flat lesions do occur and may prove to be dysplastic.
Non-neoplastic polyps include hyperplastic, juvenile, hamartomatous, inflammatory, and lymphoid polyps, which have not generally
been thought of as precursors of cancer. Three characteristics of adenomas that correlate with malignant potential have been
characterised. These include increased size, villous architecture and dysplasia. For this reason most polyps identified at colonoscopy
should be removed. The transformation from polyp to cancer is described by the adenoma - carcinoma sequence and its principles
should be appreciated. Essentially genetic changes accompany the transition from adenoma to carcinoma; key changes include APC,
c-myc, K RAS mutations and p53 deletions.
A 63 year old lady is suspected as having sarcoidosis. She is sent to the general surgeons and a lymph node biopsy is performed.

173. Which of the following histological features is most likely to be identified in a lymph node if sarcoid is present?
A. Psammoma bodies
B. Extensive necrosis
C. Dense eosinophillic infiltrates
D. Asteroid bodies
E. None of the above
Answer: D
Asteroid bodies are often found in the granulomas of individuals with sarcoid. Unlike the granulomata associated with tuberculosis the
granulomas of sarcoid are rarely associated with extensive necrosis.

174. Brown tumours of bone are associated with which of the following?
57
A. Hyperthyroidism
B. Hypothyroidism
C. Hyperparathyroidism
D. Hypoparathyroidism
E. Osteopetrosis
Answer: C
Brown tumors are tumors of bone that arise in settings of excess osteoclast activity, such as hyperparathyroidism, and consist of
fibrous tissue, woven bone and supporting vasculature, but no matrix. They are radiolucent on x-ray. The osteoclasts consume the
trabecular bone that osteoblasts lay down and this front of reparative bone deposition followed by additional resorption can expand
beyond the usual shape of the bone, involving the periosteum thus causing bone pain. They appear brown because haemosiderin is
deposited at the site.

175. A 73 year old man is recovering following an emergency Hartmans procedure performed for an obstructing sigmoid cancer.
The pathology report shows a moderately differentiated adenocarcinoma that invades the muscularis propria, 3 of 15 lymph
nodes are involved with metastatic disease. What is the correct stage for this?
A. Astler Coller Stage B2
B. Dukes stage A
C. Dukes stage B
D. Dukes stage C
E. Dukes stage D
Answer: D
The involvement of lymph nodes makes this Dukes C. In the Astler Coller system the B and C subsets are split to B1 and B2 and C1
and C2. Where C2 denotes involvement of the nodes in conjunction with penetration of the muscularis propria.

Dukes classification: Gives the extent of spread of colorectal cancer: Dukes A:Tumour confined to the mucosa (90%). Dukes
B:Tumour invading bowel wall (70%). Dukes C:Lymph node metastases (45%). Dukes D:Distant metastases (6%)(20% if resectable)
5 year survival in brackets

176. A 55 year old man with dyspepsia undergoes an upper GI endoscopy. An irregular erythematous area is seen to protrude
proximally from the gastro-oesophageal junction. Apart from specialised intestinal metaplasia, which of the following cell
types should also be present for a diagnosis of Barretts oesophagus to be made?
A. Goblet cell
B. Neutrophil
C. Lymphocytes
D. Epithelial cells
E. Macrophages
Answer: A
Goblet cells need to be present for a diagnosis of Barrett's oesophagus to be made.

Barrett's oesophagus: Intestinal metaplasia. Squamous epithelium replaced by columnar epithelium in the lower oesophagus. 3 types
of columnar epithelium: 1. Junctional. 2. Atrophic fundal. 3. Specialised. Presence of goblet cells important in identification.
Premalignant change (progress to dysplasia). Risk of adenocarcinoma. Risk factors: middle age, men, smoker, Caucasian, gastro-
oesophageal reflux, obesity
Treatment: Long term proton pump inhibitor. Consider pH and manometry studies in younger patients who may prefer to consider an
anti reflux procedure. Regular endoscopic monitoring (more frequently if moderate dysplasia). With quadrantic biopsies every 2-3 cm.
If severe dysplasia be very wary of small foci of cancer

177. Which of the following amino acids is present in all types of collagen?
A. Alanine
B. Aspartime
C. Glycine
D. Tyrosine
E. Cysteine
Answer: C
Collagen has a generic structure of Glycine- X- Y, where X and Y are variable sub units. The relatively small size of the glycine
molecule enables collagen to form a tight helical structure.

58
Collagen: One of the major connective tissue proteins. Composed of 3 polypeptide strands that are woven into a helix. Numerous
hydrogen bonds exist within molecule to provide additional strength. Many sub types but commonest sub type is I (90% of bodily
collagen). Vitamin c is important in establishing cross links
Collagen Diseases: Osteogenesis imperfect. Ehlers Danlos
Osteogenesis imperfecta: 8 Subtypes. Defect of type I collagen. In type I the collagen is normal quality but insufficient quantity. Type
II- poor quantity and quality. Type III- Collagen poorly formed, normal quantity. Type IV- Sufficient quantity but poor quality
Patients have bones which fracture easily, loose joint and multiple other defects depending upon which sub type they suffer from
Ehlers Danlos: Multiple sub types. Abnormality of types 1 and 3 collagen. Patients have features of hypermobility. Individuals are
prone to joint dislocations and pelvic organ prolapse. In addition to many other diseases related to connective tissue defects

178. An 8 year old boy presented with a painless swelling on the superotemporal aspect of his orbit. It was smooth on
examination, produced no visual disturbances. Following excision it was found to be lined by squamous epithelium and hair
follicles. Which of the following lesions most closely matches these findings?
A. Dermoid cyst
B. Desmoid tumour
C. Lipoma
D. Sebaceous cyst
E. Schwannoma
Answer: A
Dermoid cysts are embryological remnants and may be lined by hair and squamous epithelium (like teratomas). They are often located
in the midline and may be linked to deeper structures resulting in a dumbbell shape to the lesion. Complete excision is requires as they
have a propensity to local recurrence if not excised.
Desmoid tumours are a different entity, they most commonly develop in ligaments and tendons. They are also referred to as aggressive
fibromatosis and consist of fibroblast dense lesions (resembling scar tissue). They should be managed in a similar manner to soft
tissue sarcomas.

Skin Diseases: Skin lesions may be referred for surgical assessment, but more commonly will come via a dermatologist for definitive
surgical management. Skin malignancies include basal cell carcinoma, squamous cell carcinoma and malignant melanoma.

Basal Cell Carcinoma: Most common form of skin cancer. Commonly occur on sun exposed sites apart from the ear. Sub types
include nodular, morphoeic, superficial and pigmented. Typically slow growing with low metastatic potential. Standard surgical
excision, topical chemotherapy and radiotherapy are all successful. As a minimum a diagnostic punch biopsy should be taken if
treatment other than standard surgical excision is planned.

Squamous Cell Carcinoma: Again related to sun exposure. May arise in pre - existing solar keratoses. May metastasise if left.
Immunosupression (e.g. Following transplant), increases risk. Wide local excision is the treatment of choice and where a diagnostic
excision biopsy has demonstrated SCC, repeat surgery to gain adequate margins may be required.

Malignant Melanoma
The main diagnostic features (major criteria):Change in size. Change in shape. Change in colour
Secondary features (minor criteria): Diameter >6mm. Inflammation. Oozing or bleeding. Altered sensation

Treatment: Suspicious lesions should undergo excision biopsy. The lesion should be removed in completely as incision biopsy can
make subsequent histopathological assessment difficult. Once the diagnosis is confirmed the pathology report should be reviewed to
determine whether further re-exicision of margins is required (see below):

Margins of excision-Related to Breslow thickness: Lesions 0-1mm thick:1cm. Lesions 1-2mm thick:1- 2cm (Depending upon site
and pathological features). Lesions 2-4mm thick:2-3 cm (Depending upon site and pathological features). Lesions >4 mm thick:3cm

Further treatments such as sentinel lymph node mapping, isolated limb perfusion and block dissection of regional lymph node groups
should be selectively applied.

Kaposi Sarcoma: Tumour of vascular and lymphatic endothelium. Purple cutaneous nodules. Associated with immunosupression.
Classical form affects elderly males and is slow growing. Immunosupression form is much more aggressive and tends to affect those
with HIV related disease.

Non malignant skin disease: Dermatitis Herpetiformis: Chronic itchy clusters of blisters. Linked to underlying gluten enteropathy
(coeliac disease).

59
Dermatofibroma: Benign lesion. Firm elevated nodules. Usually history of trauma. Lesion consists of histiocytes, blood vessels and
fibrotic changes.

Pyogenic granuloma: Overgrowth of blood vessels. Red nodules. Usually follow trauma. May mimic amelanotic melanoma.

Acanthosis nigricans: Brown to black, poorly defined, velvety hyperpigmentation of the skin. Usually found in body folds such as the
posterior and lateral folds of the neck, the axilla, groin, umbilicus, forehead, and other areas. The most common cause of acanthosis
nigricans is insulin resistance, which leads to increased circulating insulin levels. Insulin spillover into the skin results in its abnormal
increase in growth (hyperplasia of the skin). In the context of a malignant disease, acanthosis nigricans is a paraneoplastic syndrome
and is then commonly referred to as acanthosis nigricans maligna. Involvement of mucous membranes is rare and suggests a
coexisting malignant condition

179. A 55 year old man from Hong Kong presents with left sided otalgia and recurrent episodes of epistaxis. On examination his
pharynx appears normal. Examination of his neck reveals left sided cervical lymphadenopathy. What is the most likely
underlying diagnosis?
A. Antrochoanal polyp
B. Nasopharyngeal carcinoma
C. Adenocarcinoma of the tonsil
D. Angiofibroma
E. Globus syndrome
Answer: B
Given this mans ethnic origin and presenting features a nasopharyngeal carcinoma is the most likely underlying diagnosis.

180. Patients with suspected temporal arteritis are often sent for temporal artery biopsy. Which statement is true?
A. Temporal artery biopsy is only diagnostic if there is visual loss
B. Biopsy is typically taken from the non-symptomatic side to avoid the risk of blindness
C. Pre-operative localisation with duplex is mandatory
D. Biopsies may be non diagnostic in over 50% of cases
E. Biopsies are usually performed under general anaesthesia
Answer: D
Temporal artery biopsies are frequently non diagnostic. They should be taken from the symptomatic side and though not mandatory a
duplex ultrasound is a helpful investigation, particularly if they mark the artery. It is usually performed under local anaesthetic.

Temporal artery biopsy: Superficial temporal artery is a terminal branch of the external carotid artery

Main indication: Temporal arteritis


American College of Rheumatology guidelines recommend a temporal artery biopsy if: Age of onset older than 50 years. New-onset
headache or localized head pain. Temporal artery tenderness to palpation or reduced pulsation. ESR > 50 mm/h

Histopathology: Vessel wall granulomatous arteritis with mononuclear cell infiltrates and giant cell formation

Procedure: Position: supine, head 45 degrees. USS doppler to locate the superficial temporal artery or palpate. Local anaesthetic.
Artery within temporoparietal fascia. Clamp and ligate the vessel. Cut 3-5cm. Ligate the remaining ends with absorbable suture. Close
the skin

Contraindication
Glucocorticoid therapy > 30 days
Risks
Injury to facial or auriculotemporal nerve

181. Which of the following best describes the processes underpinning type IV hypersensitivity reactions?
A. Deposition of immune complexes of IgG and antigen at the site of inflammation
B. Deposition of IgA complexes at the site of inflammation
C. Deposition of IgM and IgG complexes at the site of inflammation
D. Degranulation of mast cells at the site of inflammation
E. T cell mediated response at the site of inflammation
Answer: E
Theme: Bone tumours
60
A. Osteosarcoma
B. Fibrosarcoma
C. Osteoclastoma
D. Ewings sarcoma
E. Leiomyosarcoma
F. Chondrosarcoma
G. Rhabdomyosarcoma
H. Osteoid osteoma
I. Malignant fibrous histiocytoma

Please select the most appropriate lesion for the clinical scenario given. Each option may be used once, more than once or not at all.

182. A 16 year-old boy presents to his GP with loss of weight, pain and fever. On examination, a soft tissue mass is palpable over
the mid-thigh region
Answer: Ewings sarcoma
Ewing's sarcoma is a malignant round cell tumour occurring in the diaphysis of the long bones in the children. These are not
confined to the ends of long bones. x Rays often show a large soft-tissue mass with concentric layers of new bone formation (
'onion-peel' sign). The ESR may be elevated, thus suggesting an inflammatory or an infective cause such as osteomyelitis;
although osteomyelitis usually affects the metaphyseal region in children. Treatment is with chemotherapy and surgical excision,
an endoprothesis may be used to conserve the limb.

183. A 75 year old lady presents with weight loss, pain and a swelling over her left knee. She has been treated for Pagets
disease of the bone for some time.
Answer: Osteosarcoma
Osteosarcoma may complicate Pagets disease of bone in up to 10% cases. Radiological appearances include bone destruction coupled
with new bone formation, periosteal elevation may also occur. Surgical resection is the main treatment.

184. A 17-year-old girl presents with weight loss, fever and a swelling over her right knee. Movements of her knee are
restricted. A plain x-ray of the affected site shows multiple lytic and lucent lesions with clearly defined borders.
You answered Malignant fibrous histiocytoma
Answer: Osteoclastoma
Osteoclastoma has a characteristic appearance on x-ray with multple lytic and lucent areas (Soap bubble) appearances. Pathological
fractures may occur. The disease is usually indolent.

185. A 22 year old man is undergoing an abdominal ultrasound scan as part of a series of investigations for abdominal pain.
The radiologist notes that there is evidence of splenic atrophy. What is the most likely cause?

A. Letterer-Siwe disease
B. Coeliac disease
C. Malaria
D. Niemann-Pick disease
E. Sarcoidosis
Answer: B
Splenic atrophy may occur in coeliac disease together with the appearance of Howell-Jolly bodies in erythrocytes. Letterer - Siwe
disease is a form of Histiocytosis X in which macrophages proliferate.

186. Which statement relating to phaeochromocytoma is untrue?

A. They are tumours of chromaffin cells in the adrenal medulla.


B. They are bilateral in 10% of cases.
C. When located in an extra adrenal location have a higher incidence of malignancy.
D. May be associated with an elevated urinary VMA.
E. Up to 40% may have a blood pressure within the normal range.
Answer: E
Normotension is seen in around 10% cases. The remainder show a degree of hypertension. Neuroendocrine tumour of the chromaffin
cells of the adrenal medulla. Hypertension an

61
187. A 69 year old man presents with a purple lesion on his forearm. It is excised and an a 3 cm Merkel cell tumour is
diagnosed. Which of the following statements relating to this diagnosis is false?

A. He should undergo a sentinel lymph node biopsy.


B. Lymphovascular invasion is typically seen histologically
C. They are more common in immunosupressed patients
D. Histologically they may resemble pyogenic granuloma
E. They are associated with visceral metastasis
Answer: D
Merkel cell tumours are rare cutaneous tumours. Histologically they consist of sheets and nodules of hyperchromatic epithelial cells,
with high rates of mitosis and apoptosis. As such they are relatively easy to distinguish from pyogenic granuloma which has no
features of malignancy and would not show lymphovascular invasion.

Merkel cell tumours of the skin: Rare but aggressive tumour. Develops from intra epidermal Merkel cells. Usually presents on
elderly, sun damaged skin. The periorbital area is the commonest site. Histologically these tumours appear within the dermis and
subcutis. The lesions consist of sheets and nodules of small hyperchromatic epithelial cells with high rates of mitosis and apoptosis.
Lymphovascular invasion is commonly seen. Pre-existing infection with Merkel Cell Polyomavirus is seen in 80% cases.
TreatmentSurgical excision is first line. Margins of 1cm are required. Lesions >10mm in diameter should undergo sentinel lymph
node biopsy. Adjuvant radiotherapy is often given to reduce the risk of local recurrence.
Prognosis: With lymph node metastasis 5 year survival is 50% or less. Small lesions without nodal spread are usually associated with
a 5 year survival of 80%.

188. A 58 year old lady undergoes a screening mammogram and appearances are suspicious for ductal carcinoma in situ. A
stereotactic core biopsy is performed. If ductal carcinoma in situ is to be diagnosed, which of the following pathological
features must not be present?

A. Nuclear pleomorphism
B. Coarse chromatin
C. Abnormal mitoses
D. Angiogenesis
E. Dysplastic cells infiltrating the suspensory ligaments of the breast
Answer: E

The presence of invasion is a hallmark of invasive disease and thus would not be a feature of DCIS. Angiogenesis may occur in
association with high grade DCIS.

189. Which of the following does not occur as a pathological response to extensive burns?

A. Plasma leakage into interstitial space


B. Polycythaemia
C. Increased haematocrit
D. Keratinocyte migration during healing
E. Cardiac output reduction by 50% in first 30 minutes
Answer: B
Haemolysis is the main pathological response.

Theme: Chest pain

A. Pulmonary embolism
B. Acute exacerbation asthma
C. Physiological
D. Mitral valve stenosis
E. Aortic dissection
F. Mitral regurgitation
62
G. Bronchopneumonia
H. Tuberculosis
I. None of the above

What is the most likely diagnosis for the scenario given? Each option may be used once, more than once or not at all.

190. A 28 year old Indian woman, who is 18 weeks pregnant, presents with increasing shortness of breath, chest pain and
coughing clear sputum. She is apyrexial, blood pressure is 140/80 mmHg, heart rate 130 bpm and saturations 94% on
15L oxygen. On examination there is a mid diastolic murmur, there are bibasal crepitations and mild pedal oedema. She
suddenly deteriorates and has a respiratory arrest. Her chest x-ray shows a whiteout of both of her lungs.
Answer: Mitral valve stenosis
Mitral stenosis is the commonest cause of cardiac abnormality occurring in pregnant women. Mitral stenosis is becoming less
common in the UK population, however should be considered in women from countries were there is a higher incidence of
rheumatic heart disease. Mitral stenosis causes a mid diastolic murmur which may be difficult to auscultate unless the patient is
placed into the left lateral position. These patients are at risk of atrial fibrillation (up tp 40%), which can also contribute to rapid
decompensation. Physiological changes in pregnancy may cause an otherwise asymptomatic patient to suddenly deteriorate.
Balloon valvuloplasty is the treatment of choice.

191. A 28 year old woman, who is 30 weeks pregnant, presents with sudden onset chest pain associated with loss of
consciousness. Her blood pressure is 170/90 mmHg, saturations on 15L oxygen 93%, heart rate 120 bpm and she is
apyrexial. On examination there is an early diastolic murmur, occasional bibasal creptitations and mild peal oedema. An
ECG shows ST elevation in leads II, III and aVF.
Answer: Aortic dissection
Aortic dissection is associated with the 3rd trimester of pregnancy, connective tissue disorders (Marfan's, Ehlers- Danlos) and
bicuspid valve. Patients may complain of a tearing chest pain or syncope. Clinically they may be hypertensive. The right
coronary artery may become involved in the dissection, causing myocardial infarct in up to 2% cases (hence ST elevation in the
inferior leads). An aortic regurgitant murmur may be auscultated.

192. A 28 year old woman, who is 18 weeks pregnant, presents with sudden chest pain. Her blood pressure is 150/70 mmHg,
saturations are 92% on 15L oxygen and her heart rate is 130 bpm. There are no murmurs and her chest is clear. There is
signs of thrombophlebitis in the left leg.
Answer: Pulmonary embolism
Chest pain, hypoxia and clear chest on auscultation in pregnancy should lead to a high suspicion of pulmonary embolism.
Pregnant women can decompensate rapidly from cardiac compromise.

Chest pain in pregnancy

Aortic dissection: Predisposing factors in pregnancy are hypertension, congenital heart disease and Marfan's syndrome. Mainly
Stanford type A dissections. Sudden tearing chest pain, transient syncope. Patient may be cold and clammy, hypertensive and have an
aortic regurgitation murmur. Involvement of the right coronary artery may cause inferior myocardial infarction

Surgical management (by gestational timeframe): < 28/40: Aortic repair with the fetus kept in utero; 28-32/40:Dependent on fetal
condition; > 32/40:Primary Cesarean section followed by aortic repair at the same operation

Mitral stenosis: Most cases associated with rheumatic heart disease. Becoming less common in British women; suspect in Immigrant
women. Commonest cardiac condition in pregnancy. Commonly associated with mortality. Valve surgery; balloon valvuloplasty
preferable

Pulmonary embolism: Leading cause of mortality in pregnancy. Half dose scintigraphy; CT chest if underlying lung disease should
aid diagnosis. Treatment with low molecular weight heparin throughout pregnancy and 4-6 weeks after childbirth. Warfarin is contra
indicated in pregnancy

193. A 67 year old man is investigated for biliary colic and a 4.8 cm abdominal aortic aneurysm is identified. Which of the
following statements relating to this condition is untrue?
A. The wall will be composed of dense fibrous tissue only
B. The majority are located inferior to the renal arteries
C. They occur most often in current or former smokers
63
D. He should initially be managed by a process of active surveillance
E. Aortoduodenal fistula is a recognised complication following repair.
Answer: A
They are true aneurysms and have all 3 layers of arterial wall.

Abdominal aorta aneurysm: Abdominal aortic aneurysms are a common problem in vascular surgery. They may occur as either true
or false aneurysm. With the former all 3 layers of the arterial wall are involved, in the latter only a single layer of fibrous tissue forms
the aneurysm wall. True abdominal aortic aneurysms have an approximate incidence of 0.06 per 1000 people. They are commonest in
elderly men and for this reason the UK is now introducing the aneurysm screening program with the aim of performing an abdominal
aortic ultrasound measurement in all men aged 65 years.

Causes: Several different groups of patients suffer from aneurysmal disease. The commonest group is those who suffer from standard
arterial disease, i.e. Those who are hypertensive, have diabetes and have been or are smokers. Other patients such as those suffering
from connective tissue diseases such as Marfan's may also develop aneurysms. In patients with abdominal aortic aneurysms the
extracellular matrix becomes disrupted with a change in the balance of collagen and elastic fibres.

Management: Most abdominal aortic aneurysms are an incidental finding. Symptoms most often relate to rupture or impending
rupture. 20% rupture anteriorly into the peritoneal cavity. Very poor prognosis. 80% rupture posteriorly into the retroperitoneal space.
The risk of rupture is related to aneurysm size, only 2% of aneurysms measuring less than 4cm in diameter will rupture over a 5 year
period. This contrasts with 75% of aneurysms measuring over 7cm in diameter. This is well explained by La Places' law which relates
size to transmural pressure. For this reason most vascular surgeons will subject patients with an aneurysm size of 5cm or greater to CT
scanning of the chest, abdomen and pelvis with the aim of delineating anatomy and planning treatment. Depending upon co-
morbidities, surgery is generally offered once the aneurysm is between 5.5cm and 6cm.
Indications for surgery: Symptomatic aneurysms (80% annual mortality if untreated). Increasing size above 5.5cm if asymptomatic.
Rupture (100% mortality without surgery)

Surgical procedures
Abdominal aortic aneurysm repair: Procedure: GA. Invasive monitoring (A-line, CVP, catheter). Incision:Midline or transverse.
Bowel and distal duodenum mobilised to access aorta. Aneurysm neck and base dissected out and prepared for cross clamp. Systemic
heparinisation. Cross clamp (distal first). Longitudinal aortotomy
Atherectomy. Deal with back bleeding from lumbar vessels and inferior mesenteric artery. Insert graft either tube or bifurcated
depending upon anatomy. Suture using Prolene (3/0 for proximal , distal anastomosis suture varies according to site)
Clamps off: End tidal CO2 will rise owing to effects of reperfusion, at this point major risk of myocardial events. Haemostasis.
Closure of aneurysm sac to minimise risk of aorto-enteric fistula. Closure: Loop 1 PDS or Prolene to abdominal wall. Skin- surgeons
preference. Post operatively: ITU (Almost all). Greatest risk of complications following emergency repair. Complications: Embolic-
gut and foot infarcts. Cardiac - owing to premorbid states, reperfusion injury and effects of cross clamp. Wound problems. Later risks
related to graft- infection and aorto-enteric fistula

Special groups: Supra renal AAA: These patients will require a supra renal clamp and this carries a far higher risk of complications
and risk of renal failure. Ruptured AAA: Preoperatively the management depends upon haemodynamic instability. In patients with
symptoms of rupture (typical pain, haemodynamic compromise and risk factors) then ideally prompt laparotomy. In those with vague
symptoms and haemodynamic stability the ideal test is CT scan to determine whether rupture has occurred or not. Most common
rupture site is retroperitoneal 80%. These patients will tend to develop retroperitoneal haematoma. This can be disrupted if Bp is
allowed to rise too high so aim for Bp 100mmHg. Operative details are similar to elective repair although surgery should be swift,
blind rushing often makes the situation worse. Plunging vascular clamps blindly into a pool of blood at the aneurysm neck carries the
risk of injury the vena cava that these patients do not withstand. Occasionally a supracoeliac clamp is needed to effect temporary
control, although leaving this applied for more than 20 minutes tends to carry a dismal outcome.
EVAR: Increasingly patients are now being offered endovascular aortic aneurysm repair. This is undertaken by surgeons and
radiologists working jointly. The morphology of the aneurysm is important and not all are suitable. Here is a typical list of those
features favoring a suitable aneurysm: Long neck. Straight iliac vessels. Healthy groin vessels
Clearly few AAA patients possess the above and compromise has to be made. The use of fenestrated grafts can allow supra renal
AAA to be treated.
Procedure: GA. Radiology or theatre. Bilateral groin incisions. Common femoral artery dissected out. Heparinisation. Arteriotomy and
insertion of guide wire. Dilation of arteriotomy. Insertion of EVAR Device. Once in satisfactory position it is released. Arteriotomy
closed once check angiogram shows good position and no endoleak
Complications: Endoleaks depending upon site are either Type I or 2. These may necessitate re-intervention and all EVAR patients
require follow up . Details are not needed for MRCS.

194. Which of the following statements in relation to the p53 tumour suppressor protein is false?
A. It may induce necrosis of cells with non repairable DNA damage
64
B. It is affected in Li Fraumeni syndrome
C. It can induce DNA repair
D. It can halt the cell cycle
E. It may inhibit angiogenesis
Answer: A
When DNA cannot be repaired it will induce cellular apoptosis (not necrosis)

195. Which of the following cell types is most likely to be identified in the wall of a fistula in ano?
A. Squamous cells
B. Goblet cells
C. Columnar cells
D. Ciliated columnar cells
E. None of the above
Answer: A
A fistula is an abnormal connection between two epithelial lined surfaces, in the case of a fistula in ano it will be lined by squamous
cells.

Fistulas: A fistula is defined as an abnormal connection between two epithelial surfaces. There are many types ranging from
Branchial fistulae in the neck to entero-cutaneous fistulae abdominally. In general surgical practice the abdominal cavity generates the
majority and most of these arise from diverticular disease and Crohn's. As a general rule all fistulae will resolve spontaneously as
long as there is no distal obstruction. This is particularly true of intestinal fistulae.

The four types of fistulae are: Enterocutaneous


These link the intestine to the skin. They may be high (>1L) or low output (<1L) depending upon source. Duodenal /jejunal fistulae
will tend to produce high volume, electrolyte rich secretions which can lead to severe excoriation of the skin. Colo-cutaneous fistulae
will tend to leak faeculent material. Both fistulae may result from the spontaneous rupture of an abscess cavity onto the skin (such as
following perianal abscess drainage) or may occur as a result of iatrogenic input. In some cases it may even be surgically desirable e.g.
mucous fistula following sub total colectomy for colitis.Suspect if there is excess fluid in the drain.

Enteroenteric or Enterocolic: This is a fistula that involves the large or small intestine. They may originate in a similar manner to
enterocutaneous fistulae. A particular problem with this fistula type is that bacterial overgrowth may precipitate malabsorption
syndromes. This may be particularly serious in inflammatory bowel disease.

Enterovaginal: Aetiology as above.

Enterovesicular: This type of fistula goes to the bladder. These fistulas may result in frequent urinary tract infections, or the passage
of gas from the urethra during urination.

Management: Some rules relating to fistula management: They will heal provided there is no underlying inflammatory bowel disease
and no distal obstruction, so conservative measures may be the best option. Where there is skin involvement, protect the overlying
skin, often using a well fitted stoma bag- skin damage is difficult to treat. A high output fistula may be rendered more easily managed
by the use of octreotide, this will tend to reduce the volume of pancreatic secretions. Nutritional complications are common especially
with high fistula (e.g. high jejunal or duodenal) these may necessitate the use of TPN to provide nutritional support together with the
concomitant use of octreotide to reduce volume and protect skin. When managing perianal fistulae surgeons should avoid probing the
fistula where acute inflammation is present, this almost always worsens outcomes. When perianal fistulae occur secondary to Crohn's
disease the best management option is often to drain acute sepsis and maintain that drainage through the judicious use of setons whilst
medical management is implemented. Always attempt to delineate the fistula anatomy, for abscesses and fistulae that have an intra
abdominal source the use of barium and CT studies should show a track. For perianal fistulae surgeons should recall Goodsall's rule in
relation to internal and external openings.
196. A 22 year old man is referred to the surgical clinic. He has been complaining of varicose veins for many years. On
examination he has extensive varicosities of the right leg, there are areas of marked port wine staining. The
saphenofemoral junction is competent on doppler assessment. The most likely underlying diagnosis is:
A. Deep vein thrombosis
B. Klippel-Trenaunay syndrome
C. Varicose veins due to sapheno-popliteal junction incompetence
D. Sturge - Weber syndrome
E. Angiosarcoma
Answer: B
Sturge - Weber syndrome is a an arteriovenous malformation affecting the face and CNS, the peripheral vessels are not affected.
Simple varicose veins should not typically be associated with port wine staining, not should a DVT or angiosarcoma.
65
Klippel-Trenaunay syndrome: Klippel-Trenaunay-Weber syndrome generally affects a single extremity, although cases of multiple
affected limbs have been reported. The leg is the most common site followed by the arms, the trunk, and rarely the head and the neck

Signs and symptoms: The birth defect is diagnosed by the presence of a combination of these symptoms: One or more distinctive
port-wine stains with sharp borders. Varicose veins. Hypertrophy of bony and soft tissues, that may lead to local gigantism or
shrinking. An improperly developed lymphatic system. In some cases, port-wine stains (capillary port wine type) may be absent. Such
cases are very rare and may be classified as "atypical Klippel-Trenaunay syndrome". KTS can either affect blood vessels, lymph
vessels, or both. The condition most commonly presents with a mixture of the two. Those with venous involvement experience
increased pain and complications.

197. A 68 year old man presents with an ulcerated lesion on his right cheek. It is excised and on histological assessment a
squamous cell carcinoma is diagnosed. It measures 25mm in diameter and is 4mm deep. Which of the following statements
relating to this condition is false?
A. In this particular case margins of at least 6mm are required
B. Use of cryosurgery to treat this patients lesion would have been unsafe
C. Use of radiotherapy to treat this lesion would have been unsafe
D. This patients local recurrence rate may approach 15%
E. The disese usually spreads via lymphatics
Answer: C

Poor prognostic factors in SCC: Size >20mm (local recurrence rate of up to 15%). Depth greater than 4mm (risk of metastasis up to
30%)

This man has an SCC with significant risk of metastasis. Although cryotherapy may be used to treat SCC it would be most unsafe in
this setting as the lesion extends deeply. However, radiotherapy is a safe treatment modality for SCC and may be used in selected
cases. It is unwise to use radiotherapy in areas prone to radionecrosis e.g. the nose.

Squamous cell carcinoma of the skin: Second most common skin malignancy. Derived from epidermal keratinocytes. Commonest in
fair skinned individuals in sun exposed sites. May occur in perianal and genital skin especially in association with Human Papilloma
Virus 16 and 18 infections.

Groups at high risk: Renal transplant and on immunosuppression. Individuals with HIV. Those who have received psoralen UVA
therapy. Chronic wounds (Marjolins ulcer). Xeroderma pigmentosum. Oculocutaneous albinism

Prognosis: Good Prognosis: Well differentiated tumours. <20mm diameter. <2mm deep. No associated diseases. Poor prognosis:
Poorly differentiated tumours. >20mm in diameter. >4mm deep. Immunosupression for whatever reason

Treatment: Surgical excision with 4mm margins if lesion <20mm in diameter. If tumour >20mm then margins should be 6mm.

198. A 23 year old man presents with weight loss fatigue and lymphadenopathy. He is diagnosed with tuberculosis. Which of the
following processes most closely matches the underlying pathological process?
A. Type 1 hypersensitivity reaction
B. Type 2 hypersensitivity reaction
C. Type 3 hypersensitivity reaction
D. Type 4 hypersensitivity reaction
E. None of the above
Answer: D
Granulomas (which occur in tuberculosis) are a feature of Type 4 hypersensitivity reactions.

199. A 73 year old man undergoes an emergency amputation for severe lower limb sepsis and gangrene. Post operatively he
develops disseminated intravascular coagulation. Which of the following clotting factors will be most rapidly consumed in
this process?
A. Factor V and VIII
B. Factor I

66
C. Factor I and III
D. Factor III and VII
E. Factor VI and VIII
Answer: A
DISSEM-NATED: D-Dx: D dimer, I-Immune complexes, S-Snakebite, shock, heatstroke, S-SLE, E-Eclampsia, HELLP syndrome,
M-Massive tissue damage, I-Infections: viral and bacterial, N-Neoplasms, A-Acute promyelocytic leukemia , T-Tumor products:
Tissue Factor (TF) and TF-like factors released by carcinomas of pancreas, prostate, lung,
colon, stomach, E-Endotoxins (bacterial), D-Dead fetus (retained). DIC Will tend to consume factors five and eight intially (and
platelets).
Disseminated intravascular coagulation: Simultaneous coagulation and haemorrhage caused by initially formation of thrombi which
consume clotting factors (factors 5,8) and platelets, ultimately leading to bleeding
Causes include:Infection. Malignancy. Trauma e.g. major surgery, burns, shock, dissecting aortic aneurysm. Liver disease. Obstetric
complications
Key points: Clinically bleeding is usually a dominant feature, bruising, ischaemia and organ failure. Blood tests: prolonged clotting
times, thrombocytopenia, decreased fibrinogen, increased fibrinogen degradation products. Treat the underlying cause and supportive
management

200. A 63 year old Japanese man presents with epigastric discomfort and iron deficiency anaemia. He undergoes an upper GI
endoscopy, where the following appearances are found: The most likely diagnosis is:
A. Squamous cell carcinoma
B. Linitis plastica
C. Leiomyosarcoma
D. Gastric varices
E. None of the above
Answer: B
During upper GI endoscopy, a linitis plastica lesion may prevent gastric distension.
Linitis plastica produces a diffuse infiltrating lesion, the stomach is fibrotic and rigid and will not typically distend. This may be
described as a 'leather bottle stomach'. Diagnosis is made with a combination of pathology examination with endoscopy, radiological
or surgical assessment. Pathologically signet-ring cell proliferation occurs.

Theme: Genetics and cancer

A. Multiple endocrine neoplasia type I


B. Multiple endocrine neoplasia type II
C. Gardner's syndrome
D. Lynch Syndrome
E. Kartagener's syndrome
F. Neurofibromatosis Type I
G. Neurofibromatosis Type II

Please select the most likely condition for the disease process described. Each option may be used once, more than once or not at all.

201. A 40 year old male is found to have multiple colonic polyps during a colonoscopy. He mentions that he has extra teeth.
Answer: Gardner's syndrome
Gardner's syndrome is an AD disorder, characterised by: Colonic polyps, supernumerary teeth, jaw osteomas, congenital
hypertrophy of retinal pigment. osteomas of the skull, thyroid cancer, epidermoid cysts, fibromas and sebaceous cysts.

202. A 10 year old boy who has learning difficulties, reports a difference in size between his two legs.
Answer: Neurofibromatosis Type I
Neurofibromatosis type I. A hallmark finding is a plexiform neurofibroma, which is a sheet of neurofibromatosis tissue which
encases major nerves. In children this attracts extra blood circulation, which can accelerate growth of the affected limb.
Other features include: Schwannoma, > 6. Cafe au lait spots, axillary freckling, Lisch nodules, Optic glioma.
Meningiomas,Glioma, or Schwannoma.

203. A 22 year old is found to have bilateral acoustic neuromas.


Answer: Neurofibromatosis Type II
In NF2 bilateral acoustic neuromas are characteristic with a FH of Neurofibroma,

67
204. A 52 year old lady is referred to the breast clinic with symptoms of nipple discharge. The discharge is usually thick and
green. Which of the following statements relating to the most likely underlying diagnosis is untrue?
A. The majority of patients will be smokers
B. Typically produces blood stained nipple discharge
C. It is not associated with increased risk of breast cancer
D. May result in development of mammary duct fistula
E. May require total duct excision (Hadfields operation) if it fails to resolve
Answer: B
Blood stained nipple discharge should always be investigated.
Nipple fluid cytology is generally unhelpful.
Discharge of this type of material is most likely to be due to duct ectasia. Green or brown discharge is most common. Blood stained
discharge should raise concern of intraductal papilloma or cancer.

Causes of nipple discharge: Physiological: During breast feeding. GalactorrhoeaCommonest cause may be response to emotional
events, drugs such as histamine receptor anatagonists are also implicated. Hyperprolactinaemia: Commonest type of pituitary
tumour. Microadenomas <1cm in diameter. Macroadenomas >1cm in diameter. Pressure on optic chiasm may cause bitemporal
hemianopia. Mammary duct ectasia: Dilatation breast ducts. Most common in enopausal women. Discharge typically thick and
green in colour. Most common in smokers. Carcinoma: Often blood stained. May be underlying mass or axillary lymphadenopathy
Intraductal papilloma: Commoner in younger patients. May cause blood stained discharge. There is usually no palpable lump

Assessment of patients: Examine breast and determine whether there is mass lesion present. All mass lesions should undergo Triple
assessment.

Reporting of investigations
Where a mass lesion is suspected or investigations are requested these are prefixed using a system that denotes the investigation type
e.g. M for mammography, followed by a numerical code as shown below: 1:No abnormality; 2:Abnormality with benign features;
3:Indeterminate probably benign; 4:Indeterminate probably malignant; 5:Malignant

Management of non malignant nipple discharge: Exclude endocrine disease. Nipple cytology unhelpful. Smoking cessation advice
for duct ectasia. For duct ectasia with severe symptoms, total duct excision may be warrented.

205. Which of the following statements relating to gas gangrene is untrue?


A. There is necrosis with putrefaction
B. The causative pathogens may be detected on normal perineal skin
C. Treatment with low dose penicillin is indicated
D. Hyperbaric oxygen may be beneficial
E. Clostridium perfringens is a recognised cause
Answer: C
Rapid surgery and high dose antibiotics are indicated in the treatment of gas gangrene.

Meleney's Gangrene and Necrotising Fasciitis.


Necrotising fasciitis: Advancing soft tissue infection associated with fascial necrosis. Uncommon, but can be fatal. In many cases
there is underlying background immunosuppression e.g. Diabetes. Caused by polymicrobial flora (aerobic and anaerobic) and MRSA
is seen increasingly in cases of necrotising fasciitis. Streptococcus is the commonest organism in isolated pathogen infection (15%).
Meleneys gangrene: Meleneys is a similar principle but the infection is more superficially sited than necrotising fasciitis and often
confined to the trunk. Fournier gangrene: Necrotising fasciitis affecting the perineum. Polymicrobial with E.coli and Bacteroides
acting in synergy. Clinical features: Fever. Pain. Cellulitis. Oedema. Induration. Numbness. Late findings: Purple/black skin
discolouration. Blistering. Haemorrhagic bullae. Crepitus. Dirty Dishwater fluid discharge. Septic shock. Diagnosis is mainly clinical
Management: Radical surgical debridement forms the cornerstone of management. Sterile dressing is used to dress the wound.
Reconstructive surgery is considered once the infection is completely treated

206. A 30 year old man presents with abdominal distension, a laparotomy is performed, at operation the abdomen is filled with
a large amount of gelatinous fluid. What is the most likely underlying diagnosis?
A. Infection with entamoeba histolytica
B. Pseudomxyoma peritonei
C. Metastatic colonic cancer
68
D. Chylous ascites
E. None of the above
Answer: B
Pseudomyxoma is associated with the deposition of large amounts of gelatinous material. The appendix is the commonest organ or
origin.

Pseudomyxoma Peritonei: Rare mucinous tumour. Most commonly arising from the appendix (other abdominal viscera are also
recognised as primary sites). Incidence of 1-2/1,000,000 per year. The disease is characterised by the accumulation of large amounts
of mucinous material in the abdominal cavity. Treatment: Is usually surgical and consists of cytoreductive surgery (and often
peritonectomy c.f Sugarbaker procedure) combined with intra peritoneal chemotherapy with mitomycin C.
Survival is related to the quality of primary treatment and in Sugarbakers own centre 5 year survival rates of 75% have been quoted.
Patients with disseminated intraperitoneal malignancy from another source fare far worse.
In selected patients a second look laparotomy is advocated and some practice this routinely.

207. A 30 year old man is suspected of having appendicitis. At operation an inflamed Meckels diverticulum is found. Which of
the following vessels is responsible for the blood supply to a Meckels diverticulum?
A. Right colic artery
B. Vitelline artery
C. Appendicular artery
D. Internal iliac artery
E. External iliac artery
Answer: B
The vitelline arteries supply a Meckels these are usually derived from the ileal arcades.

208. A 23 year old man suffers a thermal injury to his left hand. It becomes red and painful. Which of the following mediators
are not involved in this process?
A. Histamine
B. Free radicals
C. Prostaglandins
D. Leukotrienes
E. Serotonin
Answer: B
Acute inflammation is not mediated by free radicals. Chemical mediators facilitate the spread of inflammation into normal tissue
Chemical mediators include: Lysosomal compounds. Chemokines such as serotinin and histamine (released by platelets and mast
cells). Other enzyme cascades producing inflammatory mediators include: Complement, kinin, coagulation system & fibrinolytic
system

209. Which of the following associations are incorrect?


A. Afro-Caribbean skin and keloid scarring
B. Extensive third degree burns and wound contraction
C. Chemotherapy and dehisence of healed wounds
D. Poor healing at the site of previous radiotherapy
E. Zinc deficiency and delayed healing
Answer: C

Wound healing: Surgical wounds are either incisional or excisional and either clean, clean contaminated or dirty. Although the stages
of wound healing are broadly similar their contributions will vary according to the wound type.

210. A 23 year old man fractures his right tibia in a sporting accident. At which point in the healing process is fracture callus
most likely to be visible radiologically?
A. 1 day
B. 7 days
C. 8 weeks
D. 6 weeks
E. 3 weeks
Answer: E
Fracture callus is composed of fibroblasts and chondroblasts and the synthesis of fibrocartilage. It is typically visible on radiographs
approximately 3 weeks following injury. If delayed then there may be risk of non union.

Fracture healing
69
Bone fracture: Bleeding vessels in the bone and periosteum. Clot and haematoma formation. The clot organises over a week
(improved structure and collagen). The periosteum contains osteoblasts which produce new bone. Mesenchymal cells produce
cartilage (fibrocartilage and hyaline cartilage) in the soft tissue around the fracture. Connective tissue + hyaline cartilage = callus. As
the new bone approaches the new cartilage, endochondral ossification occurs to bridge the gap. Trabecular bone forms. Trabecular
bone is resorbed by osteoclasts and replaced with compact bone

Factors Affecting Fracture Healing: Age, Malnutrition, Bone disorders: osteoporosis. Systemic disorders: diabetes. Marfan's
syndrome and Ehlers-Danlos syndrome cause abnormal musculoskeletal healing. Drugs: steroids, non steroidal anti inflammatory
agents. Type of bone: Cancellous (spongy) bone fractures are usually more stable, involve greater surface areas, and have a better
blood supply than cortical (compact) bone fractures. Degree of Trauma: The more extensive the injury to bone and surrounding soft
tissue, the poorer the outcome.Vascular injury: Especially the femoral head, talus, and scaphoid bones. Degree of immobilization.
Intra-articular Fractures: These fractures communicate with synovial fluid, which contains collagenases that retard bone healing.
Separation of Bone Ends: Normal apposition of fracture fragments is needed for union to occur. Inadequate reduction, excessive
traction, or interposition of soft tissue will prevent healing. Infection.

211. Of the options below, which does not cause lymphadenopathy?


A. Kawasaki disease
B. Systemic Lupus Erthematosus
C. Phenytoin
D. Hydrallazine
E. Amiodarone
Answer: E
Lymphadenopathy: Lymphadenopathy in the neck, axillae, groins and abdomen. Need to note: solitary/multiple, defined/indistinct,
hard/rubbery/soft, tender/painless

212. A 23 year old man is reviewed on the ward 10 days following a laparotomy. The wound is inspected and is healing well.
Which of the following processes is least likely to be occurring in the wound at this stage?
A. Angiogenesis
B. Synthesis of collagen
C. Necrosis of fibroblasts
D. Secretion of matrix metalloproteinases by fibroblasts
E. Proliferation of fibroblasts
Answer: C

Vasculogenesis vs Angiogenesis: Vascu is new. Angi is pre. Vasculogenesis is new vessels developing in situ from existing
mesenchyme. Angiogenesis is vessels develop from sprouting off pre-existing arteries. Fibroblasts are an important cell type in
healing wounds. They typically proliferate in the early phases of wound healing. They release matrix metalloproteinases and these
facilitate in the remodelling of the matrix within the healing wound. Necrosis in a healing wound would be unusual as wounds will
tend to show clinical evidence of angiognesis by this time.

213. A 25 year old women presents with a slowly enlarging mass on the side of the face. Clinical examination demonstrates that
the mass is located in the tail of the parotid gland. There is no evidence of facial nerve involvement. What is the most likely
cause?
A. Sialolithiasis
B. Adenocarcinoma
C. Warthins tumour
D. Oncocytoma
E. Pleomorphic adenoma
Answer: E
Pleomorphic adenomas are the commonest tumours of the parotid gland and are often slow growing, smooth and mobile. Warthins
tumours are typically found in elderly males and are composed of multiple cysts and solid components consisting of lymphoid tissue.
Warthins tumours are most often found in the tail of the parotid gland, but not in 25 year old females, where a pleomorphic adenoma
remains the most likely lesion.

214. Beta-naphthalamine is associated with which of the following malignancies?


A. Lung cancer
B. Bowel cancer
70
C. Bladder cancer
D. Liver cancer
E. Renal cancer
Answer: C
Beta-naphthalamine is used in the rubber industry.
The following factors are associated with the development of bladder cancer: smoking. Occupational: aniline dyes used in printing and
textile industry, rubber manufacture. Schistosomiasis. Drugs: cyclophosphamide

215. A 56 year old man with Wilsons disease presents with right upper quadrant discomfort. An ultrasound scan is performed
and this demonstrates a mass lesion in the right lobe of the liver. What is the most appropriate method of establishing the
underlying diagnosis?
A. PET CT scan
B. Ultrasound guided biopsy
C. Measurement of serum alpha feto protein
D. MRI scan of the liver
E. CT scan of the liver
Answer: C
High AFP + chronic liver inflammation = Hepatocellular carcinoma.
This is likely to be a hepatocellulcar carcinoma. Diagnosis is usually made by AFP measurement (with further imaging depending on
the result). Biopsy should not be performed as it may seed the tumour. Chronic liver diseases such as Wilsons disease (Hepato-
lenticular degeneration) increase the risk.

216. A 55 year old man has suffered from reflux oesophagitis for many years. During a recent endoscopy a biopsy is taken from
the distal oesophagus. The histopathology report indicates that cells are identified with features of coarse chromatin and
abnormal mitoses. The cells are confined to the superficial epithelial layer only. Which of the following accounts for this
process?
A. Metaplasia
B. Apoptosis
C. Autoimmune oesophagitis
D. Dysplasia
E. Infection with Helicobacter pylori
Answer: D
Dysplasia = pre cancerous
Dysplasia tends to develop as a result of prolonged stimulation by precipitants. Removal of these precipitants may possibly reverse
these changes. Replacement of differentiated cells wth another cell type describes metaplasia rather than dysplasia. The absence of
invasion distinguishes this from malignancy.

Dysplasia: Premalignant condition. Disordered growth and differentiation of calls. Alteration in size, shape, and organization of cells.
Features increased abnormal cell growth (increased number of mitoses/abnormal mitoses and cellular differentiation). Underlying
connective tissue is not invaded. Causes include smoking, Helicobacter pylori, Human papilloma virus. Main differences to
metaplasia is that dysplasia is considered to be part of carcinogenesis (pre cancerous) and is associated with a delay in maturation of
cells rather than differentiated cells replacing one another. The absence of invasion differentiates dysplasia from invasive malignancy.
Severe dysplasia with foci of invasion are well recognised.

217. A 63 year old man has a history of claudication that has been present for many years. He is recently evaluated in the clinic
and a duplex scan shows that he has an 85% stenosis of the superficial femoral artery. Two weeks later he presents with a 1
hour history of severe pain in his leg. On examination he has absent pulses in the affected limb and it is much cooler than
the contra-lateral limb. Which process best accounts for this presentation?
A. Thrombosis
B. Embolus
C. Atheroma growth
D. Sub intimal dissection
E. Anaemia
Answer: A
In an existing lesion a complication such as thrombosis is more likely than embolus. These patients should receive heparin and
imaging with duplex scanning. Whilst an early surgical bypass or intra-arterial thrombolysis may be indicated, an embolectomy
should not generally be performed as the lesion is not an embolus and the operation therefore ineffective.

71
Claudication: claudication is a condition in which patients develop pain in a limb during periods of exercise. The underlying disorder
is usually that of arterial insufficiency. Atheroma develops in the arterial wall and once this occludes >50-75% of the lumenal
diameter the supply to metabolising tissues distally may become compromised. The typical claudicant complains of calf pain that is
worse on exercise and relieved by rest. This typical description assumes that the SFA is the site of disease, more proximal disease may
present with other symptoms such as buttock claudication and impotence.
The history is usually a progressive one, patients presenting as an emergency with severe pain, diminished sensation, pallor and absent
pulses have critical limb ischaemia. This may complicate claudication and usually indicates a plaque related complication, such as
thrombosis. Risk factors: Risk factors for claudication include smoking, diabetes and hyperlipidaemia. Diagnosis: Diagnostic work -
up includes measurement of ankle- brachial pressure indices, duplex scanning and formal angiography. Treatment: Those with long
claudication distances, no ulceration or gangrene may be managed conservatively. Patients with rest pain, ulceration or gangrene will
almost always require intervention. All patients should receive an antiplatelet agent and a statin unless there are compelling contra-
indications.

218. The following are true of carcinoid tumours except:


A. When present in the appendix tip and measure less than 2 cm have an excellent prognosis
B. Even when metastatic disease is present it tends to follow a protracted course
C. When present in the appendix body tend to present with carcinoid syndrome even when liver metastases are not present
D. May be imaged using 5 HIAA radionucleotide scanning
E. Advanced appendiceal carcinoids may require right hemicolectomy.
Answer: C

219. During a difficult femoro-popliteal bypass operation the surgeon inadvertently places a clamp across the femoral nerve. It
remains there for most of the procedure. At the end of the operation the nerve is inspected, it is in continuity but has
evidence of being crushed. Which of the following is most likely to occur over the following months?
A. Wallerian degeneration
B. Rapid restoration of neuronal function because the axon itself is intact
C. Normal but delayed neuronal transmission due to disruption of the myelin
D. Absence of neuroma formation
E. None of the above
Answer: A
A neuronal injury such as this will result in Wallerian degeneration even though the nerve remains in continuity. Neuromas may well
form.

220. A 38 year old lady who smokes heavily presents with recurrent episodes of infection in the right breast. On examination
she has an indurated area at the lateral aspect of the nipple areaolar complex. Imaging shows no mass lesions. What is the
most likely diagnosis?
A. Duct ectasia
B. Periductal mastitis
C. Pagets disease of the nipple
D. Mondors disease of the breast
E. Radial scar
Answer: B
Periductal mastitis is common in smokers and may present with recurrent infections. Treatment is with co-amoxyclav. Mondors
disease of the breast is a localised thrombophlebitis of a breast vein.

Duct ectasia: Duct ectasia is a dilatation and shortening of the terminal breast ducts within 3cm of the nipple. It is common and the
incidence increases with age. It typically presents with nipple retraction and occaisionally creamy nipple discharge. It may be
confused with periductal mastitis, which presents in younger women, the vast majority of which are smokers. Periductal mastitis
typically presents with periareolar or sub areolar infections and may be recurrent.
Patients with troublesome nipple discharge may be treated by microdochectomy (if young) or total duct excision (if older).

221. Which of the following statements relating to chronic inflammation is true?


A. Chronic inflammation is mainly secondary to acute inflammation
B. Neutrophils are the predominant cells involved
C. Growth factors are not involved in the process
D. Appendicitis is mainly a form of chronic inflammation
E. Fibrosis is a macroscopic feature
Answer: E

72
Macroscopic features include: Ulcers, Fibrosis, Granulomatous process. It most commonly occurs as a primary event rather than as a
result of acute inflammation.

Theme: Paediatric ano-rectal disorders

A. Ulcerative colitis
B. Juvenile polyps
C. Haemorroids
D. Intussceception
E. Rectal cancer
F. Anal fissure
G. Arteriovenous malformation

Please select the most likely cause for the condition described. Each option may be used once, more than once or not at all.

222. A 4 year old boy is brought to the clinic. He gives a history of difficult, painful defecation with bright red rectal bleeding.
Answer: Anal fissure
Painful rectal bleeding in this age group is typically due to a fissure. Treatment should include stool softeners and lifestyle advice.

223. A 2 year old has a history of rectal bleeding. The parents notice that post defecation, a cherry red lesion is present at the
anal verge.
Answer: Juvenile polyps
These lesions are usually hamartomas and this accounts for the colour of the lesions. Although the lesions are not themselves
malignant they serve as a marker of an underlying polyposis disorder.

224. A 12 year old is brought to the colorectal clinic with a history of rectal bleeding, altered bowel habit, weight loss and
malaise. Abdominal examination is normal.
Answer: Ulcerative colitis
The systemic features in the history are strongly suggestive of inflammatory bowel disease rather than the other causes.

Paediatric proctology
Children may present with altered bowel habit and/ or rectal bleeding. Classical haemorroidal disease is relatively rare in children.
Painful bright red rectal bleeding is much more common since constipation is a relatively common childhood disorder. The hard stool
causes a tear of the ano-rectal mucosa with subsequent fissure. The pain from the fissure must be addressed promptly or the child will
delay defecation and this fissure will worsen. Inflammatory bowel disease may present in a similar pattern in paediatric practice with
altered bowel habit (usually diarrhoea) and bleeding. Systemic features may be present and investigation with an endoscopy may be
required. Children with intussceception usually present at a relatively young age and the history is usually one of colicky abdominal
pain, together with a mass on clinical examination.The often cited red current jelly type stool is a rare but classical feature.
Juvenile polyps may occur as part of the familial polyposis coli syndromes. The lesions, which are hamartomas, are often cherry red if
they protrude externally.

225. You review a 42-year-old woman 8 months following a renal transplant for focal segmental glomerulosclerosis. She is on a
combination of tacrolimus, mycophenolate, and prednisolone. She has now presented with a five day history of feeling
generally unwell with jaundice, fatigue and arthralgia. On examination she has jaundice, widespread lymphadenopathy
and hepatomegaly. What is the most likely diagnosis?

A. Hepatitis C
B. Epstein-Barr virus
C. HIV
D. Hepatitis B
E. Cytomegalovirus
Answer: B
Post transplant complications: CMV: 4 weeks to 6 months post transplant. EBV: post transplant lymphoproliferative disease. > 6
months post transplant. Post transplant lymphoproliferative disorder is most commonly associated with Epstein-Barr virus. It typically
occurs 6 months post transplant and is associated with high dose immunosupressant therapy. Remember cytomegalovirus presents
within the first 4 weeks to 6 months post transplant.

73
Renal transplant:HLA typing and graft failure: The human leucocyte antigen (HLA) system is the name given to the major
histocompatibility complex (MHC) in humans. It is coded for on chromosome 6.
Some basic points on the HLA system: Class 1 antigens include A, B and C. Class 2 antigens include DP,DQ and DR. When HLA
matching for a renal transplant the relative importance of the HLA antigens are as follows DR > B > A

Graft survival: 1 year = 90%, 10 years = 60% for cadaveric transplants. 1 year = 95%, 10 years = 70% for living-donor transplants
Post-op problems: ATN of graft. Vascular thrombosis. Urine leakage. UTI. Hyperacute acute rejection: Due to antibodies against
donor HLA type 1 antigens. Rarely seen due to HLA matching. Acute graft failure (< 6 months): Usually due to mismatched HLA.
Other causes include cytomegalovirus infection. Management: give steroids, if resistant use monoclonal antibodies. Causes of
chronic graft failure (> 6 months): Chronic allograft nephropathy. Ureteric obstruction. Recurrence of original renal disease (MCGN
> IgA > FSGS)

226. Which of the following processes facilitates phagocytosis?


A. Apoptosis
B. Opsonisation
C. Proteolysis
D. Angiogenesis
E. Necrosis
Answer: B
Opsonisation will facilitate phagocytosis. The micro-organism becomes coated with antibody, C3b and certain acute phase proteins.
The macrophages and neutrophils have up regulation of phagocytic cell surface receptors in these circumstances, a process mediated
by pro-inflammatory cytokines. These cells then engulf the micro organism.

Phagocytosis: Ingestion of pathogens or foreign materials by cells. First step is opsonisation whereby the organism is coated by
antibody. Second step is adhesion to cell surface. Third step is pseudopodial extension to form a phagocytic vacuole. Lysosomes fuse
with vacuole and degrade contents.

227. Features which are evaluated for the grading of breast cancer include all the following, except:
A. Tubule formation
B. Mitoses
C. Nuclear pleomorphism
D. Tumour necrosis
E. Coarse chromatin
Answer: D
The necrosis of a tumour may be suggestive of a high grade tumour which has out grown its blood supply. However, the grading of
breast cancer which classically follows the Bloom -Richardson grading model will tend to favor nuclear appearances (which include
mitoses, coarse chromatin and pleomorphism). Tubule formation is an important marker of the degree of differentiation with formation
of tubular structures being associated with well differentiated tumours.

Tumour grading: Tumours may be graded according to their degree of differentiation, mitotic activity and other features. Grade 1
tumours are the most differentiated and grade 4 the least. The assessment is subjective, in most cases high grade equates to poor
prognosis, or at least rapid growth. Tumours of glandular epithelium will tend to arrange themselves into acinar type structures
containing a central lumen. Well differentiated tumours may show excellent acinar formation and poorly differentiated tumours simply
clumps of cells around a desmoplastic stroma. Sometimes tumours demonstrate mucous production without evidence of acinar
formation. Since mucous production is evidence of a glandular function such tumours are often termed mucinous adenocarcinoma.
Squamous cell tumours will typically produce structures resembling epithelial cell components. Well differentiated tumours may also
produce keratin (depending upon tissue of origin).

228. Which of the following is associated with poor wound healing?


A. Jaundice
B. Patients taking carbamazepine
C. General anaesthesia using thiopentone
D. General anaesthesia using ketamine
E. Multiple sclerosis
Answer: A
Mnemonic to remember factors affecting wound healing: DID NOT HEAL: D iabetes, I nfection, irradiation, D rugs eg steroids,
chemotherapy, N utritional deficiencies (vitamin A, C & zinc, manganese), Neoplasia, O bject (foreign material), T issue necrosis, H
ypoxia, E xcess tension on wound, A nother wound, L ow temperature, Liver jaundice
Multiple sclerosis is associated with pressure sores, however the cellular healing process is not affected.
74
229. A 55 year old man undergoes a colonoscopy and a colonic polyp is identified. It has a lobular appearance and is located on
a stalk in the sigmoid colon. Which of the processes below best accounts for this disease?
A. Apoptosis
B. Metaplasia
C. Dysplasia
D. Calcification
E. Degeneration
Answer: C
Most colonic polyps described above are adenomas. These may have associated dysplasia. The more high grade the dysplasia the
greater the level of clinical concern.

230. 56 year old lady has just undergone a colonoscopy and a lesion was identified in the caecum. The histology report states
that biopsies have been taken from a sessile serrated polyp with traditional features? What is the best management option?
A. Perform a right hemicolectomy
B. List the patient for colonoscopic polypectomy
C. Discharge the patient
D. Re scope the patient in 6 months
E. Re scope the patient at 3 years
Answer: B
These polyps represent an alternative pathway to progression to carcinoma and may be diagnostically confused with hyperplastic
polyps. Hyperplastic polyps are more common in the left colon and confer no increased risk. SSA's are more common in the right
colon and are usually larger. Those with "traditional features" on histology have dysplasia with increased risk of malignant
transformation.

231. A 30 year old male presents with gynaecomastia. Clinically he is noted to have a nodule in the left testis. The most likely
diagnosis is:
A. Oestrogen abuse
B. Seminoma with syncytiotrophoblast giant cells
C. Teratoma
D. Choriocarcinoma
E. Leydig cell tumour
Answer: E
Leydig cell tumours are rare testicular sex cord stromal tumours (which also include sertoli cell tumours) which are associated with
hormonal activity. Patients with Leydig cell tumours may present with gynaecomastia before they notice testicular enlargement.
Majority are benign. Histology: eosinophilic cells in columns

232. Which of the following features are not typical of Crohns disease?
A. Complex fistula in ano
B. Small bowel strictures
C. Skip lesions
D. 'Rose thorn ulcers' on barium studies
E. Pseudopolyps on colonoscopy
Answer: E
Pseudopolyps are a feature of ulcerative colitis and occur when there is severe mucosal ulceration. The remaining islands of mucosa

233. A 32 year old lady presents with a 1.5cm pigmented lesion on her back. The surgeon is concerned that this may be a
melanoma. What is the most appropriate course of action?
A. 2mm punch biopsy from the centre of the lesion
B. 4mm punch biopsy from the centre of the lesion
C. Wide excision of the lesion with 3cm margins
D. Excisional biopsy of the lesion
E. Wide excision of the lesion with 1cm margins
Answer: D
Suspicious naevi should NOT be partially sampled as histological interpretation is severely compromised. Complete excision is
mandatory where lesions fulfil diagnostic criteria. However, wide excision for margins may be deferred until definitive histology is
available. Lesions that are suspicious for melanoma should be excised with complete margins. Radical excision is not routinely
undertaken for diagnostic purposes and therefore if subsequent histopathological assessment determines that the lesion is a melanoma
a re-exicision of margins may be required. Incisional punch biopsies of potential melanomas makes histological interpretation
difficult and is best avoided.
75
234. A 70 year old male presents with painless frank haematuria. Clinical examination is unremarkable. Routine blood tests
reveal a haemoglobin of 18g/dl but are otherwise normal. What is the most likely underlying diagnosis?
A. Squamous cell carcinoma of the bladder
B. Adenocarcinoma of the prostate
C. Adenocarcinoma of the kidney
D. Wilms tumour
E. Transitional cell carcinoma of the renal pelvis
Answ

er: C

Theme from April 2012 Exam. Polycythaemia is a recognised feature of renal cell carcinoma. Wilms tumours most commonly occur in
children.

Haematuria: Causes of haematuria. Trauma: Injury to renal tract. Renal trauma commonly due to blunt injury (others penetrating
injuries). Ureter trauma rare: iatrogenic. Bladder trauma: due to RTA or pelvic fractures. Infection: Remember TB. Malignancy:
Renal cell carcinoma (remember paraneoplastic syndromes): painful or painless. Urothelial malignancies: 90% are transitional cell
carcinoma, can occur anywhere along the urinary tract. Painless haematuria. Squamous cell carcinoma and adenocarcinoma: rare
bladder tumours. Prostate cancer. Penile cancers: SCC. Renal disease: Glomerulonephritis. Stones:Microscopic haematuria common.
Structural abnormalities: Benign prostatic hyperplasia (BPH) causes haematuria due to hypervascularity of the prostate gland.
Cystic renal lesions e.g. polycystic kidney disease. Vascular malformations. Renal vein thrombosis due to renal cell carcinoma.
Coagulopathy: Causes bleeding of underlying lesions. Drugs: Cause tubular necrosis or interstitial nephritis: aminoglycosides,
chemotherapy. Interstitial nephritis: penicillin, sulphonamides, and NSAIDs. Anticoagulants. Benign: Exercise.
Gynaecological:Endometriosis: flank pain, dysuria, and haematuria that is cyclical. Iatrogenic: Catheterisation. Radiotherapy;
cystitis, severe haemorrhage, bladder necrosis. Pseudohaematuri

Theme: Thyroid neoplasms

A. Follicular carcinoma
B. Follicular adenoma
C. Papillary carcinoma
D. Papillary adenoma
E. Anaplastic carcinoma
F. Medullary carcinoma

Please select the most likely underlying diagnosis for the thyroid masses described. Each option may be used once, more than once or
not at all.

235. A 78 year old lady presents to the surgical clinic with symptoms of both dysphagia and dyspnoea. On examination there is
a large mass in the neck that moves on swallowing. CT scanning of the neck shows a locally infiltrative lesion arising from
the thyroid and invading the strap muscles.
Answer: Anaplastic carcinoma
Marked local invasion is a feature of anaplastic carcinoma. These tumours are more common in elderly females.

236. A 25 year old female presents with a lump in her neck. On examination she has a discrete nodule in the right lobe of the
thyroid. A fine needle aspirate shows papillary cells. An adjacent nodule is also sampled which shows similar well
differentiated papillary cells.
Answer: Papillary carcinoma
Multifocal disease is a recognised feature of papillary lesions. Papillary adenomas are not really recognised and most well
differentiated lesions are papillary carcinomas.

237. A 45 year old man presents with a fracture of his right humerus. On examination there is a lytic lesion of the proximal
humerus and a mass in the neck, this moves on swallowing.
Answer: Follicular carcinoma
Follicular carcinomas are a recognised source of bone metastasis. Up to 60% will show vascular invasion histologically.

238. Which of the following lesions is most closely associated with Barretts oesophagus?
76
A. Squamous cell carcinoma
B. Gastro intestinal stromal tumours
C. Carcinoid tumours
D. Leiomyosarcoma
E. Adenocarcinoma
Answer: E
Barretts oesphagus is most closely associated with adenocarcinoma, and it confers at 30 fold increased risk of developing the
condition.

239. A 56 year old man presents with epigastric discomfort and episodes of migratory thrombophlebitis. On examination he is
mildly jaundiced. A CT scan shows peri hilar lymphadenopathy and a mass in the pancreatic head. Which of the
following is the most likely underlying diagnosis?
A. Squamous cell carcinoma of the pancreas
B. Adenocarcinoma of the pancreas
C. Insulinoma
D. Glucagonoma
E. Gastrinoma
Answer: B
Adenocarcinoma of the pancreas is the most likely diagnosis and migratory thrombophlebitis is associated with the condition.
Squamous cells carcinoma is extremely uncommon in the pancreas. Gastrinoma are extremely rare and thus not the most likely
diagnosis.

240. A 22 year old man presents with symptoms of headache, lethargy and confusion. On examination he is febrile and has a
right sided weakness. A CT scan shows a ring enhancing lesion affecting the left motor cortex. Which of the following is the
most likely diagnosis?
A. Arteriovenous malformation
B. Cerebral abscess
C. Herpes simplex encephalitis
D. Metastatic renal adenocarcinoma
E. Glioblastoma multiforme
Answer: B
The combination of rapidly progressive neurology, fever and headache is highly suggestive of cerebral abscess. CT scanning will
show a ring enhancing lesion because the intravenous contrast cannot penetrate the centre of the abscess cavity. HSV encephalitis does
not produce ring enhancing lesions.

Brain abscess: CNS abscesses may result from a number of causes including, extension of sepsis from middle ear or sinuses, trauma
or surgery to the scalp, penetrating head injuries and embolic events from endocarditis. The presenting symptoms will depend upon
the site of the abscess (those in critical areas e.g.motor cortex) will present earlier. Abscesses have a considerable mass effect in the
brain and raised intra cranial pressure is common. Although fever, headache and focal neurology are highly suggestive of a brain
abscess the absence of one or more of these does not exclude the diagnosis, fever may be absent and even if present, is usually not the
swinging pyrexia seen with abscesses at other sites. Assessment of the patient includes imaging with CT scanning. Treatment is
usually surgical, a craniotomy is performed and the abscess cavity debrided. The abscess may reform because the head is closed
following abscess drainage.

241. A 56 year old man from Egypt has suffered from recurrent attacks of haematuria for many years. He presents with
suprapubic discomfort and at cystoscopy is found to have a mass lesion within the bladder. What is the most likely
diagnosis?
A. Squamous cell carcinoma
B. Transitional cell papilloma
C. Adenocarcinoma
D. Leiomyosarcoma
E. Rhabdomyosarcoma
Answer: A
In Egypt Schistosomiasis is more common than in the UK and may cause recurrent episodes of haematuria. In those affected with the
condition who develop a bladder neoplasm, an SCC is the most common type.

242. A newborn infant is noted to have a unilateral cleft lip only. What is the most likely explanation for this process?
A. Incomplete fusion of the second branchial arch
B. Incomplete fusion of the nasolabial muscle rings
77
C. Incomplete fusion of the first branchial arch
D. Incomplete fusion of the third branchial arch
E. Incomplete fusion of the secondary palate
Answer: B
Theme from April 2012 Exam: Unilateral isolated cleft lip represents a failure of nasolabial ring fusion. It is not related to branchial
arch fusion. Arch disorders have a far more profound phenotype and malformation sequences.

Cleft lip and palate: Cleft lip and palate are the most common congenital deformity affecting the orofacial structures. Whilst they
may be an isolated developmental malformation they are also a recognised component of more than 200 birth defects. The incidence is
as high as 1 in 600 live births. The commonest variants are: Isolated cleft lip (15%), Isolated cleft palate (40%). Combined cleft lip
and palate (45%). The aetiology of the disorder is multifactorial; both genetic (affected first degree relative increases risk) and
environmental factors play a role.
Cleft lip: Cleft lip occurs as a result of disruption of the muscles of the upper lip and nasolabial region. These muscles comprise a
chain of muscles viz; nasolabial, bilabial and labiomental. Defects may be unilateral or bilateral.
Cleft palate: The primary palate consists of all anatomical structures anterior to the incisive foramen. The seconday palate lies more
posteriorly and is sub divided into the hard and soft palate. Cleft palate occurs as a result of non fusion of the two palatine shelves.
Both hard and soft palate may be involved. Complete cases are associated with complete separation of the nasal septum and vomer
from the palatine processes.
Treatment: Surgical reconstruction is the mainstay of management. The procedures are planned according to the extent of
malformation and child age. Simple defects are managed as a single procedure. Complex malformations are usually corrected in
stages. Affected individuals have a higher incidence of hearing and speech problems.

243. A 72 year old woman with back pain and chronic renal failure has the following results: Ca2+ 2.03 (2.15-2.55 mmol/l);
Parathyroid hormone 10.4 (1-6.5 pmol/l); Phosphate 0.80 (0.6-1.25 mmol/l). What is the most likely diagnosis?
A. Hypoparathyroidism
B. Primary hyperparathyroidism
C. Secondary hyperparathyroidism
D. Tertiary hyperparathyroidism
E. Pseudohypoparathyroidism
Answer: C
In relation to secondary hyperparathyroidism; there is a HIGH PTH and the Ca2+ is NORMAL or LOW. In secondary
hyperparathyroidism there in hyperplasia of the parathyroid glands in response to chronic hypocalcaemia (or hyperphosphataemia)
and is a normal physiological response. Calcium is restored from bone, kidneys and the gastrointestinal system.

Theme: Pharyngitis

A. Infectious mononucleosis

B. Acute bacterial tonsillitis

C. Quinsy

D. Lymphoma

E. Diptheria

Please select the most likely underlying cause for the following patients presenting with pharyngitis. Each option may be used once,
more than once or not at all.

244. An 8 year old child presents with enlarged tonsils that meet in the midline and are covered with a white film that bleeds
when you attempt to remove it. He is pyrexial but otherwise well.
Answer: Acute bacterial tonsillitis
Theme from April 2012 Exam: In acute tonsillitis the tonsils will often meet in the midline and may be covered with a membrane.
Individuals who are systemically well are unlikely to have diptheria.

245. A 10 year old child presents with enlarged tonsils that meet in the midline. Oropharyngeal examination confirms this
finding and you also notice peticheal haemorrhages affecting the oropharynx. On systemic examination he is noted to
have splenomegaly.

78
Answer: Infectious mononucleosis
A combination of pharyngitis and tonsillitis is often seen in glandular fever. Antibiotics containing penicillin may produce a rash
when given in this situation, leading to a mistaken label of allergy.

246. A 19 year old man has had a sore throat for the past 5 days. Over the past 24 hours he has notices increasing and severe
throbbing pain in the region of his right tonsil. He is pyrexial and on examination he is noted to have a swelling of this
area.
Answer: Quinsy
Unilateral swelling and fever is usually indicative of quinsy. Surgical drainage usually produces prompt resolution of symptoms.

Acute tonsillitis: Characterised by pharyngitis, fever, malaise and lymphadenopathy. Over half of all cases are bacterial with
Streptococcus pyogenes the most common organism. The tonsils are typically oedematous and yellow or white pustules may be
present. Infectious mononucleosis may mimic the condition.Treatment with penicillin type antibiotics is indicated for bacterial
tonsillitis. Bacterial tonsillitis may result in local abscess formation (quinsy)

247. A 4 year old boy is brought to the clinic by his mother who has noticed a small lesion at the external angle of his eye. On
examination there is a small cystic structure which has obviously been recently infected. On removal of the scab, there is
hair visible within the lesion. What is the most likely diagnosis?
A. Dermoid cyst
B. Desmoid cyst
C. Sebaceous cyst
D. Epidermoid cyst
E. Keratoacanthoma
Answer: A
Dermoid cysts occur at sites of embryonic fusion and may contain multiple cell types. They occur most often in children.
The lesion is unlikely to be a desmoid cyst as these are seldom located either at this site or in this age group. In addition they do not
contain hair. Sebaceous cysts will usually have a punctum and contain a cheesy material. Epidermoid cysts contain keratin plugs.

Dermoid cysts: A cutaneous dermoid cyst may develop at sites of embryonic developmental fusion. They are most common in the
midline of the neck, external angle of the eye and posterior to the pinna of the ear. They typically have multiple inclusions such as hair
follicles that bud out from its walls. They may develop at other sites such as the ovary and in these sites are synonymous with
teratomas. A desmoid tumour is a completely different entity and may be classified either as low grade fibrosarcomas or non
aggressive fibrous tumours. They commonly present as large infiltrative masses. They may be divided into abdominal, extra
abdominal and intra abdominal. All types share the same biological features. Extra abdominal desmoids have an equal sex distribution
and primarily arise in the musculature of the shoulder, chest wall, back and thigh. Abdominal desmoids usually arise in the
musculoaponeurotic structures of the abdominal wall. Intra abdominal desmoids tend to occur in the mesentery or pelvic side walls
and occur most frequently in patients with familial adenomatous polyposis coli syndrome.

248. Which of the following tumours are most likely to give rise to para-aortic nodal metastasis early?
A. Ovarian
B. Bladder
C. Rectal
D. Caecal
E. Cervical
Answer: A
Theme from 2009 Exam. Ovarian tumours are supplied by the ovarian vessels, these branch directly from the aorta. The cervix drains to
the internal and external iliac nodes.

Para-aortic lymphadenopathy: Organ sites that may metastasise (early) to the para-aortic lymph nodes: Testis. Ovary. Uterine
fundus. Many other organs may result in para-aortic nodal disease. However, these deposits will represent a much later stage of the
disease, since other nodal stations are involved earlier.

249. Which of the following statements relating to malignant mesothelioma is false?


A. It may be treated by extrapleural pneumonectomy.
B. It is linked to asbestos exposure.
79
C. It is linked to cigarette smoking independent of asbestos exposure.
D. It may occur intra abdominally.
E. It is relatively resistant to radiotherapy
Answer: C
It is not linked to cigarette smoking. When identified at an early stage a radical resection is the favored option. Radiotherapy is often
given perioperatively. However, it is not a particularly radiosensitive tumour. Combination chemotherapy gives some of the best
results and most regimes are cisplatin based.

Theme: Thyroid disorders

A. Sick euthyroid
B. Hyperthyroidism
C. Hypothyroidism
D. Normal euthyroid
E. Anxiety state
F. Factitious hyperthyroidism

For each of the scenarios please match the scenario with the most likely underlying diagnosis. Each answer may be used once, more
than once or not at all.

250. A 33 year old man is recovering following a protracted stay on the intensive care unit recovering from an anastomotic leak
following a difficult Trans hiatal oesophagectomy. His progress is slow, and the intensive care doctors receive the following
thyroid function test results: TSH 1.0 u/L, Free T48, T3 1.0 (1.2-3.1 normal)
Answer: Sick euthyroid
Theme from April 2012 Exam
Sick euthyroid syndrome is caused by systemic illness. With this, the patient may have an apparently low total and free T4 and T3,
with a normal or low TSH. Note that the levels are only mildly below normal.

251. A 28 year old female presents to the general practitioner with symptoms of fever and diarrhoea. As part of her diagnostic
evaluation the following thyroid function tests are obtained: TSH< 0.01, Free T4 30, T3 4.0
Answer: Hyperthyroidism
The symptoms are suggestive of hyperthyroidism. This is supported by the abnormal blood results; suppressed TSH with an
elevated T3 and T4.

252. A 19 year old lady presents with palpitations. The medical officer takes a blood sample for thyroid function tests. The
following results are obtained:TSH > 6.0, Free T4 20, T3 2.0
Answer: Hypothyroidism
An elevated TSH with normal T4 indicates partial thyroid failure. This is caused by Hashimotos, drugs (lithium, antithyroids)
and dyshormogenesis.

Hyperthyroidism
Causes of hyperthyroidism include: Diffuse toxic goitre (Graves Disease). Toxic nodular goiter. Toxic nodule. Rare causes
Graves disease: Graves disease is characterised by a diffuse vascular goitre that appears at the same time as the clinical
manifestations of hyperthyroidism. It is commonest in younger females and may be associated with eye signs. Thyrotoxic symptoms
will predominate. Up to 50% of patients will have a familial history of autoimmune disorders. The glandular hypertrophy and
hyperplasia occur as a result of the thyroid stimulating effects of the TSH receptor antibodies.
Toxic nodular goiter: In this disorder the goitre is present for a long period of time prior to the development of clinical symptoms. In
most goitres the nodules are inactive and in some cases it is the internodular tissue that is responsible for the goitre.
Toxic nodule: Overactive, autonomously functioning nodule. It may occur as part of generalised nodularity or be a true toxic
adenoma. The TSH levels are usually low as the autonomously functioning thyroid tissue will exert a negative feedback effect.

Clinical features: Symptoms: Lethargy. Emotionally labile. Heat intolerance. Weight loss. Excessive appetite. Palpitations. Signs:
Tachycardia. Agitation. Hot, moist palms. Exopthalmos. Thyroid goitre and bruit. Lid lag/retraction. Diagnosis: The most sensitive
test for diagnosing hyperthyroidism is plasma T3 (which is raised). Note in hypothyroidism the plasma T4 and TSH are the most
sensitive tests. A TSH level of <0.5U/L suggests hyperthyroidism. TSH receptor antibodies may be tested for in the diagnosis of
Graves. Treatment: First line treatment for Graves disease is usually medical and the block and replace regime is the favored option.
Carbimazole is administered at higher doses and thyroxine is administered orally. Patient are maintained on this regime for between 6

80
and 12 months. Attempts are then made to wean off medication. Where relapse then occurs the options are between ongoing medical
therapy, radioiodine or surgery.

253. Which of the following conditions is least likely to exhibit the Koebner phenomenon?
A. Vitiligo
B. Molluscum contagiosum
C. Lichen planus
D. Psoriasis
E. Lupus vulgaris
Answer: E
Lupus vulgaris is not associated with the Koebner phenomenon.

Koebner phenomenon: The Koebner phenomenon describes skin lesions which appear at the site of injury. It is seen in: Psoriasis.
Vitiligo. Warts. Lichen planus. Lichen sclerosus. Molluscum contagiosum

254. Which of the tumour types listed below is found most frequently in a person with aggressive fibromatosis?
A. Medullary thyroid cancer
B. Basal cell carcinoma of the skin
C. Desmoid tumours
D. Dermoid tumours
E. Malignant melanoma
Answer: C
Aggressive fibromatosis: Aggressive fibromatosis is a disorder consisting of desmoid tumours, which behave in a locally aggressive
manner. Desmoid tumours may be identified in both abdominal and extra-abdominal locations. Metastatic disease is rare. The main
risk factor (for abdominal desmoids) is having APC variant of familial adenomatous polyposis coli. Most cases are sporadic.
Treatment is by surgical excision.

255. A 20 year old man is suspected of having an inflamed Meckels diverticulum. At which of the following locations is it most
likely to be found?
A. Approximately 60 cm distal to the ileo-caecal valve
B. Approximately 60 cm proximal to the ileocaecal valve
C. Approximately 200cm distal to the ileocaecal valve
D. Approximately 200cm proximal to the ileocaecal valve
E. 50cm distal to the DJ flexure
Answer: B
256. A 43 year old lady presents with severe chest pain. Investigations demonstrate a dissecting aneurysm of the ascending aorta
which originates at the aortic valve. What is the optimal long term treatment?
A. Endovascular stent
B. Medical therapy with beta blockers
C. Medical therapy with ACE inhibitors
D. Sutured aortic repair
E. Aortic root replacement
Answer: E
Proximal aortic dissections are generally managed with surgical aortic root replacement. The proximal origin of the dissection together
with chest pain (which may occur in all types of aortic dissection) raises concerns about the possibility of coronary ostial involvement
(which precludes stenting). There is no role for attempted suture repair in this situation.

257. A 45 year old lady presents with a pathological fracture of her femoral shaft. She is a poor historian, but it transpires that
she underwent a thyroidectomy 1 year previously. She has no other illness or co-morbidities. What is the most likely
underlying diagnosis?
A. Hyperparathyroidism
B. Metastatic papillary carcinoma of the thyroid
C. Metastatic medullary carcinoma of the thyroid
D. Metastatic follicular carcinoma of the thyroid
E. None of the above
Answer: D
Follicular carcinomas are a recognised cause of bone metastasis. Papillary lesions typically spread via the lymphatics.
81
Theme: Cardiac murmurs

A. Pulmonary stenosis
B. Mitral regurgitation
C. Tricuspid regurgitation
D. Aortic stenosis
E. Mitral stenosis
F. Aortic sclerosis

What is the most likely cause of the cardiac murmur in the following patients? Each option may be used once, more than once or not
at all.

258. A 35 year old Singaporean female attends a varicose vein pre operative clinic. On auscultation a mid diastolic murmur is
noted at the apex. The murmur is enhanced when the patient lies in the left lateral position.
Answer: Mitral stenosis
Theme from September 2011 exam
A mid diastolic murmur at the apex is a classical description of a mitral stenosis murmur. The most common cause is rheumatic
heart disease. Complications of mitral stenosis include atrial fibrillation, stroke, myocardial infarction and infective endocarditis.

259. A 22 year old intravenous drug user is found to have a femoral abscess. The nursing staff contact the on call doctor as the
patient has a temperature of 39oC. He is found to have a pan systolic murmur loudest at the left sternal edge at the 4th
intercostal space.
Answer: Tricuspid regurgitation
Intravenous drug users are at high risk of right sided cardiac valvular endocarditis. The character of the murmur fits with a
diagnosis of tricuspid valve endocarditis.

260. An 83 year old woman is admitted with a left intertrochanteric neck of femur fracture. On examination the patient is
found to have an ejection systolic murmur loudest in the aortic region. There is no radiation of the murmur to the carotid
arteries. Her ECG is normal.
Answer: Aortic sclerosis
The most likely diagnosis is aortic sclerosis. The main differential diagnosis is of aortic stenosis, however as there is no radiation
of the murmur to the carotids and the ECG is normal, this is less likely.

Cardiac murmurs: Ejection systolic: Aortic stenosis. Pulmonary stenosis, HOCM. ASD, Fallot's. Pan-systolic: Mitral regurgitation.
Tricuspid regurgitation. VSD. Late systolic:Mitral valve prolapse. Coarctation of aorta. Early diastolic: Aortic regurgitation.
Graham-Steel murmur (pulmonary regurgitation). Mid diastolic: Mitral stenosis. Austin-Flint murmur (severe aortic regurgitation)

261. A 55 year old lady presents with discomfort in the right breast. On clinical examination a small lesion is identified and
clinical appearances suggest fibroadenoma. Imaging confirms the presence of a fibroadenoma alone. A core biopsy is
taken, this confirms the presence of the fibroadenoma. However, the pathologist notices that a small area of lobular
carcinoma in situ is also present in the biopsy. What is the best management?
A. Whole breast irradiation
B. Simple mastectomy
C. Mastectomy and sentinal lymph node biopsy
D. Wide local excision and sentinel lymph node biopsy
E. Breast MRI scan
Answer: E
Lobular carcinoma in situ has a low association with invasive malignancy. It is seldom associated with microcalcification and
therefore MRI is the best tool for determining disease extent. Resection of in situ disease is not generally recommended and most
surgeons would simply pursue a policy of close clinical and radiological follow up.

Lobular carcinoma of the breast: Lobular breast cancers are less common than their ductal counterparts. They typically present
differently, the mass is usually more diffuse and less obvious on the usual imaging modalities of ultrasound and mammography. This
is significant since the disease may be understaged resulting in inadequate treatment when wide local excision is undertaken.
In women with invasive lobular carcinoma it is usually safest to perform an MRI scan of the breast, if breast conserving surgery is
planned. Lobular carcinomas are also more likely to be multifocal and metastasise to the contralateral breast. Lobular carcinoma in

82
situ is occasionally diagnosed incidentally on core biopsies. Unlike DCIS, lobular carcinoma in situ is far less strongly associated with
foci of invasion and is usually managed by close monitoring.

262. Which one of the following may be associated with an increased risk of venous thromboembolism?
A. Diabetes
B. Cannula
C. Hyperthyroidism
D. Tamoxifen
E. Amiodarone
Answer: D

Consider thromboembolism in breast cancer patients on tamoxifen!

Venous thromboembolism: risk factors: Common predisposing factors include malignancy, pregnancy and the period following an
operation. The comprehensive list below is partly based on the 2010 SIGN venous thromboembolism (VTE) guidelines:
General: increased risk with advancing age. Obesity. Family history of vte. Pregnancy (especially puerperium). Immobility.
Hospitalisation. Anaesthesia. Central venous catheter: femoral >> subclavian

Underlying conditions: Malignancy. Thrombophilia: e.g. Activated protein c resistance, protein c and s deficiency. Heart failure.
Antiphospholipid syndrome. Behcet's. Polycythaemia. Nephrotic syndrome. Sickle cell disease. Paroxysmal nocturnal
haemoglobinuria. Hyperviscosity syndrome. Homocystinuria

Medication: combined oral contraceptive pill: 3rd generation more than 2nd generation. Hormone replacement therapy. Raloxifene
and tamoxifen. Antipsychotics (especially olanzapine) have recently been shown to be a risk factor

SIGN also state that the following are risk factors for recurrent VTE: Previous unprovoked VTE. Male sex. Obesity. Thrombophilias

263. A 42 year old lady is investigated for symptoms of irritability and altered bowel habit. On examination she is noted to have a
smooth enlargement of the thyroid gland. As part of her investigations thyroid function tests are requested, these are as
follows: TSH: 0.1 mug/l, Free T4:35 pmol/l. The most likely underlying diagnosis is:
A. Multinodular goitre
B. Follicular carcinoma of the thyroid gland
C. Graves disease
D. Pregnancy
E. None of the above
Answer: C
TSH receptor antibodies will cause stimulation of the thyroid to synthesise T4. However, this will have a negative feedback effect on
the pituitary causing decrease in TSH levels.
Where hyperthyroidism occurs secondary to pregnancy the TSH is typically elevated.

Thyroid disease: Patients may present with a number of different manifestations of thyroid disease. They can be broadly sub
classified according to whether they are euthyroid or have clinical signs of thyroid dysfunction. In addition it needs to be established
whether they have a mass or not.
Assessment: History: Examination including USS. If a nodule is identified then it should be sampled ideally via an image guided fine
needle aspiration. Radionucleotide scanning is of limited use

Thyroid Tumours: Papillary carcinoma. Follicular carcinoma. Anaplastic carcinoma. Medullary carcinoma. Lymphoma's

Multinodular goiter: One of the most common reasons for presentation. Provided the patient is euthyroid and asymptomatic and no
discrete nodules are seen, they can be reassured. In those with compressive symptoms surgery is required and the best operation is a
total thyroidectomy. Sub total resections were practised in the past and simply result in recurrent disease that requires a difficult
revisional resection.

Endocrine dysfunction: In general these patients are managed by physicians initially. Surgery may be offered alongside radio iodine
for patients with Graves disease that fails with medical management or in patients who would prefer not to be irradiated (e.g. pregnant
women).Patients with hypothyroidism do not generally get offered a thyroidectomy. Sometimes people inadvertently get offered
resections during the early phase of Hashimotos thyroiditis, however, with time the toxic phase passes and patients can simply be
managed with thyroxine.
83
Complications following surgery: Anatomical such as recurrent laryngeal nerve damage. Bleeding. Owing to the confined space
haematoma's may rapidly lead to respiratory compromise owing to laryngeal oedema. Damage to the parathyroid glands resulting in
hypocalcaemia.

264. Which of the following is not a risk factor for developing tuberculosis?
A. Gastrectomy
B. Solid organ transplantation with immunosupression
C. Intravenous drug use
D. Haematological malignancy
E. Amiodarone
Answer: E
Risk factors for developing active tuberculosis include: Silicosis. Chronic renal failure. Hiv positive. Solid organ transplantation with
immunosuppression. Intravenous drug use. Haematological malignancy. Anti-tnf treatment. Previous gastrectomy

Tuberculosis: is an infection caused by Mycobacterium tuberculosis that most commonly affects the lungs. Understanding the
pathophysiology of TB can be difficult - the key is to differentiate between primary and secondary disease. Primary tuberculosis: A
non-immune host who is exposed to M. tuberculosis may develop primary infection of the lungs. A small lung lesion known as a
Ghon focus develops. The Ghon focus is composed of tubercle-laden macrophages. The combination of a Ghon focus and hilar lymph
nodes is known as a Ghon complex. In immunocompotent people the intially lesion usually heals by fibrosis. Those who are
immunocompromised may develop disseminated disease (miliary tuberculosis). Secondary (post-primary) tuberculosis
If the host becomes immunocompromised the initial infection may become reactivated. Reactivation generally occurs in the apex of
the lungs and may spread locally or to more distant sites. Possible causes of immunocomprise include: immunosuppressive drugs
including steroids. HIV. Malnutrition

The lungs remain the most common site for secondary tuberculosis. Extra-pulmonary infection may occur in the following areas:
central nervous system (tuberculous meningitis - the most serious complication). Vertebral bodies (Pott's disease). Cervical lymph
nodes (scrofuloderma). Renal. Gastrointestinal tract

265. What is the most common presentation of a parotid gland tumour?


A. Parapharyngeal mass
B. Mass at anterior border of masseter
C. Mass inferior to the angle of the mandible
D. Mass behind the angle of the mandible
E. Mass anterior to the ear
Answer: D
Parotid tumours may present at any region in the gland. However, most lesions will be located behind the angle of the mandible,
inferior to the ear lobe. Tumours of the deep lobe of the parotid may present as a parapharyngeal mass and large lesions may displace
the tonsil.

266. A 52 year old male is referred to urology clinic with impotence. He is known to have hypertension. He does not have any
morning erections. On further questioning the patient reports pain in his buttocks, this worsens on mobilising. On
examination there is some muscle atrophy. The penis and scrotum are normal. What is the most likely diagnosis?
A. Leriche syndrome
B. S4-S5 cord lesion
C. Pudendal nerve lesion
D. Psychological impotence
E. Beta blocker induced impotence
Answer: A
Leriche syndrome: Classically, it is described in male patients as a triad of symptoms: 1. Claudication of the buttocks and thighs
2.Atrophy of the musculature of the legs 3. Impotence (due to paralysis of the L1 nerve)
Leriche syndrome, is atherosclerotic occlusive disease involving the abdominal aorta and/or both of the iliac arteries. Management
involves correcting underlying risk factors such as hypercholesterolaemia and stopping smoking. Investigation is usually with
angiography.

Leriche syndrome: Atheromatous disease involving the iliac vessels. Blood flow to the pelvic viscera is compromised. Patients may
present with buttock claudication and impotence (in this particular syndrome). Diagnostic work up will include angiography, where
feasible iliac occlusions are usually treated with endovascular angioplasty and stent insertion.
84
267. A 23 year old man presents with diarrhoea and passage of mucous. He is suspected of having ulcerative colitis. Which of
the following is least likely to be associated with this condition?
A. Superficial mucosal inflammation in the colon
B. Significant risk of dysplasia in long standing disease
C. Epsiodes of large bowel obstruction during acute attacks
D. Haemorrhage
E. Disease sparing the anal canal
Answer: C
Large bowel obstruction is not a feature of UC, patients may develop megacolon. However, this is a different entity both
diagnostically and clinically. Ulcerative colitis does not affect the anal canal and the anal transitional zone. Inflammation is
superficial. Dysplasia can occur in 2% overall, but increases significantly if disease has been present over 20 years duration.
Granulomas are features of crohn's disease.
Other features:Disease maximal in the rectum and may spread proximally. Contact bleeding. Longstanding UC crypt atrophy and
metaplasia/dysplasia

268. A 52 year old woman attends clinic for investigation of abdominal pain and constipation. On examination you note blue
lines on the gum margin. She mentions that her legs have become weak in the past few days. What is the most likely
diagnosis?
A. Acute intermittent porphyria
B. Lead poisoning
C. Constipation
D. Guillan Barre syndrome
E. Rectal carcinoma
Answer: B
This would be an impressive diagnosis to make in the surgical out patient department! The combination of abdominal pain and a
motor periperal neuropathy, should indicate this diagnosis. The blue line along the gum margin can occur in up to 20% patients with
lead poisoning.

Lead poisoning: Along with acute intermittent porphyria, lead poisoning should be considered in questions giving a combination of
abdominal pain and neurological signs. Features: Abdominal pain. Peripheral neuropathy (mainly motor). Fatigue. Constipation. Blue
lines on gum margin (only 20% of adult patients, very rare in children)

Investigations: The blood lead level is usually used for diagnosis. Levels greater than 10 mcg/dl are considered significant. Full blood
count: microcytic anaemia. Blood film shows red cell abnormalities including basophilic stippling and clover-leaf morphology. Raised
serum and urine levels of delta aminolaevulinic acid may be seen making it sometimes difficult to differentiate from acute intermittent
porphyria. Urinary coproporphyrin is also increased (urinary porphobilinogen and uroporphyrin levels are normal to slightly
increased). Management: various chelating agents are currently used: Dimercaptosuccinic acid (DMSA). D-penicillamine. EDTA.
Dimercaprol

Theme: Vasculitis

A. Wegeners granulomatosis
B. Polyarteritis nodosa
C. Giant cell arteritis
D. Takayasu's arteritis
E. Buergers disease

For each of the scenarios provided please select the most likely underlying diagnosis from the list below. Each option may be used
once, more than once or not at all.

269. A 20 year old lady is referred to the vascular clinic. She has been feeling generally unwell for the past six weeks. She works
as a typist and has noticed increasing pain in her forearms whilst working. On examination she has absent upper limb
pulses. Her ESR is measured and mildly elevated.
Answer: Takayasu's arteritis
Takayasus arteritis may be divided into acute systemic phases and the chronic pulseless phase. In the latter part of the disease

85
process the patient may complain of symptoms such as upper limb claudication. In the later stages of the condition the vessels will
typically show changes of intimal proliferation, together with band fibrosis of the intima and media.

270. A 32 year old man presents to the vascular clinic with symptoms of foot pain during exertion. He is a heavy smoker and has
recently tried to stop smoking. On examination he has normal pulses to the level of the popliteal. However, foot pulses are
absent. A diagnostic angiogram is performed which shows an abrupt cut off at the level of the anterior tibial artery,
together with the formation of corkscrew shaped collateral vessels distally.
Answer: Buergers disease
Buergers disease is most common in young male smokers. This demographic is changing in those areas where young female
smokers are more common. In the acute lesion the internal elastic lamina of the vessels is usually intact. As the disease progresses
the changes progress to hypercellular occlusive thrombus. Tortuous corkscrew collaterals may reconstitute patent segments of the
distal tibial or pedal vessels.

271. A 78 year old man presents with symptoms of headaches and deteriorating vision. He notices that there is marked pain on
the right hand side of his face when he combs his hair.
Answer: Giant cell arteritis
Temporal arteritis may present acutely with symptoms of headache and visual loss, or with a less acute clinical picture. Sight may
be threatened and treatment with immunosupressants should be started promptly. The often requested temporal artery biopsy
(which can be the bane of many surgeons) is often non diagnostic and unhelpful.

The vasculitides are a group of conditions characterised by inflammation of the blood vessel walls. This may, in turn, compromise
vessel integrity. Constitutional symptoms may be present. Whilst certain disease subtypes are reported to affect specific vessels, there
is often a degree of overlap clinically.

Vessel diameter and vasculitis classification


Aorta and branches: Takayasu's arteritis. Buergers disease. Giant cell arteritis. Large and medium sized arteries: Buergers disease.
Giant cell arteritis. Polyarteritis nodosa. Medium sized muscular arteries: Polyarteritis nodosa. Wegeners granulomatosis. Small
muscular arteries: Wegeners granulomatosis. Rheumatoid vasculitis

Specific conditions: Takyasu's arteritis: Inflammatory, obliterative arteritis affecting aorta and branches. Females> Males.
Symptoms may include upper limb claudication. Clinical findings include diminished or absent pulses. ESR often affected during the
acute phase. Buergers disease: Segmental thrombotic occlusions of the small and medium sized lower limb vessels. Commonest in
young male smokers. Proximal pulses usually present, but pedal pulses are lost. An acuter hypercellular occlusive thrombus is often
present. Tortuous corkscrew shaped collateral vessels may be seen on angiography. Giant cell arteritis: Systemic granulomatous
arteritis that usually affects large and medium sized vessels. Females > Males. Temporal arteritis is commonest type. Granulomatous
lesions may be seen on biopsy (although up to 50% are normal). Polyarteritis nodosa: Systemic necrotising vasculitis affecting small
and medium sized muscular arteries. Most common in populations with high prevalence of hepatitis B. Renal disease is seen in 70%
cases. Angiography may show saccular or fusiform aneurysms and arterial stenosis. Wegeners granulomatosis: Predominantly
affects small and medium sized arteries. Systemic necrotising granulomatous vasculitis. Cutaneous vascular lesions may be seen
(ulceration, nodules and purpura). Sinus imaging may show mucosal thickening and air fluid levels. TreatmentConditions such as
Buergers disease are markedly helped by smoking cessation. Immunosupression is the main treatment for vasculitides.

272. A 28 year old man presents with right upper quadrant pain and hydatid disease is suspected. Which of the following
statements relating to the disease is untrue?
A. First line treatment is with pentavalent antimony.
B. Peritoneal contamination with active daughter cysts may complicate surgery.
C. CT scanning of the liver may show a floating membrane.
D. Biliary communication with the cysts may occur.
E. It is caused by Echinococcus granulosus.
Answer: A
Drug treatment is with albendazole or mebendazole. Praziquantzel may be used in the pre operative stages.

273. Which of the following statements relating to neutrophil polymorphs is true?


A. Produce nitrogen peroxide as a microbicidal agent
B. Not involved in opsonisation
C. Deficiency leads to AIDS
D. Neutrophil disorders cause chronic granulomatous diseases
E. Have a lifespan of 9 hours
Answer: D
86
Neutrophils are the main cells of acute inflammation, important action against gram -ve and +ve bacteria. Appearance of segmented
nucleus and granulated cytoplasm. Have a lifespan of 1-3 days (shorter when consumed during septic process, though 9 hours is
unusual). Actions include: movement, opsonise microorganisms, phagocytosis & intracellular killing of microorganisms via aerobic
(produce HYDROGEN PEROXIDE) & anaerobic mechanisms. Neutrophil disorders include chronic granulomatous diseases: rare.
AIDS associated with T cell deficiency

274. A 33 year old man is involved in a road traffic accident. He is initially stable and transferred to the accident and emergency
department. On arrival he is catheterised. One minute later he becomes hypotensive, with evidence of angioedema
surrounding his penis. What is the most likely explanation for this event?
A. Type V latex hypersensitivity reaction
B. Type IV latex hypersensitivity reaction
C. Type III latex hypersensitivity reaction
D. Type I latex hypersensitivity reaction
E. Type II latex hypersensitivity reaction
Answer: D
Theme from April 2012 Exam. Sudden collapse and angioedema following exposure to latex (of which most urinary catheters are
manufactured) suggests a type I hypersensitivity reaction.

275. A 43 year old female develops severe chest wall cellulitis following a mastectomy. On examination the skin is markedly
erythematous. Which of the acute inflammatory mediators listed below is least likely to produce vasodilation?
A. Complement component C5a
B. Lysosomal compounds
C. Histamine
D. Serotonin
E. Prostaglandins
Answer: D
Erythema is a classical feature of acute inflammation. Potent mediators of vascular dilatation include; histamine, prostaglandins, nitric
oxide, platelet activating factor, complement C5a (and C3a) and lysosomal compounds. Although serotonin is associated with acute
inflammation it is a vasoconstrictor. The effects of serotonin are dependant upon the state of the vessels in the tissues. Intact and
healthy tissues and vessels will respond to a serotonin infusion with vasodilation (hence the flushing seen in carcinoid syndrome). In
contrast it worsens cardiac ischaemia in myocardial infarcts when released from damaged platelets.

276. A 43 year old man presents with a 3 week history of malaise, sore throat, odynophagia and dysphagia. On examination he
is found to have patchy white spots in his oropharynx. An upper GI endoscopy is performed and similar lesions are
identified in the oesophagus. Which investigation is most likely to identify the underlying pathology in this case?
A. Serum urea and electrolytes
B. Oesophageal biopsy for culture
C. Oesophageal biopsy for histology
D. Glucose tolerance testing
E. Viral serology
Answer: E
Oesophageal candidiasis is associated with immunosupression; mainly in patients on chemotherapy, with haematological malignancy,
HIV or inhaled steroids. In patients with HIV, oesophageal candidiasis is part of the spectrum of AIDS defining illnesses and usually
occurs when the CD4 count is less than 200. Others include PCP pneumonia and CMV infections.

Oesophageal candidiasis: Characterised by white spots in the oropharynx with extension into the oesophagus. Associated with broad
spectrum antibiotic usage, immunosupression and immunological disorders. Patients may present with oropharyngeal symptoms,
odynophagia and dysphagia. Treatment is directed both at the underlying cause (which should be investigated for) and with oral
antifungal agents.

Theme: Colonic obstruction

A. Malignant obstruction
B. Olgilvies syndrome
C. Volvulus
D. Diverticular stricture
87
E. Ischaemic stricture

Please select the most likely cause of obstruction for the situation described. Each option may be used once, more than once or not at
all.

277. A 78 year old man has undergone a hemi-arthroplasty for a intracapsular hip fracture. Post operatively he develops
electrolyte derangement and receives intravenous fluids. Over the previous 24 hours he develops marked abdominal
distension. On examination he has a tense, tympanic abdomen which is not painful. A contrast enema shows flow of
contrast through to the caecum and through the ileocaecal valve.
Answer: Olgilvies syndrome
Patients with electrolyte disturbance and previous surgery may develop colonic pseudo-obstruction (olgilvies syndrome). The
diagnosis is made using a contrast enema and treatment is usually directed at the underlying cause with colonic decompression if
indicated.

278. A 67 year old man has had multiple episodes with fever and left iliac fossa pain. These have usually resolved with courses
of intravenous antibiotics. He is admitted with a history of increasing constipation and abdominal distension. A contrast x-
ray is performed which shows flow of contrast to the sigmoid colon, here the contrast flows through a long narrow segment
of colon into dilated proximal bowel.
Answer: Diverticular stricture
The long history of left iliac fossa pain and development of bowel obstruction suggests a diverticular stricture. These may contain
a malignancy and most will require resection.

279. A 78 year old lady from a nursing home is admitted with a 24 hour history of absolute constipation and abdominal pain.
On examination she has a distended soft mass in her left iliac fossa. An x-ray is performed which shows a large dilated loop
of bowel in the left iliac fossa which contains a fluid level.

Answer: Volvulus
Sigmoid volvulus may present with an asymmetrical mass in an elderly patient. It may contain a fluid level, visible on plain films.

Colonic obstruction
Cancer: Usually insidious onset. History of progressive constipation. Systemic features (e.g. anaemia). Abdominal distension
Absence of bowel gas distal to site of obstruction. Establish diagnosis (e.g. contrast enema/ endoscopy). Treatment: Laparotomy and
resection, stenting, defunctioning colostomy or bypass
Diverticular stricture: Usually history of previous acute diverticulitis. Long history of altered bowel habit
Evidence of diverticulosis on imaging or endoscopy. Once diagnosis established, usually surgical resection. Colonic stenting should
not be performed for benign disease
Volvulus: Twisting of bowel around its mesentery. Sigmoid colon affected in 76% cases. Patients usually present with abdominal
pain, bloating and constipation. Examination usually shows asymmetrical distension. Plain X-rays usually show massively dilated
sigmoid colon, loss of haustra and "U" shape are typical, the loop may contain fluid levels. Treatment: Initial treatment is to untwist
the loop, a flexible sigmoidoscopy may be needed. Those with clinical evidence of ischaemia should undergo surgery. Patient with
recurrent volvulus should undergo resection
Acute colonic pseudo-obstruction: Symptoms and signs of large bowel obstruction with no lesion. Usually associated with metabolic
disorders. Usually a cut off in the left colon (82% cases). Although abdomen tense and distended, it is usually not painful. Treatment:
All patients should undergo contrast enema (may be therapeutic!). Colonoscopic decompression. Correct metabolic disorders. IV
neostigmine. Surgery.

280. A 52 year old man with dyspepsia is found to have a duodenal ulcer. A CLO test is taken and is positive. Which statement
relating to the likely causative organism is false?
A. It is a gram negative organism
B. It lives only on gastric type mucosa
C. It may occupy areas of ectopic gastric metaplasia
D. In patients who are colonised there is commonly evidence of fundal gastritis on endoscopy
E. It produces a powerful urease that forms the basis of the Clo test
Answer: D
Helicobacter pylori accounts for >75% cases of duodenal ulceration. It may be diagnosed with either serology, microbiology,
histology or CLO testing.

88
Theme from January 2011 Exam. Helicobacter pylori rarely produces any typical features on endoscopy. Where infection is suspected
the easiest course of action is to take an antral biopsy for Clo testing in the endoscopy suite.

Helicobacter Pylori

Infection with Helicobacter Pylori is implicated in many cases of duodenal ulceration and up to 60% of patients with gastric
ulceration. Gram negative, helix shaped rod, microaerophillic. Produces hydrogenase that can derive energy from hydrogen released
by intestinal bacteria. Flagellated and mobile. Those carrying the cag A gene may cause ulcers. It secretes urease that breaks down
gastric urea> Carbon dioxide and ammonia> ammonium>bicarbonate (simplified!) The bicarbonate can neutralise the gastric acid.
Usually colonises the gastric antrum and irritates resulting in increased gastrin release and higher levels of gastric acid. These patients
will develop duodenal ulcers. In those with more diffuse H-Pylori infection gastric acid levels are lower and ulcers develop by local
tissue damage from H-Pylori- these patients get gastric ulcers. Diagnosis may be made by serology (approx. 75% sensitive). Biopsy
urease test during endoscopy probably the most sensitive. In patients who are colonised 10-20% risk of peptic ulcer, 1-2% risk gastric
cancer, <1% risk MALT lymphoma.

281. A 34 year old male presents with painful rectal bleeding and a fissure in ano is suspected. On examination he has an
epithelial defect at the mucocutaenous defect that is located anteriorly. Approximately what proportion of patients with
fissure in ano will present with this pattern of disease?
A. 90%
B. 10%
C. 50%
D. 25%
E. 100%
Answer: B
Only a minority of patients with fissure in ano will have an anteriorly sited fissure. They are particularly rare in males and an anterior
fissure in a man should prompt a search for an underlying cause.

Anal fissure: Anal fissures are a common cause of painful, bright red, rectal bleeding. Most fissures are idiopathic and present as a
painful mucocutaneous defect in the posterior midline (90% cases). Fissures are more likely to be anteriorly located in females,
particularly if they are multiparous. Multiple fissures and those which are located at other sites are more likely to be due to an
underlying cause. Diseases associated with fissure in ano include: Crohns disease. Tuberculosis . Internal rectal prolapse

Diagnosis: In most cases the defect can be visualised as a posterior midline epithelial defect. Where symptoms are highly suggestive
of the condition and examination findings are unclear an examination under anaesthesia may be helpful. Atypical disease presentation
should be investigated with colonoscopy and EUA with biopsies of the area.

Treatment: Stool softeners are important as the hard stools may tear the epithelium and result in recurrent symptoms. The most
effective first line agents are topically applied GTN (0.2%) or Diltiazem (2%) paste. Side effects of diltiazem are better tolerated.
Resistant cases may benefit from injection of botulinum toxin or lateral internal sphincterotomy (beware in females). Advancement
flaps may be used to treat resistant cases.Sphincterotomy produces the best healing rates. It is associated with incontinence to flatus in
up to 10% of patients in the long term.

282. The pathogenesis of osteopetrosis is best explained by a defect in which of the following?
A. Osteoclast function
B. PTH receptors
C. Osteoblast function
D. Calcium resorption in proximal tubule
E. Calcium absorption
Answer: A
Osteopetrosis: aka marble bone disease. Rare disorder of defective osteoclast function resulting in failure of normal bone resorption.
Stem cell transplant and interferon-gamma have been used for treatment

283. A 13 month old boy is brought to the surgical clinic by his mother because his left testicle is not located in the scrotum. At
which of the following sites would the testicle be located if it were an ectopic testis?
A. Canalicular
89
B. Inguinal
C. External inguinal ring
D. Superficial inguinal pouch
E. High scrotal
Answer: D
Theme from September 2011 Exam. Theme from January 2012 Exam. Ectopic testes are those that come to lie outside the normal
range of embryological descent (i.e. in the superficial inguinal pouch). Other sites of ectopic testes include; base of penis, femoral and
perineal.

Testicular disorders-paediatric

Testicular disorders
Testicular disorders are some of the commonest conditions present in paediatric urological practice.

Cryptorchidism: The embryological descent of the testicle from within the abdominal cavity may be subject to a number of
variations. Distinctions need to be made clinically from a non descended testis and a testis that is retractile. Testis that lie outside the
normal path of embryological descent are termed ectopic testis. Undescended testis occurs in 1% of male infants. Where the testis
does not lie in an intra scrotal location, its location should be ascertained. Where both testes are absent the infant may be intersex.
MRI scanning may reveal intra-abdominal testes; however a GA is often needed to perform this investigation in this age group. Testes
that are undescended should be placed in the scrotum after 1 year of age as the testosterone surge that may facilitate descent occurs at
6 months of age. Where the testes lie distally e.g. Superficial inguinal pouch an open orchidopexy is the procedure of choice. With
abdominal testes a laparoscopy should be performed. The risk of seminoma is increased in individuals with a non descended testes and
this risk is not reduced by orchidopexy.

Testicular torsion: Typically the patient has severe sudden onset of scrotal pain. The difficulty in paediatric practice is the lack of
clear history. On examination the testis is tender and enlarged. Management is by surgical exploration. Delay beyond 6 hours is
associated with low salvage rates. A torted hyatid produces pain that is far more localised and the testis itself should feel normal.
However, diagnostic doubt often exists and in such cases surgical exploration is warranted.

284. A splenectomy increases the risk of infection from all the following organisms except?
A. Pneumococcus
B. Klebsiella
C. Haemophilus influenzae
D. Staphylococcus aureus
E. Neisseria meningitides
Answer: D
Theme from 2010 Exam. Staphylococcus aureus infection following splenectomy is no more common than in non splenectomised
individuals. The other organisms are encapsulateed, which is why they are more likely to cause overwhelming post splenectomy
sepsis.

285. Which of the following is not an extraintestinal feature Crohns disease?


A. Iritis
B. Clubbing
C. Aphthous ulcers
D. Erythema multiforme
E. Pyoderma gangrenosum
Answer: D
Extraintestinal manifestation of inflammatory bowel disease: A PIE SAC: Aphthous ulcers. Pyoderma gangrenosum. Iritis. Erythema
nodosum. Sclerosing cholangitis. Arthritis. Clubbing

286. Which of the following is not considered a risk factor for the development of oesophageal malignancy?
A. Oesophageal metaplasia
B. Smoking
C. Excessive intake of alcoholic spirits
D. Achalasia
E. Blood group O
Answer: E
90
Blood group O is not a risk factor for oesophageal cancer. Achalasia is associated with the risk of developing squamous cell
carcinoma of the oesophagus.

Theme: Thyroid blood testing

A. Measurement of antibodies to TSH receptor


B. Thyroid peroxidase antibodies
C. Thyroglobulin antibodies
D. Serum calcitonin

Please select the blood test most commonly performed for the diagnosis or assessment of the thyroid disorder described. Each answer
may be used once, more than once or not at all.

287. A 32 year old lady is diagnosed with Medullary carcinoma of the thyroid and has undergone resection of the tumour.
Answer: Serum calcitonin
Measurement of basal or stimulated calcitonin concentrations is used to assess the completeness of surgical resection, and is of use
in detecting diseases recurrences during follow up.

288. A 20 year old lady has undergone a total thyroidectomy for a well differentiated papillary carcinoma. She attends clinic
and is well and the surgeon wishes to screen for disease recurrence.
Answer: Thyroglobulin antibodies
Antibodies to thyroglobulin, the major constituent of colloid and precursor of thyroid hormones may be elevated in those with
metastatic or recurrent thyroid cancer. Results may be erronoeous in those with other thyroid disorders.

289. A 33 year old lady presents with a recently diagnosed goitre and a diagnosis of Hashimotos thyroiditis is suspected.
Answer: Thyroid peroxidase antibodies
Antibodies to thyroid peroxidase are found in most patients with Graves disease or Hashimotos thyroiditis.

Blood testing in thyroid disease: Thyroid peroxidase (microsomal) antibodies:Found in autoimmune disease affecting the thyroid
(Hashimotos 100%) and Graves (70%). Antibodies to TSH receptor:Individuals with Graves disease (95%). Thyroglobulin
antibodies:Not useful for clinically distinguishing between different types of thyroid disease, may be used as part of thyroid cancer
follow up. Calcitonin:Released from the parafollicular cells. Usually found in patient with Medullary carcinoma of the thyroid

290. Which one of the following complications is least associated with ventricular septal defects?
A. Right heart failure
B. Aortic regurgitation
C. Eisenmenger's complex
D. Infective endocarditis
E. Atrial fibrillation
Answer: E
Atrial fibrillation is associated more with atrial septal defects

Ventricular septal defect: Ventricular septal defects are the most common cause of congenital heart disease. They close
spontaneously in around 50% of cases. Non-congenital causes include post myocardial infarction. Features: classically a pan-systolic
murmur which is louder in smaller defects. Complications: aortic regurgitation*. Infective endocarditis. Eisenmenger's complex.
Right heart failure
*aortic regurgitation is due to a poorly supported right coronary cusp resulting in cusp prolapse

291. A 24 year old man from Sudan presents with a lymphadenopathy and weight loss. A diagnosis of tuberculosis is suspected
and a lymph node biopsy is performed. Staining with which of the agents below is most likely to facilitate identification of the
causative organism?
A. Gram stain
B. Ziehl-Neelsen stain
C. Von Kossa stain
D. Van Gieson stain
E. Masson Trichrome stain
Answer: B

91
Ziehl-Neelsen stain is typically used to identify mycobacteria. They are not stained in the Gram staining process. Van Gieson and
Masson trichrome are histological staining methods for identification of connective tissues. The Von Kossa technique is useful for
identifying tissue mineralisation.

292. Which of the following is not a pathological feature of breast cancer?


A. Resemblance to ductal epithelial cells
B. Angiogenesis
C. Nuclear pleomorphism
D. Metastatic calcification
E. Vascular invasion
Answer: D
Dystrophic calcification may be present in breast malignancy and is the basis for the breast screening programme. Metastatic
calcification is calcification which occurs in otherwise normal tissues, usually as a result of hypercalcaemia. Invasive ductal
carcinoma is the most common type of breast cancer, unless the tumour is very poorly differentiated there is usually some
resemblance to ductal epithelial cells.

Breast cancer pathology:


The histological features of breast cancer depend upon the underlying diagnosis. The invasive component is usually comprised of
ductal cells (unless it is an invasive lobular cancer). In situ lesions may co-exist (such as DCIS). Typical changes seen in conjunction
with invasive breast cancer include: 1. Nuclear pleomorphism; 2. Coarse chromatin; 3. Angiogenesis; 4. Invasion of the basement
membrane; 5. Dystrophic calcification (may be seen on mammography); 6. Abnormal mitoses; 7. Vascular invasion; 8. Lymph node
metastasis
The primary tumour is graded on a scale of 1-3 where 1 is the most benign lesion and 3 the most poorly differentiated.
Immunohistochemistry for oestrogen receptor and herceptin status is routinely performed.
The grade, lymph node stage and size are combined to provide the Nottingham prognostic index.

Theme: Renal lesions

A. Renal cell carcinoma


B. Renal transitional cell carcinoma
C. Nephroblastoma
D. Neuroblastoma
E. Angiomyolipoma
F. Renal squamous cell carcinoma
G. Retroperitoneal fibrosis

For each scenario please select the most likely underlying diagnosis. Each option may be used once, more than once or not at all.

293. A 69 year old male presents with haematuria. He worked in the textile industry. He has a left flank mass. A CT IVU shows
a lesion of the left renal pelvis.
Answer: Renal transitional cell carcinoma
TCC is a rare form of renal cancer, accounting for approximately 7% of all renal tumours. Risk factors include exposure to
chemicals in the textile, plastic and rubber industry.

294. A 2 year old boy presents with a right renal mass. On examination he has an irregular mass arisong from the right flank
and is hypertensive. A CT scan shows a non calcified irregular lesion affecting the apex of the right kidney and the right
adrenal gland.
Answer: Nephroblastoma
Wilm's tumour of the kidney is the most common renal tumour in children. Both nephroblastoma and neuroblastoma may occupy
the adrenal and apex of the kidney. In the case of neuroblastoma the lesion will have arisen from the adrenal, in the case of
nephroblastoma the lesion will have arisen from the kidney. Hypertension is more commonly associated with nephroblastoma.
Neuroblastomas are usually calcified, whereas nephroblastomas are not and this may be of diagnostic usefulness pre operatively.

295. A 35 year old male presents with haematuria. He is found to have bilateral masses in the flanks. He has a history of
epilepsy and learning disability.
Answer: Angiomyolipoma
This patient has tuberous sclerosus. This is associated with angiomyolipoma, which is present in 60-80% patients. It is a benign
lesion.

92
296. An 18 month old boy presents with recurrent urinary tract infections. As part of the diagnostic work-up he is noted to have
abnormal renal function. An ultrasound scan is performed and shows bilateral hydronephrosis. What is the most likely
underlying diagnosis?
A. Urethral valves
B. Meatal stenosis
C. Hydronephrosis
D. Pelvico-ureteric junction obstruction
E. Benign prostatic hyperplasia
Answer: A
Theme from April 2012 Exam. A posterior urethral valve is an obstructive, developmental uropathy that usually affects male infants
(incidence 1 in 8000). Diagnostic features include bladder wall hypertrophy, hydronephrosis and bladder diverticula.

Urethral valves: Posterior urethral valves are the commonest cause of infravesical outflow obstruction in males. They may be
diagnosed on ante natal ultrasonography. Because the bladder has to develop high emptying pressures in utero the child may develop
renal parenchymal damage. This translates to renal impairment noted in 70% of boys at presentation. Treatment is with bladder
catheterisation. Endoscopic valvotomy is the definitive treatment of choice with cystoscopic and renal follow up.

297. At which of the following anatomical sites does dormant tuberculosis most frequently reactivate?
A. Apex of the lung
B. Base of the lung
C. Brain
D. Terminal ileum
E. Lumbar spine
Answer: A
TB reactivation most commonly occurs at the lung apex. This site is better oxygenated than elsewhere allowing the
mycobacteria to multiply more rapidly and then spread both locally and distantly.

298. What is the commonest tumour type encountered in the colon?


A. Squamous cell carcinoma
B. Adenocarcinoma
C. Lymphoma
D. Anaplastic carcinoma
E. Sarcoma
Answer: B
Adenocarcinoma are the most common and typically arise as a result of the adenoma - carcinoma sequence.

299. Which of the following changes are most likely to be identified in the aortic wall of a 38 year old lady with a Marfans
syndrome and a dissecting aortic aneurysm?
A. Transmural aortitis
B. Cystic medial necrosis
C. Foamy macrophages
D. Dense dystrophic calcification
E. None of the above
Answer: B
Cystic medial necrosis ( or cystic medial degeneration) occurs when basophils and mucoid material lie in between the intimal elastic
fibres of the aorta. It is typically found in the aortic degeneration of Marfans syndrome, but may also be seen in aortic degeneration in
older adults.

300. A 58 year old man undergoes an upper GI endoscopy for the investigation of odynophagia. At endoscopy a reddish area is
seen to protrude up into the oesophagus from the gastrooesophageal junction. Which of the following pathological
processes is most likely to explain this process?
A. Metaplasia
B. Anaplasia
C. Dysplasia
D. Hypoplasia
93
E. Hyperplasia
Answer: A
This is most likely to represent Barretts oesphagus and is thus metaplasia. Dysplasia is less likely in this setting although biopsies are
mandatory.

301. A male infant is born by emergency cesarean section at 39 weeks gestation for foetal distress. Soon after the birth the baby
becomes progressively hypoxic and on examination is found to have a scaphoid abdomen. What is the most likely
underlying diagnosis?
A. Intestinal malrotation
B. Hiatus hernia
C. Foramen of Bochdalek hernia
D. Foramen of Morgagni hernia
E. Tracheooesphageal fistula
Answer: C

The finding of a scaphoid abdomen and respiratory distress suggests extensive intra thoracic herniation of the abdominal contents.
This is seen most frequently with Bochdalek hernias. Morgagni hernias seldom present in such a dramatic fashion. The other options
do not typically present with the symptoms and signs described.

Embryology: The diaphragm is formed between the 5th and 7th weeks of gestation through the progressive fusion of the septum
transversum, pleuroperitoneal folds and via lateral muscular ingrowth. The pleuroperitoneal membranes from which the diaphragm
originates originate at somites located in cervical segments 3 to 5, which accounts for the long path taken by the phrenic nerve. The
components contribute to the following diaphragmatic segments: Septum transversum - Central tendon. Pleuroperitoneal membranes -
Parietal membranes surrounding viscera. Cervical somites C5 to C7 - Muscular component of the diaphragm

Diaphragmatic hernia: Morgagni: Anteriorly located. Minimal compromise on lung development. Minimal signs on antenatal
ultrasound. Usually present later. Usually good prognosis. Bochdalek hernia: Posteriorly located. Larger defect. Often diagnosed
antenatally. Associated with pulmonary hypoplasia. Poor prognosis.
The posterior hernias of Bochdalek are the most common type and if not diagnosed antenatally will typically present soon after birth
with respiratory distress. The classical finding is that of a scaphoid abdomen on clinical examination because of herniation of the
abdominal contents into the chest. Bochdalek hernias are associated with a number of chromosomal abnormalities such as Trisomy 21
and 18. Infants have considerable respiratory distress due to hypoplasia of the developing lung. Historically this was considered to be
due to direct compression of the lung by herniated viscera. This view over simplifies the situation and the pulmonary hypoplasia
occurs concomitantly with the hernial development, rather than as a direct result of it. The pulmonary hypoplasia is associated with
pulmonary hypertension and abnormalities of pulmonary vasculature. The pulmonary hypertension renders infants at risk of right to
left shunting (resulting in progressive and worsening hypoxia). Diagnostic work up of these infants includes chest x-rays/ abdominal
ultrasound scans and cardiac echo. Surgery forms the mainstay of treatment and both thoracic and abdominal approaches may be
utilised. Following reduction of the hernial contents a careful search needs to be made for a hernial sac as failure to recognise and
correct this will result in a high recurrence rate. Smaller defects may be primarily closed, larger defects may require a patch to close
the defect. Malrotation of the viscera is a recognised association and may require surgical correct at the same procedure (favoring an
abdominal approach). The mortality rate is 50-75% and is related to the degree of lung compromise and age at presentation
(considerably better in infants >24 hours old).

302. A 72 year old lady falls and lands on her left hip. She attends the emergency department and is given some paracetamol by
the junior doctor and discharged. Several months later she presents with ongoing pain and discomfort of the hip.
Avascular necrosis of the femoral head is suspected. Which of the following features is least likely to be present?
A. Non union of the fracture
B. Angiogenesis at the fracture site
C. Increased numbers of fibroblasts at the fracture site
D. Osteochondritis dissecans
E. Apoptosis of osteoblasts
Answer: E
Apoptosis is not a feature of necrotic cell death. By this stage there would usually be attempted repair so angiogenesis and
proliferation of fibroblasts would be expected. These cells may differentiate further to become osteoblasts which in turn will lay down
new matrix.

94
Avascular necrosis: Cellular death of bone components due to interruption of the blood supply, causing bone destruction. Main joints
affected are hip, scaphoid, lunate and the talus. It is not the same as non union. The fracture has usually united. Radiological evidence
is slow to appear. Vascular ingrowth into the affected bone may occur. However, many joints will develop secondary osteoarthritis.
Causes: P ancreatitis. L upus. A lcohol. S teroids. T rauma. I diopathic, infection. C aisson disease, collagen vascular disease. R
adiation, rheumatoid arthritis. A myloid. G aucher disease. S ickle cell disease
Presentation: Usually pain. Often despite apparent fracture union. Investigation: MRI scanning will show changes earlier than plain
films. Treatment: In fractures at high risk sites anticipation is key. Early prompt and accurate reduction is essential.
Non weight bearing may help to facilitate vascular regeneration.
Joint replacement may be necessary, or even the preferred option (e.g. Hip in the elderly).

303. Which one of the following is least associated with the development of colorectal cancer in patients with ulcerative colitis?
A. Unremitting disease
B. Disease duration > 10 years
C. Onset before 15 years old
D. Poor compliance to treatment
E. Disease confined to the rectum
Answer: E
Ulcerative colitis and colorectal cancer: Overview: risk of colorectal cancer is 10-20 times that of general population. The increased
risk is mainly related to chronic inflammation. Worse prognosis than patients without ulcerative colitis (partly due to delayed
diagnosis). Lesions may be multifocal
Factors increasing risk of cancer: Disease duration > 10 years. Patients with pancolitis. Onset before 15 years old. Unremitting
disease. Poor compliance to treatment

304. Which of the following statements about wound healing is false?


A. During the course of the first 2 weeks the fibrin plug is replaced by a collagen rich mesh.
B. Scar contraction is a late feature and is mediated by myofibroblasts.
C. May be impaired when an infected foreign body is present.
D. Is more cosmetically acceptable when incisions are made parallel to Langer's lines.
E. Has features of chronic inflammation from the beginning.

Answer: E
Although wounds often contain cell types (macrophages) and processes (angiogenesis) these typically take time to occur. They are not
present in the immature wound.

Theme: Causes of chest pain

A. Pulmonary embolism
B. Anterior myocardial infarction
C. Inferior myocardial infarction
D. Proximal aortic dissection
E. Distal aortic dissection
F. Boerhaave Syndrome
G. Mallory weiss tear
H. Perforated gastric ulcer

Please select the most likely cause of chest pain for the scenario given. Each option may be used once, more than once or not at all.

305. A 52 year old male presents with tearing central chest pain. On examination he has an aortic regurgitation murmur. An
ECG shows ST elevation in leads II, III and aVF.
Answer: Proximal aortic dissection
An inferior myocardial infarction and AR murmur should raise suspicions of an ascending aorta dissection rather than an inferior
myocardial infarction alone. Also the history is more suggestive of a dissection. Other features may include pericardial effusion,
carotid dissection and absent subclavian pulse.

306. A 52 year old male presents with central chest pain and vomiting. He has drunk a bottle of vodka. On examination there is
some mild crepitus in the epigastric region.
Answer: Boerhaave Syndrome

95
The Mackler triad for Boerhaave syndrome: vomiting, thoracic pain, subcutaneous emphysema. It commonly presents in middle
aged men with a background of alcohol abuse.

307. A 52 year old male presents with central chest pain. On examination he has an mitral regurgitation murmur. An ECG
shows ST elevation in leads V1 to V6. There is no ST elevation in leads II, III and aVF.
Answer: Anterior myocardial infarction
The most likely diagnosis is an anterior MI. As there are no ST changes in the inferior leads, aortic dissection is less likely.

Chest pain
Aortic dissection: This occurs when there is a flap or filling defect within the aortic intima. Blood tracks into the medial layer and
splits the tissues with the subsequent creation of a false lumen. It most commonly occurs in the ascending aorta or just distal to the left
subclavian artery (less common). It is most common in Afro-carribean males aged 50-70 years. Patients usually present with a tearing
intrascapular pain, which may be similar to the pain of a myocardial infarct. The dissection may spread either proximally or distally
with subsequent disruption to the arterial branches that are encountered. In the Stanford classification system the disease is classified
into lesions with a proximal origin (Type A) and those that commence distal to the left subclavian (Type B). Diagnosis may be
suggested by a chest x-ray showing a widened mediastinum. Confirmation of the diagnosis is usually made by use of CT angiography.
Proximal (Type A) lesions are usually treated surgically, type B lesions are usually managed non operatively.

Pulmonary embolism: Typically sudden onset of chest pain, haemoptysis, hypoxia and small pleural effusions may be present. Most
patients will have an underlying deep vein thrombosis. Diagnosis may be suggested by various ECG findings including S waves in
lead I, Q waves in lead III and inverted T waves in lead III. Confirmation of the diagnosis is usually made through use of CT
pulmonary angiography. Treatment is with anticoagulation, in those patients who develop a cardiac arrest or severe compromise from
their PE, consideration may be given to thrombolysis.

Myocardial infarction: Traditionally described as sudden onset of central, crushing chest pain. It may radiate into the neck and down
the left arm. Signs of autonomic dysfunction may be present. The presenting features may be atypical in the elderly and those with
diabetes. Diangosis is made through identification of new and usually dynamic ECG changes (and cardiac enzyme changes). Inferior
and anterior infarcts may be distinguished by the presence of specific ECG changes (usually II, III and aVF for inferior, leads V1-V5
for anterior). Treatment is with oral antiplatelet agents, primary coronary angioplasty and/ or thrombolysis.

Perforated peptic ulcer: Patients usually develop sudden onset of epigastric abdominal pain, it may be soon followed by generalised
abdominal pain. There may be features of antecendant abdominal discomfort, the pain of gastric ulcer is typically worse immediately
after eating. Diagnosis may be made by erect chest x-ray which may show a small amount of free intra-abdominal air (very large
amounts of air are more typically associated with colonic perforation). Treatment is usually with a laparotomy, small defects may be
excised and overlaid with an omental patch, larger defects are best managed with a partial gastrectomy.

Boerhaaves syndrome: Spontaneous rupture of the oesophagus that occurs as a result of repeated episodes of vomiting. The rupture is
usually distally sited and on the left side. Patients usually give a history of sudden onset of severe chest pain that may complicate
severe vomiting. Severe sepsis occurs secondary to mediastinitis. Diagnosis is CT contrast swallow. Treatment is with thoracotomy
and lavage, if less than 12 hours after onset then primary repair is usually feasible, surgery delayed beyond 12 hours is best managed
by insertion of a T tube to create a controlled fistula between oesophagus and skin. Delays beyond 24 hours are associated with a very
high mortality rate.

308. A 78 year old lady presents with a tender swelling in her right groin. On examination there is a tender swelling that lies
lateral to the pubic tubercle. It has a cough impulse. What is the most likely underlying diagnosis?
A. Thrombophlebitis of the great saphenous vein
B. Femoral hernia
C. Thrombophlebitis of saphena varix
D. Inguinal hernia
E. Obturator hernia
Answer: B
Theme from April 2012 Exam. Whilst a thrombophlebitis of a saphena varix may cause a tender swelling at this site, it would
not usually be associated with a cough impulse.

Femoral canal: The femoral canal lies at the medial aspect of the femoral sheath. The femoral sheath is a fascial tunnel containing
both the femoral artery laterally and femoral vein medially. The canal lies medial to the vein.
Borders of the femoral canal: Laterally:Femoral vein. Medially:Lacunar ligament. Anteriorly: Inguinal ligament.
96
Posteriorly:Pectineal ligament
Contents: Lymphatic vessels. Cloquet's lymph node
Physiological significance: Allows the femoral vein to expand to allow for increased venous return to the lower limbs.
Pathological significance: As a potential space, it is the site of femoral hernias. The relatively tight neck places these at high risk of
strangulation.

309. A 3 month old boy is suspected of having hypospadias. At which of the following locations is the urethral opening most
frequently located in boys suffering from the condition?
A. On the distal ventral surface of the penis
B. On the proximal ventral surface of the penis
C. On the distal dorsal surface of the penis
D. On the proximal dorsal surface of the penis
E. At the base of the scrotum
Answer: A
The defect is located ventrally and most often distally. Proximally located urethral openings are well recognised. Circumcision may
compromise reconstruction.

310. A 52 year old male attends for a preoperative assessment for an inguinal hernia repair. You notice that the chest x-ray
shows a loculated left pleural effusion. On further questioning the patient reports that he worked as a builder 30 years ago.
What is the most likely cause for the effusion?
A. Asbestosis
B. Pneumonia
C. Mesothelioma
D. Silicosis
E. Left ventricular failure
Answer: C

This patient has a risk of asbestos exposure through his occupation as a builder. As there a is latent period of 30 years and a
complicated effusion, the most likely cause is mesothelioma.

Mesothelioma: Features: Dyspnoea, weight loss, chest wall pain. Clubbing. 30% present as painless pleural effusion. Only 20% have
pre-existing asbestosis. History of asbestos exposure in 85-90%, latent period of 30-40 years
Basics: Malignancy of mesothelial cells of pleura. Metastases to contralateral lung and peritoneum. Right lung affected more often
than leftManagement: Investigation: pleural biopsy, CT Scanning, (PET Scanning if surgery considered). Symptomatic. Industrial
compensation. Chemotherapy, Surgery if operable. Prognosis poor, median survival 12 months

311. A 64-year-old woman who is reviewed due to multiple non-healing leg ulcers. She reports feeling generally unwell for many
months. Examination findings include a blood pressure of 138/72 mmHg, pulse 90 bpm, pale conjunctivae and poor
dentition associated with bleeding gums. What is the most likely underlying diagnosis?
A. Thyrotoxicosis
B. Vitamin B12 deficiency
C. Vitamin C deficiency
D. Diabetes mellitus
E. Sarcoidosis
Answer: C

Bleeding gums and poor healing are suggestive of vitamin C deficiency.

Vitamin C deficiency: Vitamin C deficiency (scurvy) leads to defective synthesis of collagen resulting in capillary fragility (bleeding
tendency) and poor wound healing. Features: Gingivitis. Loose teeth. Poor wound healing. Bleeding from gums, haematuria,
epistaxis. General malaise

312. Which of the following is not a typical feature of neuropraxia?


A. Transient delay in neuronal transmission
B. Axonal degeneration distal to the site of injury
C. Absence of neuroma formation
D. Preservation of autonomic function
E. Absence of axonal degeneration proximal to the site of injury
Answer: B
97
Full recovery may occur 6-8 weeks after nerve injury in neuropraxia. Wallerian degeneration does not usually occur in simple
neuropraxia. Autonomic function is usually preserved.

313. A 44 year old lady presents with a pathological fracture of the left femur. She has previously undergone a renal transplant
for end stage renal failure. Her blood test results are as follows: Serum Ca2+ 2.80, PTH 88pg/ml, Phosphate 0.30. A surgeon
decides to perform a parathyroidectomy on the basis of these results. When the glands are assessed histologically, which of
the appearances is most likely to be identified?
A. Metaplasia the gland
B. Hypertrophy of the gland
C. Hyperplasia of the gland
D. Parathyroid carcinoma
E. Necrosis of the parathyroid gland
Answer: C

This is likely to be a case of tertiary hyperparathyroidism (high Calcium, high PTH, low phosphate). Therefore the glands will be
hyperplastic. Hypertrophy is not correct as this implies an increase in size without an increase in cellularity. This mistake has cost
many candidates marks in the MRCS exams over the years!

314. A 56 year old man presents with lethargy, haematuria and haemoptysis. On examination he is hypertensive and has a left
loin mass. A CT scan shows a lesion affecting the upper pole of the right kidney, it has a small cystic centre. Which of the
options below is the most likely diagnosis?
A. Squamous cell carcinoma of the kidney
B. Nephroblastoma
C. Renal adenocarcinoma
D. Transitional cell carcinoma of the kidney
E. Polycystic kidney disease
Answer: C

Renal adenocarcinoma are the most common renal tumours. These will typically affect the renal parenchyma. Transitional cell
carcinoma will usually affect urothelial surfaces. Nephroblastoma would be very rare in this age group. Renal adenocarcinoma may
produce cannon ball metastasis in the lung which cause haemoptysis, this is not a feature of PKD.

315. A 34-year-old man is taken immediately to theatre with aortic dissection. You note he is tall with pectus excavatum and
arachnodactyly. His condition is primarily due to a defect in which one of the following proteins?
A. Polycystin-1
B. Fibrillin
C. Type IV collagen
D. Type I collagen
E. Elastin
Answer: B
Although fibrillin is the primary protein affected (due to a defect in the fibrillin-1 gene) it should be noted that fibrillin is used as a
substrate of elastin.Marfan's syndrome: is an autosomal dominant connective tissue disorder. It is caused by a defect in the fibrillin-1
gene on chromosome 15 and affects around 1 in 3,000 people.
Features: Tall stature with arm span to height ratio > 1.05. High-arched palate. Arachnodactyly. Pectus excavatum. Pes planus.
Scoliosis of > 20 degrees. Heart: dilation of the aortic sinuses (seen in 90%) which may lead to aortic aneurysm, aortic dissection,
aortic regurgitation, mitral valve prolapse (75%). Lungs: repeated pneumothoraces. Eyes: upwards lens dislocation (superotemporal
ectopia lentis), blue sclera, myopia. Dural ectasia (ballooning of the dural sac at the lumbosacral level)

The life expectancy of patients used to be around 40-50 years. With the advent of regular echocardiography monitoring and beta-
blocker/ACE-inhibitor therapy this has improved significantly over recent years. Aortic dissection and other cardiovascular problems
remain the leading cause of death however.

316. Which of the following are not typical of Lynch syndrome?


A. It is inherited in an autosomal recessive manner
B. Affected patients are more likely to develop right colon mucinous tumours than the general population
C. Affected individuals have an 80% lifetime risk of colon cancer
D. Endometrial cancer is seen in 80%.
E. Gastric cancers are more common
Answer: A
Lynch syndrome is inherited in an autosomal dominant fashion. It is characterised by microsatellite instability in the DNA mismatch
98
repair genes. Colonic tumours in patients with Lynch syndrome are more likely to be right sided tumours and to be poorly
differentiated.

317. An enthusiastic medical student approaches you with a list of questions about blood transfusion reactions. Which of her
following points is incorrect?
A. Graft versus host disease involves neutrophil proliferation
B. Thrombocytopaenia may occur in women with a prior pregnancy
C. IgA antibodies may cause blood pressure compromise during transfusion
D. Hypocalcaemia can occur
E. Iron overload can be avoided by chelation therapy
Answre: A
Mnemonic for transfusion reactions:

Got a bad unit: G raft vs. Host disease. O verload. T hrombocytopaenia. A lloimmunization. B lood pressure unstable. A cute
haemolytic reaction. D elayed haemolytic reaction. U rticaria. N eutrophilia. I nfection. T ransfusion associated lung injury
GVHD results from lymphocytic proliferation. The patient's own lymphocytes are similar to the donor's lymphocytes, therefore don't
perceive them as being foreign. The donor lymphocytes, however, sees the recipient lymphocytes as being foreign. Therefore they
proliferate causing severe complications. Thrombocytopaenia occurs a few days after transfusion and may resolve spontaneously.
Patients with IGA antibodies need IgA deficient blood transfusions.

Blood transfusion reactions


Immune mediated: Pyrexia. Alloimmunization. Thrombocytopaenia. Transfusion associated lung injury. Graft vs Host disease.
Urticaria. Acute or delayed haemolysis. ABO incompatibility. Rhesus incompatibility.
Non immune mediated: Hypocalcaemia. CCF. Infections. Hyperkalaemia
Notes: GVHD: lymphocyte proliferation causing organ failure. Transfusion associated lung injury: neutrophil mediated allergic
pulmonary oedema. ABO and Rhesus incompatibility: causes acute haemolytic transfusion reaction leading to agglutination and
haemolysis

318. An 82 year old lady presents with a carcinoma of the caecum. Approximately what proportion of patients presenting with
this diagnosis will have synchronous lesions?
A. <1%
B. 60%
C. 50%
D. 20%
E. 5%
Answer: E
Synchronous colonic tumours are seen in 5% cases and all patients having a flexible sigmoidoscopy should have completion
colonoscopy if tumours or polyps are found
Synchronous lesions may occur in up to 5% of patients with colorectal cancer. A full and complete lumenal study with either
colonoscopy, CT cologram or barium enema is mandatory in all patients being considered for surgery.

319. A 22 year old man undergoes a splenectomy for an iatrogenic splenic injury. On the second post operative day a full blood
count is performed. Which of the following components of the full blood count is the first to be affected ?
A. Erythrocyte count
B. Reticulocyte count
C. Eosinophil count
D. Monocyte count
E. Lymphocyte count
Answer: B

Theme from January 2012 Exam.The granulocyte and platelet count are the first to be affected following splenectomy. Then
reticulocytes increase. Although a lymphocytosis and monocytosis are reported, these take several weeks to develop.

320. A 28 year old lady presents with benign cyclical mastalgia. Which of the following is not a recognised treatment for the
condition?
A. Evening primrose oil
B. Bromocriptine
C. Methotrexate
D. Danazol
E. Tamoxifen
99
Answer: C

Surgical excision of tender breast tissue is inappropriate. Methotrexate is used for the treatment of breast cancer. Whilst the use of
tamoxifen is of benefit other agents such as flaxseed oil or evening primrose oil should be tried first. Danazol is effective, but many
women dislike the side effects.
Benign cyclical mastalgia is a common cause of breast pain in younger females. It varies in intensity according to the phase of the
menstrual cycle. It is not associated with point tenderness of the chest wall (more likely to be Tietze's syndrome).
The underlying cause is difficult to pinpoint, examination should focus on identifying focal lesions (such as cysts) that may be treated
to provide symptomatic benefit. Women should be advised to wear a supportive bra. Conservative treatments include flax seed oil and
evening primrose oil. There is slightly more evidence in favor of flax seed oil, though neither has performed much better than placebo
in RCT's. Hormonal agents such as bromocriptine and danazol may be more effective. However, many women discontinue these
therapies due to adverse effects.

321. A 39 year old lady has undergone surgery for breast cancer. As part of the histopathology report the pathologist provides
the surgeon with a Nottingham Prognostic Index score of 6.4. He also states that the tumour size is 2cm. Which of the
following inferences can be made in relation to this statement?
A. The tumour is likely to be grade 1
B. Vascular invasion is present
C. Lymph node metastasis are definitely present
D. The tumour is oestrogen receptor positive
E. None of the above
Answer: C
A score of this value is unlikely to be reached with a grade 1 tumour and a size of 2cm. Therefore lymph node metastasis are definitely
present. In addition since the maximal score for lymph node metastasis is 3 the tumour is likely be of a higher grade (see below). The
Nottingham Prognostic Index provides no information about oestrogen receptor status or the presence or absence of vascular invasion.

Nottingham prognostic index: The Nottingham Prognostic Index can be used to give an indication of survival. In this system the
tumour size is weighted less heavily than other major prognostic parameters. Calculation of NPI: Tumour Size x 0.2 + Lymph node
score(From table below)+Grade score(From table below).
Lymph nodes involved Grade
Score
1 0 1
2 1-3 2
3 >3 3

Prognosis: Score to Percentage 5 year survival: 2.0 to 2.4: 93%. 2.5 to 3.4:85%. 3.5 to 5.4:70%. >5.4:50%. This data was
originally published in 1992. It should be emphasised that other factors such as vascular invasion and receptor status also impact on
survival and are not included in this data and account for varying prognoses often cited in the literature.

322. In patients with multiple endocrine neoplasia type IIb which of the following clinical appearances is the patient most likely to
display?
A. Acromegalic facies
B. Turners type features
C. Profound kyphoscoliosis
D. Multiple bony exostoses
E. Marfanoid features
Answer: E
Patients with MEN IIb may display Marfanoid features. It is unclear at the present time whether they have discrete changes in the
microfibrils of elastic fibres that are present in Marfans.

323. 32 year old man undergoes an appendicectomy. A large carcinoid tumour is identified and a completion right
hemicolectomy is performed. He is well for several months and then develops symptoms of palpitations and facial flushing.
Which of the following diagnostic markers should be requested?
A. Alpha feto protein
B. 5-Hydroxyindoleacetic acid
C. Urinary catecholamines
D. Urinary VMA measurements
E. None of the above

100
Answer: B
5 HIAA is the most commonly used diagnostic marker for carcinoid syndrome.

324. Which of the following breast tumours is most commonly associated with a risk of metastasis to the contralateral breast?
A. Invasive ductal carcinoma
B. Invasive lobular carcinoma
C. Phyllodes tumour
D. Pagets disease of the breast
E. Atypical ductal hyperplasia

Answer: B

Risk of metastasis to the contralateral breast is a classical feature of invasive lobular carcinoma.

325. With respect to oncogenes which statement is false?


A. Mutations in oncogenes lead to cell survival
B. Cells with oncogene mutations are resistant to cell necrosis
C. Oncogene mutations must typically involve two allelic mutations for biological effects to become manifest
D. May prevent cellular apoptosis
E. Include MYC mutations in the development of Burkitts Lymphoma
Answer: C
Tumour suppressor gene mutations tend to be recessive and thus twin allelic mutations are required as exemplified in the Knudson two
hit hypothesis in the development of retinoblastoma. Necrosis will occur at the centre of tumours that outgrow their blood supply and
induction of angiogenesis is a key feature of tumour progression.

326. Which is the characteristic finding on a blood film post splenectomy?


A. Stipple cell
B. Tear drop cell
C. Reticulocytes
D. Howell-Jolly bodies
E. Schistocyte
Answer: D
Blood film in hyposplenism: Howell-Jolly bodies. Pappenheimer bodies. Poikilocytes (Target cells). Erythrocyte containing siderotic
granules. Heinz bodies
Indications: Trauma: 1/4 are iatrogenic. Spontaneous rupture: EBV. Hypersplenism: hereditary spherocytosis or elliptocytosis etc.
Malignancy: lymphoma or leukaemia. Splenic cysts, hydatid cysts, splenic abscesses. Post splenectomy changes: Platelets will rise
first (therefore in ITP should be given after splenic artery clamped). Blood film will change over following weeks, Howell Jolly
bodies will appear. Other blood film changes include target cells and Pappenheimer bodies. Increased risk of post splenectomy sepsis,
therefore prophylactic antibiotics and pneumococcal vaccine should be given. Post splenectomy sepsis: Typically occurs with
encapsulated organisms. Opsonisation occurs but then not recognised

101

You might also like